!#" $ %& ' & ( % )!* !#+ , . /103254 637*839 6*9:054 ;3<:=5639 >58?. /1@ 7#65@ /A8*B 859:6 !#" $ %& ' & ( % )!* !#+ , . /103254 637*839 6*9:054 ;3<:=5639 >58?E 65F3;3G5>38*B 859:6 !#" $ %& ' & ( % )!* !#+ , . /103254 637*839 6*7*634 H305/A659 9:054 ;3<:=5639 >38 IJ63/1K:63@ /18#B 839:6 !#" $ %& ' & ( % )!* !#+ , L 639:;54 M 83>5039 N O P&QARTS U " ' V !* & W X Z >5;385/A>30T[#85F3G365/ \ X " \%& !*" $ &% ]' & & & % !? !*+ ,& . 85;34 0*^ 63_`83/V. @ G5M 0*^ 83/bac834 H50 d e f )% )" ' !# & )" " 3 " ' V !* & " & " 3 & , & % & , !# \ & W& ] d& kj l X W&% $ !*" (, , %& W& % & k kX W
C5D 5 -5D -5Y D5 D5Y 5g h 5g Y 5g i 5g m 5h n 5h C
Sociedade Brasileira de Matemática
o prqs Tt( ) u Tp Iniciamos este segundo ano da revista vVwyx vVz&{*| transmitindo nossa satisfação pela acolhida no primeiro ano de vida da revista, de toda a comunidade estudantil e dos diretores e professores dos colégios envolvidos. Agradecemos a todos os que têm apoiado esta iniciativa e esperamos continuar apoiando, através desta publicação, o trabalho dos professores. Nesta nova edição da revista vVwyx vVz&{*| aproveitamos para registrar a XX Olimpíada Brasileira de Matemática, da qual publicamos as provas da primeira, segunda e terceira fases com soluções (que, esperamos, serão úteis para a preparação para a XXI OBM), bem como as listas de premiados nos três níveis. Nesta edição, também publicamos artigos de dificuldade intermediária e material enviado por numerosos professores e alunos. Esperamos seguir recebendo colaborações dos nossos leitores: soluções dos problemas propostos, pequenos artigos e curiosidades matemáticas. Devido ao interesse manifestado por diversas pessoas, criamos recentemente as assinaturas individuais da revista vVwyx vVz&{*| . Para maiores informações, veja página 59 desta edição. Por outro lado, estamos planejando criar, nos próximos números, um pequeno espaço publicitário ligado ao ensino da matemática, para o qual aguardamos propostas de leitores, editoras e instituições de ensino. Desta forma, estaremos gerando recursos que ajudarão a manter a publicação da revista. Aproveitamos, por fim, para registrar que foi realizada em janeiro de 1999 a segunda Semana Olímpica, que reuniu premiados na XX Olimpíada Brasileira de Matemática nos 3 níveis e professores de vários estados. A atividade foi realizada em Maracanaú, Ceará, no centro de treinamento do Colégio 7 de Setembro, ao qual gostaríamos de agradecer pelo apoio.
Comitê Editorial.
EUREKA! N°4, 1999
2
Sociedade Brasileira de Matemática
}T} ~c s ( s ~ s c s 3 c k V T 3 ¡5 £¢¥¤y¦(§¨ ©«ª 01. Qual dos números a seguir é o maior? A) 345 B) 920 C) 2714
D) 2439
E) 8112
02. Um menino joga três dados e soma os números que aparecem nas faces voltadas para cima. O número dos diferentes resultados dessa adição é: A)12 B) 18 C) 216 D) 16 E) 15 03. Renata digitou um número em sua calculadora, multiplicou-o por 3, somou 12, dividiu o resultado por 7 e obteve o número 15. O número digitado foi: A) 31 B) 7 C) 39 D) 279 E) 27 04. Numa competição de ciclismo, Carlinhos dá uma volta completa na pista em 30 segundos, enquanto que Paulinho leva 32 segundos para completar uma volta. Quando Carlinhos completar a volta número 80, Paulinho estará completando a volta número: A) 79 B) 78 C) 76 D) 77 E) 75 05. Elevei um número positivo ao quadrado, subtrai do resultado o mesmo número e o que restou dividi ainda pelo mesmo número. O resultado que achei foi igual: A) Ao próprio número B) Ao dobro do número C) Ao número mais 1 D) À raiz quadrada do número E) Ao número menos 1 06. Quantos números de 3 algarismos existem cuja soma dos algarismos é 25 ? A) 2 B) 4 C) 6 D) 8 E) 10 07. João é mais velho que Pedro, que é mais novo que Carlos; Antônio é mais velho do que Carlos, que é mais novo do que João. Antônio não é mais novo do que João e todos os quatro meninos têm idades diferentes. O mais jovem deles é: A) João B) Antônio C) Pedro D) Carlos E) impossível de ser identificado a partir dos dados apresentados
EUREKA! N°4, 1999
3
Sociedade Brasileira de Matemática
08. Escreva um número em cada círculo da fila abaixo, de modo que a soma de três números quaisquer vizinhos (consecutivos) seja 12. 3
5
No último círculo à direita deve estar escrito o número: A) 3 B) 2 C) 1 D) 4
E) 7
09. Dezesseis cubos de 1cm de lado são colocados juntos, formando o paralelepípedo representado abaixo.
A superfície do mesmo foi pintada de verde e, em seguida, os cubos foram separados. O número de cubos com exatamente duas faces verdes é: A) 2 B) 6 C) 4 D) 8 E) 10 10. Uma fazenda retangular que possui 10 km de largura por 20 km de comprimento foi desapropriada para reforma agrária. Se a fazenda deve ser dividida para 200 famílias de modo que todas as famílias recebam a mesma área, então cada família deve receber: A) 1.000.000 m2 B) 100.000 m2 C) 5.000 m2 D) 1.000 m2 2 E) 10.000 m 11. Um estacionamento para carros cobra 1 real pela primeira hora e 75 centavos a cada hora ou fração de hora seguinte. André estacionou seu carro às 11h 20min e saiu às 15h 40min. Quantos reais ele deve pagar pelo estacionamento? A) 2,50 B) 4,00 C) 5,00 D) 4,75 E) 3,75 12. Para fazer 12 bolinhos, preciso exatamente de 100g de açúcar, 50g de manteiga, meio litro de leite e 400g de farinha. A maior quantidade desses bolinhos que serei capaz de fazer com 500g de açúcar, 300g de manteiga, 4 litros de leite e 5 quilogramas de farinha é: A) 48 B) 60 C) 72 D) 54 E) 42
EUREKA! N°4, 1999
4
Sociedade Brasileira de Matemática
13. Joãozinho brinca de formar quadrados com palitos de fósforo como na figura a seguir.
A quantidade de palitos necessária para fazer 100 quadrados é: A) 296 B) 293 C) 297 D) 301
E) 28
14. A soma de todos os números ímpares de dois algarismos menos a soma de todos os números pares de dois algarismos é: A) 50 B) 46 C) 45 D) 49 E) 48 15. O número que devemos somar ao numerador e subtrair do denominador 1478 da fração para transformá-la na sua inversa é: 5394 A) 3.916 B) 3.913 C) 3.915 D) 3.912 E) 3.917 16. O alfabeto usado no planeta X tem somente duas letras: X e x. O sobrenome (nome de família) de cada um de seus habitantes é uma seqüência formada por 4 letras. Por exemplo, xXxx é um possível sobrenome utilizado nesse planeta. O maior número de sobrenomes diferentes que podem ser dados no planeta X é: A) 12 B) 14 C) 15 D) 16 E) 18 17. João quer desfazer-se de sua coleção de 1.000 bolinhas. Para tanto escolhe dez garotos da rua onde mora. Dá ao primeiro garoto x bolinhas, ao segundo x + 1 bolinhas. Assim faz até chegar ao décimo garoto. Sempre dá uma bolinha a mais para o próximo garoto. No final, João ainda fica com um resto de bolinhas. Sendo x o número que deixa João com o menor resto possível, x é igual a: A) 94 B) 95 C) 96 D) 97 E) 98 18. No planeta Z todos os habitantes possuem 3 pernas e cada carro possui 5 rodas. Em uma pequena cidade desse planeta, existem ao todo 97 pernas e rodas. Então podemos afirmar: A) É possível que existam 19 carros nessa cidade B) Existem no máximo 16 carros nessa cidade EUREKA! N°4, 1999
5
Sociedade Brasileira de Matemática
C) Essa cidade tem 9 habitantes e 14 carros D) Essa cidade possui no máximo 17 carros E) Nessa cidade existem mais carros do que pessoas 19. São dados um tabuleiro e uma peça, como mostra a figura.
De quantas maneiras diferentes podemos colocar a peça no tabuleiro, de modo que cubra completamente 3 casas? A) 16 B) 24 C) 36 D) 48 E) 60 20. Pedro e Maria formam um estranho casal. Pedro mente às quartas, quintas e sextas-feiras, dizendo a verdade no resto da semana. Maria mente aos domingos, segundas e terças-feiras, dizendo a verdade no resto da semana. Certo dia, ambos dizem: ''Amanhã é dia de mentir''. O dia em que foi feita essa afirmação era: A) segunda-feira B) terça-feira C) sexta-feira D) sábado E) domingo
¬s ®k¯ °3±
01. Quantos são os números inteiros x que satisfazem à inequação 3< x < 7? A) 13 B) 26 C) 38 D) 39 E) 40 02. Hoje é sábado. Que dia da semana será daqui a 99 dias? A) segunda-feira B) sábado C) domingo D) sexta-feira E) quinta feira 03. Anulada. 04. Um pai tem 33 anos e seu filho, 7 anos. Depois de quantos anos a idade do pai será o triplo da idade do filho? A) 3 B) 7 C) 6 D) 9 E) 13
EUREKA! N°4, 1999
6
Sociedade Brasileira de Matemática
05. O quadrilátero ABCD é um quadrado de área 4m2. Os pontos M e N estão no meio dos lados a que pertencem. Podemos afirmar que a área do triângulo em destaque é, em m2,
A
N
D
A) 2
B) 1,5
C) 2,5
B
D) 3
06. Qual é o dígito das unidades do número 31998? A) 1 B) 3 C) 5 D) 7
M
C
E) 3,5
E) 9
07. Num código secreto, as 10 primeiras letras do nosso alfabeto representam os algarismos de 0 a 9, sendo que a cada letra corresponde um único algarismo e vice-versa. Sabe-se que d + d = f, d . d = f, c + c = d, c + d = a e a – a = b. Podemos concluir que a + b + c + d é igual a: A) 0 B) 2 C) 4 D) 6 E) 8 08. O número 1234a6 é divisível por 7. O algarismo a vale: A) 0 B) 2 C) 5 D) 6
E) 8
09. No trapézio abaixo, têm-se: AB paralelo a CD, AD = 10 cm e CD = 15 cm. O ângulo C mede 75º e o ângulo D, 30º . Quanto mede o lado AB, em centímetros?
A) 5
B) 7,5
C) 10
D) 12,5
E) 5 3
10. No quadrado mágico abaixo, a soma dos números em cada linha, coluna e diagonal é sempre a mesma. Por isso, no lugar do X devemos colocar o número:
EUREKA! N°4, 1999
7
Sociedade Brasileira de Matemática
A)30
B) 20
C) 35
D) 45
E) 40
11. Passarinhos brincam em volta de uma velha árvore. Se dois passarinhos pousam em cada galho, um passarinho fica voando. Se todos os passarinhos pousam, com três em um mesmo galho, um galho fica vazio. Quantos são os passarinhos? A) 6 B) 9 C) 10 D) 12 E) 15 12. Pelo menos quantos metros de barbante são necessários para amarrar 15 pacotes, conforme a figura, sabendo que cada pacote mede 10cm × 20cm × 40cm, sendo reservados 20cm para o laço?
A) 39
B) 36
C) 48
D) 56
E) 42
13. Para assistir ao filme Central do Brasil, cada um dos x alunos de uma turma deveria pagar y reais pelo frete do ônibus. Como faltaram 3 alunos, cada um dos alunos presentes teve que pagar 2 reais a mais para cobrir o preço do frete. Qual foi esse preço? A) (x + 3)(y – 2) B) (x – 3) y + 2 C) x(y + 2) – 3 D) xy – 6 E) (x – 3)(y + 2) 14. Seu Horácio resolveu incrementar a venda de CDs em sua loja e anunciou uma liquidação para um certo dia, com descontos de 30% sobre o preço das etiquetas. Acontece que, no dia anterior à liquidação, seu Horário aumentou o preço marcado nas etiquetas, de forma que o desconto verdadeiro fosse de apenas 9%. De quanto foi o aumento aplicado por seu Horácio? A) 30% B) 39% C) 21% D) 40% E) 31% 15. Um fabricante de brinquedos embala bolas de pingue-pongue em dois tipos de caixas. Num dos tipos ele coloca 10 bolas e no outro coloca 24 EUREKA! N°4, 1999
8
Sociedade Brasileira de Matemática
bolas. Num certo dia foram embaladas 198 bolas e usadas mais de 10 caixas. Quantas caixas foram feitas nesse dia? A)14 B) 16 C) 15 D) 17 E) 11 16. Coloque em cada quadradinho, no desenho a seguir, os algarismos 1, 2, 3, 4 ou 5, de forma que cada um deles apareça pelo menos uma vez e que o número formado seja o maior possível e múltiplo de 9.
No número que você construiu, o algarismo mais repetido apareceu: A) 6 vezes B) 5 vezes C) 4 vezes D) 3 vezes E) 2 vezes 17. Observe as igualdades a seguir: 32 + 42 = 52 52 + 122 = 132 7 2 + 242 = 252 92 + 402 = 412 ...
Considere a igualdade 17 + x = y 2 com base nos exemplos anteriores, procure determinar os números naturais x e y. Podemos concluir que x + y é igual a: A) 289 B) 121 C) 81 D) 144 E) 196 2
2
18. Você vai pintar a bandeira abaixo utilizando 4 cores: azul, verde, amarelo e vermelho, uma em cada região.
Se o vermelho e o amarelo não podem ficar juntos, de quantas maneiras pode ser pintada a bandeira? A) 12 B) 4 C) 18 D) 20 E) 16 19. Um crime é cometido por uma pessoa e há quatro suspeitos: André, Eduardo, Rafael e João. Interrogados, eles fazem as seguintes declarações:
EUREKA! N°4, 1999
9
Sociedade Brasileira de Matemática
•André: Eduardo é o culpado. •Rafael: Eu não sou culpado.
•Eduardo: João é o culpado. •João: Eduardo mente quando diz que eu sou culpado. Sabendo que apenas um dos quatro disse a verdade, quem é o culpado? A) André. B) Eduardo. C) Rafael. D) João. E) Não se pode saber. 20. Anulada.
¬s ®k¯ °3² 01. Veja Problema 1 Nível 2. 03. Veja Problema 5 do Nível 2. 05. Veja Problema 15 do Nível 2. 06.- 0,4444... = A) 0,2222… B) 0,3333…
02. Veja Problema 2 do Nível 2. 04. Veja Problema 6 do Nível 2.
C) 0,4444…
D) 0,5555…
E) 0,6666…
07.- Veja Problema 8 do Nível 2. 08.- Todos os ângulos internos de um polígono convexo são menores que (não podendo ser iguais a) 160°. O número de lados desse polígono é, no máximo, igual a: A) 12 B) 14 C) 15 D) 17 E) 18 09.- A média aritmética de seis números é 4. Quando acrescentamos um sétimo número, a nova média é 5. O número que foi acrescentado é: A) 5 B) 6 C) 8 D) 10 E) 11 10. Veja Problema 19 do Nível 2. 11.- Em uma calculadora, a tecla A transforma o número x que está no visor em
1 e a tecla B multiplica por 2 o número que está no visor. Se o número x
2 está no visor e digitamos a seqüência ABABABAB...AB (total de digitações: 998), obteremos no visor um número que é igual a: A) 1 B) 2-498 C) 2-500 D) 2499 E) 2500
EUREKA! N°4, 1999
10
Sociedade Brasileira de Matemática
12.- Um número inteiro n é bom quando 4n + 1 é um múltiplo de 5. Quantos números bons há entre 500 e 1.000? A) 50 B) 51 C)100 D) 101 E) 102 13.- Em um conjunto de pontos do espaço, a distância entre dois pontos diferentes quaisquer é igual a 1. O número máximo de pontos que pode haver nesse conjunto é: A) 2 B) 3 C) 4 D) 6 E) 8 14.- Se x homens fazem x embrulhos em x segundos, em quantos segundos y homens farão y embrulhos? A) y
B) x
x2 C) y
D)
y2 x
E)
y x
15.- Você entra em um restaurante para comer pizza e espera pagar uma quantia proporcional à quantidade de comida pedida. Se uma pizza com 20 cm de diâmetro custa R$ 3,60, quanto você espera pagar por uma outra do mesmo sabor com 30cm de diâmetro? A) R$ 5,40 B) R$ 5,80 C) R$ 6,60 D) R$ 7,50 E) R$ 8,10 16.- A função f associa a cada real x o menor elemento do conjunto 15 − x x + 1, . O valor máximo de f(x) é: 2 A) 4 B) 5 C) 11/2 D) 16/3 E) 19/4 17.- Vendi dois rádios por preços iguais. Em um deles tive lucro de 25% sobre o preço de compra e no outro tive prejuízo de 25%. Em relação ao capital investido: A) não tive lucro nem prejuízo B) lucrei 6,25% C) lucrei 16% D) tive prejuízo de 6,25% E) tive prejuízo de 16% 18.- A respeito da resposta de um problema, Maurício, Paulo, Eduardo e Carlos fizeram as seguintes afirmações: – Maurício: É maior que 5. – Paulo: É menor que 10. – Eduardo: É um número primo. – Carlos: É maior que 12. Entre as afirmações acima, quantas, no máximo, podem ser verdadeiras? A) 0 B) 1 C) 2 D) 3 E) 4
EUREKA! N°4, 1999
11
Sociedade Brasileira de Matemática
1 ≤ 2 são: x D) x ≥ 1
x+
19.- Os valores reais de x que satisfazem a inequação A) –1 ≤ x ≤ 1 E) x ≤ 2
C) x ≤ 1
B) x = 1
20.- De quantos modos se pode colocar na tabela abaixo duas letras A, duas letras B e duas letras C, uma em cada casa, de modo que não haja duas letras iguais na mesma coluna?
A) 12
B) 24
C) 36
D) 48
E) 64
21.- Um viajante deveria caminhar durante uma hora num sentido entre o norte e o leste, fazendo 300 com o norte. Atrapalhou-se e caminhou uma hora num sentido entre o norte e o oeste, formando 300 com o norte. Para chegar ao seu destino, ele deve agora tomar um rumo que faça com o norte um ângulo de: A) 00 B) 300 C) 450 D) 600 E) 900 22.- Barcas vão do Rio a Niterói em 25 minutos e lanchas fazem a viagem em 15 minutos. A que horas a barca que partiu do Rio às 10h 01min é alcançada pela lancha que saiu do Rio às 10h 07min? A) 10h 15min B) 10h 16min C) 10h 17min D) 10h 18min E)10h 20min 23.- Veja Problema 17 do Nível 2. 3 2 24.- A soma das raízes reais de x + 3x + 3x − 1 = 0 é:
A) –3
B) 1 − 3 2
C) 1
D)
3
2 −1
E) 3
25.- Dado um cubo, considere o conjunto de 27 pontos formado pelos vértices desse cubo, pelos pontos médios de suas arestas, pelos centros de suas faces e pelo centro do cubo. Quantas são as retas que passam por três desses pontos? A) 49 B) 54 C) 63 D) 81 E) 108
EUREKA! N°4, 1999
12
Sociedade Brasileira de MatemĂĄtica
Respostas NĂvel 1: 01.- E 06.- C 02.- D 07.- C 03.- A 08.- A 04.- E 09.- D 05.- E 10.- A
11.- B 12.- B 13.- D 14.- C 15.- A
16.- D 17.- B 18.- D 19.- C 20.- B
Respostas NĂvel 2: 01.- D 06.- E 02.- C 07.- D 03.- Anulada 08.- D 04.- C 09.- A 05.- B 10.- B
11.- B 12.- B 13.- E 14.- A 15.- D
16.- B 17.- A 18.- A 19.- C 20.- Anulada
Respostas NĂvel 3: 01.- D 06.- E 02.- C 07.- D 03.- B 08.- D 04.- E 09.- E 05.- D 10.- C
11.- A 12.- C 13.- C 14.- B 15.- E
16.- D 17.- D 18.- D 19.- B 20.- D
21.- E 22.- B 23.- A 24.- D 25.- A
)Âł ´ ¾¡œš¸ Âş#½ŸAžĂ&#x20AC;Âż Ă Ă&#x192;Ă&#x201A;5Ă&#x201E;ÂŁĂ&#x2026; Ă&#x2020; Ă&#x2021;Â?Ă&#x2C6; Ă&#x2030;?Ă&#x160;3Ă&#x2039;Â?Ă&#x2026;3Ă&#x152;Ă?Ă&#x2026; Ă&#x17D;VĂ?Ă?Ă&#x2026;5Ă&#x2018; Ă&#x2C6;Ă&#x2019;Ă&#x2021; Ă&#x201E;kĂ&#x2C6; Ă?JĂ&#x2026; Ă&#x152;VĂ&#x201E; Ă&#x201C;)Ă&#x2026;3Ă&#x201D;:Ă&#x201E;Ă?Ă&#x2030;*Ă&#x2022;3Ă&#x201D;ÂĽĂ&#x2C6; Ă&#x2013;5Ă&#x2026;ÂŁĂ&#x201D;:Ă&#x201E; Ă&#x2030;Ă&#x2014;Ă&#x201A;3Ă&#x2030;#Ă&#x2026;Ă&#x2DC;Ă&#x2021;(Ă&#x2C6; Ă&#x2018;3Ă&#x201D;:Ă&#x2026;]Ă&#x201E; Ă&#x2021; Ă&#x201E; Ă&#x201D;Ă?Ă?JĂ&#x2122; Ă&#x161; Ă&#x2C6; Ă&#x2013;5Ă&#x2026; Ă&#x152;Ă?Ă&#x201E;Ă&#x203A;Ă&#x152;Ă&#x153;Ă&#x2C6; Ă&#x2018;3Ă&#x2013;ÂĽĂ&#x201A;5Ă&#x2018;Ă?Ă&#x2018;5Ă&#x17E;5Ă&#x; Ă Våãâ¼ä:ĂĽVĂŚ ç ĂĄ è*ĂŠ5ĂŞĂŤĂ VĂĄĂŁĂŹ ĂŠÂĽĂ:ĂĄVè#ĂŽ5ĂÂĽĂŻ Ă°Ă?ĂŠVĂąÂ&#x2018;ĂŠ Ìòå äJĂ:ÊóÊôĂ:üþà Ă?ü¼êÜü¼êÜÊĂ?ç áþø5ü¼êÜåÚâ¼äĂ?üþúĂ?åòê5ĂŞĂ?üòä:åòê Ăť Ăź5Ă˝:Ăž Ăż: Ăž
ß5ý�ÿ:ÞT Þ ß� ý þ ý ß
! " $#% ! '&( )!"+*
EUREKA! N°4, 1999
13
Sociedade Brasileira de MatemĂĄtica
Â&#x2030; u o pTtÂ&#x201A;q? tÂ&#x201A;u o rÂ&#x2019; o Â&#x2019;0/ )tc o } } Ă&#x2DC;qst Â&#x2019; ,.- o o 1 3 Â&#x17E; 254 637 Â&#x;ÂŁÂ ÂĽÂ&#x;ò¥3Â&#x17E; ¢þ¤Â&#x2C6;ÂŚĂ&#x2019;§Â?Â&#x17E; Šª " ' V !* 98 JoĂŁo comprou um livro e reparou que ele tinha 200 pĂĄginas. Seu irmĂŁo mais novo arrancou ao acaso 25 folhas e somou os nĂşmeros das 50 pĂĄginas. Explique porque o resultado desta soma nĂŁo pode ser igual a 1998. Atenção: cada folha tem duas pĂĄginas. A primeira folha tem as pĂĄginas 1 e 2, a segunda folha tem as pĂĄginas 3 e 4, e assim por diante.
;: < = > ? :
Como cada folha contĂŠm duas pĂĄginas tais que a soma dos seus respectivos nĂşmeros ĂŠ Ămpar, ao adicionarmos todos esses 25 nĂşmeros, obteremos necessariamente uma soma Ămpar que, portanto, nĂŁo pode ser igual a 1998.
" ' V !* 9@
Que fraçþes devem ser retiradas da soma
1 2
+
1 4
+
1 6
+
1 8
+
1 10
+
1 12
para que
a soma das restantes seja igual a 1?
;: < = > ? : 1 2
+
1 4
+
1 6
+
1 8
+
1 10
+
1 12
=
60 120
+
30 120
+
20 120
+
15 120
+
12 120
+
10
(*)
120
Uma vez que 60 + 30 + 20 + 10 = 120, ĂŠ claro que podemos remover 15 120
=
1
e
8
12 120
=
1
. (alĂŠm disso, vĂŞ-se claramente no lado direito da
10
igualdade (*) que nĂŁo existem outros termos cuja soma seja igual a
27
)
120
Assim, devemos remover
" ' V !* 9A
1 8
e
1
.
10
Encontre dois nĂşmeros de trĂŞs algarismos cada um, usando cada um dos dĂgitos 1, 2, 3, 4, 5, 6 exatamente uma vez, de forma que a diferença entre eles (o maior menos o menor) seja a menor possĂvel.
EUREKA! N°4, 1999
14
Sociedade Brasileira de MatemĂĄtica
;: < = > ? : Para que a diferença seja a menor possĂvel, os nĂşmeros devem ser os mais prĂłximos possĂveis. Assim, os algarismos das centenas devem ser consecutivos. A melhor escolha ĂŠ aquela em que as dezenas formadas pelos algarismos restantes tenham a maior diferença possĂvel, o que ocorre para as dezenas 65 e 12. Assim, os algarismos das centenas devem ser 3 e 4. O menor nĂşmero começado por 4 ĂŠ 412 e o maior começado por 3 ĂŠ 365, cuja diferença ĂŠ 47.
" ' V !* 9B
Existem casas em volta de uma praça. João e Pedro dão uma volta na praça, caminhando no mesmo sentido e contando as casas. Como não começaram a contar da mesma casa, a 5ª. casa de João Ê a 12ª. de Pedro e a 5ª. casa de Pedro Ê a 30ª. de João. Quantas casas existem em volta da praça?
;: < = > ? : Sejam Jn e Pn respectivamente as n-ĂŠsimas casas de JoĂŁo e Pedro. De J5 a J30 exclusive, existem 30 â&#x20AC;&#x201C; 5 â&#x20AC;&#x201C; 1 = 24 casas. De P5 a P12 exclusive existem 12 â&#x20AC;&#x201C; 5 â&#x20AC;&#x201C; 1 = 6. Logo, no total existem 24 + 6 + 2 = 32 casas.
" ' V !* 9C
Existem 20 balas sobre uma mesa e duas crianças começam a comê-las, uma criança de cada vez. Em cada vez, cada criança deve comer pelo menos uma bala e estå proibida de comer mais que a metade das balas que existem sobre a mesa. Nesta brincadeira, ganha a criança que deixar apenas uma bala sobre a mesa. Qual das duas crianças pode sempre ganhar na brincadeira: a primeira ou a segunda a jogar? Como deve fazer para ganhar?
;: < = > ? :
Ganha a primeira criança. No inĂcio ele deve comer 5 balas, deixando 15 balas sobre a mesa. A segunda criança deve comer no mĂnimo uma e no mĂĄximo 7 balas, sobrando entre 8 e 14 balas sobre a mesa. Em qualquer caso a primeira criança pode comer algumas balas, deixando exatamente 7 sobre a mesa. A segunda criança agora deve comer entre uma e trĂŞs balas, deixando de 4 a 6 balas sobre a mesa. A primeira criança agora come algumas delas, deixando exatamente 3 balas, forçando a segunda criança a comer uma. Comendo mais uma apĂłs isso, a primeira criança acaba deixando apenas uma bala no final e ganhando o jogo. De um modo mais geral, a estratĂŠgia ganhadora consiste em deixar o adversĂĄrio com 2k â&#x20AC;&#x201C; 1 balas, para algum k â&#x2C6;&#x2C6; D
. O adversĂĄrio ĂŠ obrigado a comer de 1 a ( 2 k â&#x2C6;&#x2019;1 â&#x2C6;&#x2019; 1) balas, deixando sobre a
EUREKA! N°4, 1999
15
Sociedade Brasileira de MatemĂĄtica
mesa um nĂşmero de balas que estĂĄ sempre entre 2kâ&#x20AC;&#x201C;1 e 2k â&#x20AC;&#x201C; 2. O primeiro jogador pode, entĂŁo, jogar novamente de modo a deixar o adversĂĄrio com 1 1 2 k â&#x2C6;&#x2019; â&#x2C6;&#x2019; 1 balas. O processo prossegue atĂŠ o adversĂĄrio ser reduzido a 2 â&#x20AC;&#x201C; 1 = 1 bala.
" ' V !* 9E
Pintam-se de preto todas as faces de um cubo de madeira cujas arestas medem 10 centĂmetros. Por cortes paralelos Ă s faces, o cubo ĂŠ dividido em 1.000 cubos pequenos, cada um com arestas medindo 1 centĂmetro. Determine: a) O nĂşmero de cubos que nĂŁo possuem nenhuma face pintada de preto. b) O nĂşmero de cubos que possuem uma Ăşnica face pintada de preto. c) O nĂşmero de cubos que possuem exatamente duas faces pintadas de preto. d) O nĂşmero de cubos que possuem trĂŞs faces pintadas de preto.
;: < = > ? :
EstĂŁo sem nenhuma face pintada, os cubos interiores ao cubo maior. Portanto devem ser retiradas uma fila de cima e uma fila de baixo, uma da frente e outra de trĂĄs, e uma de cada lado, ficando assim com um cubo de aresta 8 que contĂŠm 83 = 512 cubos pequenos. a)
b)
c)
EstĂŁo com uma face pintada aqueles que pertencem a uma face mas nĂŁo possuem lado comum com a aresta do cubo maior, isto ĂŠ, 82 = 64 em cada face. Como sĂŁo seis faces, temos 6 Ă&#x2014; 64 = 384 cubos pequenos. EstĂŁo com duas faces pintadas aqueles que estĂŁo ao longo de uma aresta mas nĂŁo no vĂŠrtice do cubo maior, isto ĂŠ, 8 cubos em cada aresta. Como sĂŁo 12 arestas, temos 8 Ă&#x2014; 12 = 96 cubos pequenos. EstĂŁo com 3 faces pintadas aqueles que estĂŁo nos vĂŠrtices do cubo maior, ou seja, 8 cubos pequenos.
" ' V !* 98
s Žk¯ °3¹
1 1 1 1 1 1 + + + + + para 2 3 4 6 8 10 12 que a soma das restantes seja igual a 1? Dê todas as soluçþes. Que fraçþes devem ser retiradas da soma
EUREKA! N°4, 1999
16
1
+
Sociedade Brasileira de MatemĂĄtica
;: < = > ? : 1 2
+
1 60 30 20 15 12 10 1 1 1 1 1 40 + + + + + = + + + + + + (*) 3 4 6 8 10 12 120 120 120 120 120 120 120
Devemos escrever 120 como soma de algumas parcelas 60, 40, 30, 20, 15, 12, 10. As soluçþes possĂveis sĂŁo 60 + 40 + 20 = 120 60 + 30 + 20 + 10 = 120. Assim, podemos remover
1 1 1 1 1 1 1 , , e ou , e . 4 8 10 12 3 8 10
Evidentemente 15 e 12 nĂŁo podem aparecer, pois a soma nĂŁo seria mĂşltipla de 10 nesse caso.
" ' V !* 9@
Veja Problema 3 do NĂvel 1.
" ' V !* 9A
Cinco cartĂľes numerados com 3, 4, 5, 6 e 7, respectivamente, sĂŁo colocados em uma caixa. Os cartĂľes sĂŁo retirados da caixa, um de cada vez e colocados sobre a mesa. Se o nĂşmero de um cartĂŁo retirado ĂŠ menor do que o nĂşmero do cartĂŁo imediatamente anterior, entĂŁo este cartĂŁo imediatamente anterior ĂŠ colocado de volta na caixa. O procedimento continua atĂŠ que todos os cartĂľes estejam sobre a mesa. Qual ĂŠ o nĂşmero mĂĄximo de vezes que retiramos cartĂľes da caixa?
;: < = > ? :
O número måximo de vezes que retiramos cartþes da caixa Ê 15, o que corresponde à seqßência de cartþes retirados 7, 6, 5, 4, 3, 7, 6, 5, 4, 7, 6, 5, 7, 6, 7. De fato, dentre os primeiros 5 cartþes hå necessariamente um que Ê menor que o cartão seguinte, e que portanto não voltarå mais para a caixa, o mesmo acontecendo para pelo menos um cartão dentre os 4 seguintes, depois para pelo menos um dentre os 3 seguintes, depois para pelo menos um dentre os dois seguintes, sobrando no måximo um cartão, que serå o último a ser retirado da caixa.
" ' V !* 9B
Em um triângulo acutângulo ABC o ângulo interno de vÊrtice A mede 300. Os pontos B1 e C1 são os pÊs das alturas traçadas por B e C, respectivamente e os pontos B2 e C2 são mÊdios dos lados AC e AB, respectivamente. Mostre que os segmentos B1C2 e B2C1 são perpendiculares.
EUREKA! N°4, 1999
17
Sociedade Brasileira de MatemĂĄtica
;: < = > ? : O segmento B1 C2 ĂŠ uma mediana do triângulo AB1 B e portanto AC2 = B1 C2 e C2 BË&#x2020;1 A = B AË&#x2020; C = 30°.
A 30°
DaĂ BCË&#x2020; 2 B1 = C 2 BË&#x2020;1 A + BAË&#x2020; C = 60°. Analogamente, ACË&#x2020; B = 30°. Finalmente 1
C2
2
C1OË&#x2020; C 2 = 180° â&#x2C6;&#x2019; BCË&#x2020; 2 B1 â&#x2C6;&#x2019; ACË&#x2020; 1 B 2 = 90°
C1 °
°
O °
°
B2 °
" ' V !* 9C
B
B1
C
Veja Problema 5 do NĂvel 1.
" ' V !* 9E
Pintam-se de preto todas as faces de um cubo de madeira cujas arestas medem n centĂmetros onde n â&#x2030;Ľ 3. Por cortes paralelos Ă s faces, o cubo ĂŠ dividido em n3 cubos pequenos, cada um com arestas medindo 1 centĂmetro. Sabendo que o nĂşmero total de cubos pequenos com exatamente uma face pintada de preto ĂŠ igual ao nĂşmero de cubos pequenos apresentando todas as faces sem pintura, determine o valor de n.
;: < = > ? :
Um cubo pequeno que nĂŁo possui qualquer face pintada provĂŠm do interior do cubo grande. Isto significa que esse cubo pequeno ĂŠ parte de um cubo de lado n â&#x20AC;&#x201C; 2, obtido quando retiramos uma unidade de cada face do cubo original. Assim, existem (n â&#x20AC;&#x201C; 2)3 cubos pequenos nĂŁo pintados. Por outro lado, um cubo pequeno com uma face pintada provĂŠm da face do cubo original, mas nĂŁo tendo qualquer parte da aresta deste cubo. Assim, existem 6(n â&#x20AC;&#x201C; 2)2 cubos pequenos com face pintada. Portanto, (n â&#x20AC;&#x201C; 2)3 = 6(n â&#x20AC;&#x201C; 2)2, com n > 2. Logo, n â&#x20AC;&#x201C; 2 = 6, ou seja, n = 8.
s Žk¯ °3²
" ' V !* 98 Veja Problema 3 do NĂvel 2.
" ' V !* 9@
Veja problema 5 do NĂvel 1. EUREKA! N°4, 1999
18
Sociedade Brasileira de MatemĂĄtica
" ' V !* 9A Uma reta que passa pelos pontos mÊdios de dois lados opostos de um quadrilåtero convexo forma ângulos iguais com ambas as diagonais. Mostre que as duas diagonais têm o mesmo comprimento.
;: < = > ? :
Sejam ABCD o quadrilĂĄtero, M,N,P e Q os pontos mĂŠdios dos lados AB, BC, CD e DA, respectivamente. MN e PQ sĂŁo paralelos Ă diagonal AC e medem a metade de seu comprimento, enquanto NP e QM sĂŁo paralelos Ă diagonal BD e medem a metade de seu comprimento. Assim, MNPQ ĂŠ um paralelogramo. As condiçþes do problema dizem que a reta que passa pelos pontos mĂŠdios de dois lados opostos de ABCD (digamos MP , sem perda de generalidade) formam ângulos iguais com AC e BD , portanto com PQ e NP , donde MP ĂŠ bissetriz de NPË&#x2020; Q . Logo MNPQ deve ser um losango,
donde MN = NP , e portanto AC = BD (pois MN = AC / 2 e NP = BD / 2 ).
" ' V !* 9B Sobre os lados AB e AC de um triângulo acutângulo ABC sĂŁo construĂdos, exteriormente ao triângulo, semicĂrculos tendo estes lados como diâmetros. As retas contendo as alturas relativas aos lados AB e AC cortam esses semicĂrculos nos pontos P e Q. Prove que AP = AQ.
;: < = > ? :
Sejam M o pÊ da altura relativa ao lado AB. Como o triângulo APB Ê retângulo em P, e PM Ê a altura de P em relação a AB temos 2
AP = AB â&#x2039;&#x2026; AM = AB â&#x2039;&#x2026; AC â&#x2039;&#x2026; cos BAË&#x2020; C .
Analogamente
mostra-se
2
AQ = AC â&#x2039;&#x2026; AB â&#x2039;&#x2026; cos BAË&#x2020; C . Portanto, AP = AQ.
" ' V !* 9C
Seja f : F â&#x2020;&#x2019; G uma função tal que f(1) = 999 e f(1) + f(2) + ... + f(n) = n2â&#x2039;&#x2026;f(n) para todo n inteiro positivo. Determine o valor de f(1998).
EUREKA! N°4, 1999
19
que
Sociedade Brasileira de MatemĂĄtica
;: < = > ? : Calculemos alguns valores de f(n): 2 f (1) = 999; f (1) + f ( 2) = 2 â&#x2039;&#x2026; f ( 2) â&#x2021;&#x2019; 3 f ( 2) = 999 â&#x2021;&#x2019; f ( 2) = 333 333 2 f (1) + f ( 2) + f (3) = 3 â&#x2039;&#x2026; f (3) â&#x2021;&#x2019; 8 f (3) = 999 + 333 â&#x2021;&#x2019; f (3) = 2 333 999 2 â&#x2021;&#x2019; f ( 4) = f (1) + f ( 2) + f (3) + f ( 4) = 4 â&#x2039;&#x2026; f ( 4) â&#x2021;&#x2019; 15 f ( 4) = 999 + 333 + 2 10 999 999 999 999 , f ( 2) = , f (3) = , f ( 4) = , e ĂŠ razoĂĄvel Assim, temos f (1) = 1 3 6 10 999 1998 = conjecturar que f ( n) = Para todo n â&#x2C6;&#x2C6; H . Vamos 1 + 2 + ... + n n ( n + 1)
provar esse fato: Para n â&#x2030;Ľ 2 temos f (1) + f ( 2) + ... + f ( n ) = n 2 f ( n ) â&#x2021;&#x2019; ( n 2 â&#x2C6;&#x2019; 1) f ( n) = f (1) + f ( 2) + ... + f ( n â&#x2C6;&#x2019; 1) â&#x2021;&#x2019; f (n) =
f (1) + f ( 2) + ... + f ( n â&#x2C6;&#x2019; 1) . n2 â&#x2C6;&#x2019;1
Por hipĂłtese de indução, â&#x2C6;&#x2019;
1998
f (1) + f ( 2) + ... f (n â&#x2C6;&#x2019; 1) =
1998
f (k ) =
1998 k ( k + 1)
=
1998 k
k +1
, para k = 1, 2, â&#x20AC;Ś, n â&#x20AC;&#x201C; 1, e portanto â&#x2C6;&#x2019;
1998
+
1998
â&#x2C6;&#x2019;
1998
+ ... +
1998
â&#x2C6;&#x2019;
1998
=
1998
2 2 3 1 n â&#x2C6;&#x2019;1 n 1998 1998( n â&#x2C6;&#x2019; 1) 1998(n â&#x2C6;&#x2019; 1) 1998 2 = â&#x2021;&#x2019; f (n) = = pois n â&#x2C6;&#x2019; 1 = ( n â&#x2C6;&#x2019; 1)(n + 1)), 2 n n n ( n + 1) n ( n â&#x2C6;&#x2019; 1) 1
â&#x2C6;&#x2019;
como querĂamos demonstrar. Fazendo n = 1998 temos f(1998) =
" ' V !* 9E
1998 1998 â&#x2039;&#x2026; 1999
=
1
.
1999
O menor mĂşltiplo de 1998 que possui apenas os algarismos 0 e 9 ĂŠ 9990. Qual ĂŠ o menor mĂşltiplo de 1998 que possui apenas os algarismos 0 e 3?
;: < = > ? : 1998 = 2 Ă&#x2014; 999 = 2 Ă&#x2014; 33 Ă&#x2014; 37. Um nĂşmero formado apenas pelos algarismos 0 e 3 ĂŠ mĂşltiplo de 33 se e somente se o nĂşmero de algarismos 3 ĂŠ mĂşltiplo de 9 (pois ao dividi-lo por 3 obtemos um nĂşmero que possui apenas os EUREKA! N°4, 1999
20
Sociedade Brasileira de MatemĂĄtica
algarismos 0 e 1 que deve ser mĂşltiplo de 9, o que ocorre se e sĂł se o nĂşmero de algarismos 1 ĂŠ mĂşltiplo de 9 ). Assim, o nĂşmero desejado deve ter pelo menos 9 algarismos 3, e deve terminar por 0, por ser par. O menor nĂşmero com essas propriedades ĂŠ 3333333330, que ĂŠ mĂşltiplo de 1998 pois ĂŠ par, ĂŠ mĂşltiplo de 33 e ĂŠ mĂşltiplo de 37 por ser mĂşltiplo de 111 (ĂŠ igual a 111 Ă&#x2014; 30030030)).
IKJMLONQPSR TVUWRYX Z\[ ]_^a` b c c dfe ]hgjilk ] b m n o prq ] s k ]3tKu5n e n u5n m [rq pav b;gfn s w p'u ]yx [ u ]zs [rq{s |rq ]3m b }3~ ~ Â&#x20AC;  Â&#x192;Â&#x201A; Â&#x201E;Â&#x2020;Â&#x2026; Â&#x201E; Â&#x201A;Â&#x2C6;Â&#x2021; Â&#x2030;  Â&#x160; Â&#x2039;(Â&#x152; Â&#x201E;3Â?Â&#x17D; 3 Â&#x20AC; Â? Â? Â&#x2018;  Â&#x160;Â&#x201C;Â&#x2019;  Â?Â&#x201D;Â?  Â&#x2022; ~ Â?Â&#x17D;Â&#x201E; Â&#x20AC;  Â&#x160;Â&#x201C;Â&#x152; Â&#x2013;fÂ&#x2014;;~ Â&#x201A;  Â&#x160;Â&#x2DC;Â&#x201E;Â&#x2122; 3Â&#x161; Â&#x20AC; Â&#x201E; Â&#x152;Â&#x153;Â&#x203A;rÂ?Â&#x17D;Â&#x2021;Â&#x2122;Â&#x161;  ;Â&#x2018; Â&#x2021; Â?!Â&#x160; Â&#x201E;3Â?zÂ&#x161;3Â&#x203A; Â&#x152; Â&#x2021;rÂ&#x2019;  Â&#x160; Â&#x17E;Â&#x201C;Â&#x201A;  Â&#x20AC; Â&#x2021; Â?Â&#x17D;Â&#x2021; rÂ&#x;r Â&#x201A; Â&#x2021; Â&#x20AC; Â&#x201E;3Â&#x152; Â&#x152; Â&#x2021; Â&#x2019;  rÂ?ÂĄÂ&#x203A; Â? Â&#x2021;¢Â?Â&#x17D;Â&#x2021;r Â&#x2021; Â&#x201A; Â&#x2021;ÂŁÂ&#x201A; Â&#x201E;¤Â&#x201E;ÂĽÂ&#x2018; Â&#x201E; } Â&#x2021; Â&#x20AC; Â&#x201E; Â&#x160;+Â&#x201E;3Â? Â&#x2019;3~ Â?Â&#x17D;Â&#x2021; ÂŚ
§r¨ ¨ ŠYÂŞÂŹÂŤ Š  ¨ ÂŽ ÂŻ ° ÂąÂ&#x2C6; 3² ÂłY´Â&#x2C6;Š ÂŤ Âľ Âł Âś Âł ´Â&#x2C6;Š ¡ ¸ š ¸ ÂşÂ&#x2C6;Âť Ÿ¼½º ž'Âż;¸ Ă&#x20AC; ¸rĂ Ă&#x201A;Ă&#x201E;Ă&#x192;SĂ&#x2026;r¸rš Ă&#x2020; Ă&#x2021;Â&#x2C6;Ă&#x2C6; Ă&#x2030;
Ă&#x160;KĂ&#x2039;MĂ&#x152;OĂ?QĂ&#x17D;SĂ? Ă?VĂ&#x2018;WĂ?YĂ&#x2019; Ă&#x201C;\Ă&#x201D; Ă&#x2022; Ă&#x2013;Ă&#x2DC;Ă&#x2014; Ă&#x2013; Ă&#x2122; Ă&#x161;Ă&#x203A;Ă&#x201D;rĂ&#x2122; Ă&#x153; Ă&#x2013;VĂ? Ă&#x2022;.Ă&#x17E; Ă&#x;Â&#x2C6;Ă Ă&#x2013; ĂĄ Ă&#x2022; âVĂŁ Ă&#x2013; â ä Ă&#x153; ä Ă&#x17E; Ă&#x2013; â+Ă&#x; â â Ă&#x201D; ĂĽVä Ă? Ă&#x2013; âVĂŁ Ă&#x2022;hĂ Ă&#x2013;.ĂŁ Ă&#x2013;;Ă fä Ă&#x2122; ĂŚ ĂĽĂ&#x2DC;ä Ă&#x2013;
(k + 2){1â&#x20AC;&#x201C; ( [wz + h + j â&#x20AC;&#x201C; q]2 + [ (gk + 2g + k + 1) â&#x2039;&#x2026; (h + j) + h â&#x20AC;&#x201C; z]2 + [16(k + 1)3 â&#x2039;&#x2026; (k + 2) (n + 1)2 + 1 â&#x20AC;&#x201C; f 2]2 + [2n + p + q + z â&#x20AC;&#x201C; e]2 + [e3 â&#x2039;&#x2026; (e + 2) â&#x2039;&#x2026; (a + 1)2 + 1 â&#x20AC;&#x201C; o2]2 + [(a2 â&#x20AC;&#x201C; 1) y2 + 1 â&#x20AC;&#x201C; x2]2 + [16r2y4 â&#x2039;&#x2026; (a2â&#x20AC;&#x201C; 1) + 1 â&#x20AC;&#x201C; u2]2 + [((a + u2 â&#x2039;&#x2026; (u2 â&#x20AC;&#x201C; a))2 â&#x20AC;&#x201C; 1) â&#x2039;&#x2026; (n + 4dy)2 + 1 â&#x20AC;&#x201C; (x + cu)2]2 + [(a2 â&#x20AC;&#x201C; 1)ç 2 + 1 â&#x20AC;&#x201C; m2]2 +[ai + k + 1 â&#x20AC;&#x201C; ç â&#x20AC;&#x201C; i]2 + [n + ç + v â&#x20AC;&#x201C; y]2 + [p + ç (a â&#x20AC;&#x201C; n â&#x20AC;&#x201C; 1) + b (2an +2a â&#x20AC;&#x201C; n2 â&#x20AC;&#x201C; 2n â&#x20AC;&#x201C; 2) â&#x20AC;&#x201C; m]2 + [q + y (a â&#x20AC;&#x201C; p â&#x20AC;&#x201C; 1) + s â&#x2039;&#x2026; (2ap + 2a â&#x20AC;&#x201C; p2 â&#x20AC;&#x201C; 2p â&#x20AC;&#x201C; 2) â&#x20AC;&#x201C; x]2 + [z +pç (a â&#x20AC;&#x201C; p) + t (2ap â&#x20AC;&#x201C; p2 â&#x20AC;&#x201C; 1) â&#x20AC;&#x201C; pm]2)} è ĂŠ ĂŞrĂŤ ĂŹ ĂVĂŞ ĂŽ+ĂŻ ĂŞrĂ° Ăą ò ï óôù ĂŽ Ăľ Üôá ܼø ÜôÚ Ăş Ăť Ăź Ă˝ Ă˝ Ăž Ăż 3Ăż Ăź Ă˝ Ăź  Þ Ăź Ă˝ Ă&#x203A; Ăž Ă˝
Ăż Ă˝ Ăż Ă˝
EUREKA! N°4, 1999
21
Sociedade Brasileira de MatemĂĄtica
Â&#x2030; u o Tp tÂ&#x201A;q? tÂ&#x201A;u o rÂ&#x2019; o Â&#x2019;0/ )tc o } } Ă&#x2DC;qst Â&#x2019; , t o o Â&#x2122;V"Â&#x161; ! # Š Â&#x17E; Â? Â&#x; ÂĄ?Â&#x17E; Â?TÂ&#x17E; Š $ ! Â&#x161; Â&#x17E; sÂĄ ÂĄ ! ŠW4 % & Â&#x17E; ÂĄ 7 ( Â&#x; 'ÂĽÂ&#x17E; Â&#x161;*)3Â&#x17E; Â&#x203A; Â&#x161;Â&#x153;Â&#x;ÂŁÂ 3Â&#x; ÂĄ3Â&#x17E; " ' V !* 98
,+
s Žk¯ °.-
Considere a tabela 3 Ă&#x2014; 3 abaixo, onde todas as casas, inicialmente, contĂŠm zeros: 0
0
0
0
0
0
0
0
0
Para alterar os nĂşmeros da tabela, ĂŠ permitida a seguinte operação: escolher uma sub-tabela 2 Ă&#x2014; 2 formada por casas adjacentes, e somar 1 a todos os seus nĂşmeros. a)
b)
Diga se ĂŠ possĂvel, apĂłs uma seqßência de operaçþes permitidas, chegar Ă tabela abaixo: 7
9
2
15
25
12
8
18
10
Complete o quadro abaixo, sabendo que foi obtido por uma seqßência de operaçþes permitidas: 14
19
36
14
/10325436 780 93: ;=<8>3?@0BAC?ED8FHG 03IJ: ?5IJD K Antes de resolver o problema, ĂŠ preciso notar que existem quatro quadrados 2 Ă&#x2014; 2 no quadrado 3 Ă&#x2014; 3. Analisando-os, percebemos que os quadrados do canto sĂŁo afetados por apenas um deles. Com isso, deduzimos que cada nĂşmero nos quadradinhos do canto indica o nĂşmero de vezes que a operação permitida foi utilizada com o quadrado 2 Ă&#x2014; 2 que continha o quadradinho do canto. Os quadradinhos do lado sĂŁo afetados por dois quadrados diferentes, assim como no diagrama abaixo. EUREKA! N°4, 1999
22
Sociedade Brasileira de Matemática
1
3 1 ,3 B
1
3
1
3 1 ,2 A
3 ,4 C
2
4 2 ,4 D
2
4
4
2
Como a cada utilização da operação permitida, se for utilizada uma das subtabelas escritas em um dos quadrados laterais, o quadradinho lateral aumentará em um, concluímos que o valor do quadradinho lateral é igual à soma dos dois quadradinhos da ponta adjacentes. O quadradinho central, como é afetado por todos os quatro quadrados, é fácil deduzir que ele nada mais é que o número de operações aplicados na tabela. a) Como os números dos quadradinhos do canto representam quantas vezes a sub-tabela que contém aquele quadradinho foi utilizada para fazer uma operação e o quadradinho central o número de operações feitas, podemos concluir que de acordo com o diagrama, A + B + C + D = E*. Mas no nosso caso, temos 7 + 2 + 8 + 10 = 25, ou 27 = 25. Absurdo. Concluímos então que é impossível obter esta tabela. Outra prova é que a cada operação, aumentamos em um o valor de quatro casos, mantendo o resultado divisível por quatro. No nosso caso, a soma dos números é 106, que não é divisível por 4. A
F
B
G
E
I
C
H
D
b) No diagrama acima, com os raciocínios antes do problema, temos: G = A + C, F = A + B, I = B + D e H = C + D. Portanto, no problema, 19 = 14 + C, onde C = 5. Com isso, 14 = 5 + D, onde D = 9. Prosseguindo com base em *, 36 = 5 + 9 + 14 + B, onde B = 8. F = 8 + 14 ⇒ F = 22. Finalizando, 1 = 8 + 9 = 17. A tabela é 14
22
8
19
36
17
5
14
9
EUREKA! N°4, 1999
23
Sociedade Brasileira de MatemĂĄtica
" ' V !* 9@ Encontre uma maneira de se escrever os algarismos de 1 a 9 em seqßência, de forma que os nĂşmeros determinados por quaisquer dois algarismos consecutivos sejam divisĂveis ou por 7 ou por 13.
L1M3N5O3P Q8MBR3S TVU3R=WXS Y8Y Z [V\5Y8Y S8]"]"\ ^1Z8\5_a` MĂşltiplos de 7 : 7, 14, 21, 28, 35, 42, 49, 56, 63, 70, 77, 84, 91, 98. MĂşltiplos de 13 : 13, 26, 39, 52, 65, 78, 91. Como nenhum dos mĂşltiplos de 7 e 13, a nĂŁo ser 77, terminava em 7, na seqßência, 7 deveria ser o 1°. nĂşmero, entĂŁo, o nĂşmero formado foi 784913526. 5* 3 9 â&#x20AC;&#x201C; 1*
6 2 7â&#x20AC;&#x201C;8â&#x20AC;&#x201C;4
5* 5 â&#x20AC;&#x201C; 6* 1â&#x20AC;&#x201C;3
9â&#x20AC;&#x201C;1â&#x20AC;&#x201C;3â&#x20AC;&#x201C;5
9* 2â&#x20AC;&#x201C;6 6*
Nota do editor: * significa que nĂŁo dĂĄ para continuar sem repetir um nĂşmero jĂĄ usado.
" ' V !* 9A Em um jogo existem 20 buracos vazios em fila e o jogador deve colocar um pino em cada buraco de acordo com as seguintes regras: a) b) c)
Se colocar um pino em um buraco e se os dois buracos vizinhos estiverem vazios, o pino permanece. Se colocar um pino em um buraco e se um dos buracos vizinhos estiver ocupado, o pino deste buraco vizinho deve ser retirado. Se colocar um pino em um buraco e se os dois buracos vizinhos estiverem ocupados, entĂŁo um dos pinos vizinhos deve ser retirado.
Determine qual ĂŠ o nĂşmero mĂĄximo de pinos que podem ser colocados.
EUREKA! N°4, 1999
24
Sociedade Brasileira de MatemĂĄtica
L1M3N5O3P Q8MBR3S bVZ \5MacHZ8d3U3MBbVZ8Y8]"WJM3` O número måximo Ê 19. Veja a explicação abaixo: Começo colocando um pino no primeiro buraco da esquerda, pulo um buraco e coloco outro pino. Depois eu coloco um pino no buraco que estå entre os dois e retiro o da direita. Depois pulo uma casa do segundo pino e coloco um terceiro pino. Depois coloco um pino entre o terceiro e o segundo pino e retiro o terceiro pino, o da direita. E faço essa operação sucessivamente atÊ chegar ao último pino.
" ' V !* 9B Sete nĂşmeros naturais sĂŁo escritos em cĂrculo. Sabe-se que, em cada par de nĂşmeros vizinhos, um deles divide o outro. Mostre que hĂĄ dois nĂşmeros nĂŁo vizinhos com a mesma propriedade (isto ĂŠ: um deles divide o outro).
L1M3N5O3P Q8MBR3S c e8WJf8\EMhg8O=UhiV\5Y8Z _aM3]"M3` a
b
g c d
f e
Em cada dois nĂşmeros adjacentes pelo menos um ĂŠ mĂşltiplo do outro; desse modo ĂŠ impossĂvel fechar o cĂrculo sem que algum nĂşmero divida um outro nĂşmero que nĂŁo seja adjacente a ele, pois se a for mĂşltiplo de b e b for mĂşltiplo de c, entĂŁo c divide a e jĂĄ haverĂĄ dois nĂşmeros nĂŁo vizinhos com a propriedade. Se a for mĂşltiplo de b e c for mĂşltiplo de b e d, e for mĂşltiplo de d e q e o g nĂŁo existisse, poderia nĂŁo haver dois nĂşmeros nĂŁo vizinhos com a mesma propriedade mas como o g existe ele terĂĄ de ser mĂşltiplo de f e divisor de a. Desse modo f terĂĄ de ser divisor de a, sendo que com isso haverĂĄ a propriedade.
" ' V !* 98
kj
kj
? Ž ¯ °¼¹
Prove que em qualquer pentågono convexo existem dois ângulos internos consecutivos cuja soma Ê maior ou igual a 216°.
L1M3N5O3P Q8MBR3S l=m3\5Z d3MBnVZ8WJWJM3Y [VM3R=WX\Ed3O3S8YHbVM3Y8]"Z ` Considere A, B, C, D e E os vÊrtices do pentågono. EUREKA! N°4, 1999
25
Sociedade Brasileira de MatemĂĄtica
Suponha que nĂŁo existam dois ângulos consecutivos cuja soma seja maior ou igual a 216°. Assim: â&#x2C6;§
â&#x2C6;§
â&#x2C6;§
â&#x2C6;§
â&#x2C6;§
â&#x2C6;§
â&#x2C6;§
â&#x2C6;§
â&#x2C6;§
â&#x2C6;§
A+ B < 216° A
B
B + C < 216° C + D < 216°
E
C
D + E < 216°
D
E + A < 216° â&#x2C6;§
â&#x2C6;§
â&#x2C6;§
â&#x2C6;§
â&#x2C6;§
â&#x2C6;§
â&#x2C6;§
â&#x2C6;§
â&#x2C6;§
â&#x2C6;§
â&#x2C6;§
Somando membro a membro: A+ B + B + C + C + D + D + E + A < 5216° â&#x2C6;§
â&#x2C6;§
â&#x2C6;§
â&#x2C6;§
â&#x2C6;§
â&#x2C6;§
â&#x2C6;§
â&#x2C6;§
2 ( A + B + C + D + E ) < 1080° â&#x2021;&#x2019; A + B + C + D + E < 540° , mas a soma dos â&#x2C6;§
â&#x2C6;§
â&#x2C6;§
â&#x2C6;§
â&#x2C6;§
ângulos internos de um pentĂĄgono ĂŠ 540°*. Logo A+ B + C + D + E = 540° ĂŠ absurdo. EntĂŁo pelo menos 2 ângulos consecutivos tem soma maior ou igual a 216°. *Lema: A soma dos ângulos de um pentĂĄgono ĂŠ 540°. Prova: Seja P um ponto interior a ABCDE, logo P vai formar 5 triângulos no pentĂĄgono, a soma de todos os ângulos P dos triângulos ĂŠ 360°. E a soma dos ângulos que restam ĂŠ justamente a soma dos ângulos de ABCDE. A soma dos ângulos do pentĂĄgono ĂŠ 5180° â&#x20AC;&#x201C; 360° = 540°.
" ' V !* 9@
No triângulo ABC, D ĂŠ o ponto mĂŠdio de AB e E o ponto do lado BC tal que BE = 2 â&#x2039;&#x2026; EC. Dado que os ângulos
oqpsâ&#x2C6;§ r
e
t1o r L1M3N5O3P .Q8MBR3S vVZ U3\5S8N8^1\5U3w3S \5WJMhL1M3x3WJS \5WJZ8`
t1o â&#x2C6;§ u
são iguais, encontre o ângulo
â&#x2C6;§
B x y
P x
D y A
Îą
W
Îą
W 90°°â&#x20AC;&#x201C; Îą
â&#x20AC;˘E
O â&#x20AC;˘ 2Îą
x W 90°°â&#x20AC;&#x201C; Îą
EUREKA! N°4, 1999
26
C
Sociedade Brasileira de MatemĂĄtica
Chamarei de P o ponto mĂŠdio de BE . E chamarei BP = PE = EC = X. e BD = DA = y. A interseção de AE com DC , chamarei de O. O triângulo DOA ĂŠ isĂłsceles, portanto AO = DO . Chamarei AO de W. O segmento DP ĂŠ a base mĂŠdia do triângulo ABE, pois D ĂŠ o ponto mĂŠdio de AB e P ĂŠ o ponto mĂŠdio de BE , entĂŁo DP // AE. ConseqĂźentemente os triângulos OCE e DCP sĂŁo semelhantes, na razĂŁo de 1/2. EntĂŁo temos CO = 1 / 2 = D , 2 CO = CO + W = D , W = CO. CO + W Chamarei o ângulo OĂ&#x201A;D de Îą, â&#x2C6; ADO, tambĂŠm serĂĄ Îą, e o â&#x2C6; ADC, serĂĄ 2Îą, pois ĂŠ externo ao triângulo DOA. Como AO = W e OC = W o triângulo ADC ĂŠ isĂłsceles e o ângulo da base ĂŠ 90° â&#x20AC;&#x201C; Îą. O ângulo BAC = Îą + 90° â&#x20AC;&#x201C; Îą, BĂ&#x201A;C = 90°.
" ' V !* 9A Veja problema 3 do NĂvel 1.
" ' V !* 9B SĂŁo dados 15 nĂşmeros naturais maiores que 1 e menores que 1998 tais que dois quaisquer sĂŁo primos entre si. Mostre que pelo menos um desses 15 nĂşmeros ĂŠ primo.
L1M3N5O3P Q8MBR3S iVO3_Bx3S8WJ]"MBL1\5N5y ZHzVZ8y8S Y8` Teorema: Dado um nĂşmero n, composto, entĂŁo ele possui um fator (â&#x2030; 1) menor ou igual Ă raiz quadrada deste nĂşmero. Prova: Se n = a â&#x2039;&#x2026; b, podemos ter ou a < 1.-
a=
n
2.-
a<
n
3.-
a>
n â&#x2021;&#x2019; b=
n,a=
n ou a >
n:
n n < < n a n
Em qualquer caso, temos um fator menor ou igual a
EUREKA! N°4, 1999
27
n e diferente de 1.
Sociedade Brasileira de MatemĂĄtica
Resposta: Dado 1 < n < 1998, se ele nĂŁo for primo, ele tem que ter um fator primo menor que 1998 , ou seja, um fator primo, menor que 45. Como sĂł existem 14 primos menores que 45, e sĂŁo 15 nĂşmeros, entĂŁo um desses nĂŁo terĂĄ fator primo menor que 45, logo serĂĄ primo. (Pelo CorolĂĄrio do teorema anterior.)
|{
|{
|{
} ~ Â&#x20AC;3Â Â&#x192;Â&#x201A; Â&#x201E;
Primeira Prova.
Â&#x2026;.Â&#x2020;qÂ&#x2021;Â&#x2030;Â&#x2C6;qÂ&#x160;Â&#x152;Â&#x2039;.Â? Â&#x17D;Â?Â? Veja Problema 4 do NĂvel 2.
Â&#x2018; Â&#x2019; Â&#x201C;Â&#x2022;Â&#x201D; Â&#x2013; Â&#x2014; Â&#x2DC; Â&#x2122;sÂ&#x161; No triângulo ABC, D ĂŠ o ponto mĂŠdio de AB e E o ponto do lado BC tal que o p r Â? Â&#x153; BE = 2 . EC. Dado que os ângulos Ë&#x2020; e Ë&#x2020; Â&#x203A; sĂŁo iguais, encontre o ângulo
t oË&#x2020;r .
Veja a solução do Problema 2 do NĂvel 2.
L1M3N5O3P Q8MBZ8N5]"S WXU=Z8]"\5y ZHR3SHÂ&#x17E;3Z8x=WX\Ef \5MhL1\EÂ&#x;3O3S8\EWXZHÂ VS8U3S8y8\5R3S Y8`
A Îą y
θ
D Îą F
y
θ A
2x
E
x
C
Seja F = AE â&#x2C6;Š CD. Denotamos por [XYZ] a ĂĄrea do triângulo XYZ. Veja que [ABE] = 2 â&#x2039;&#x2026; [ACE] pois possuem a mesma altura relativa a BE e EC respectivamente e DE = 2 â&#x2039;&#x2026; EC. Analogamente [BFE] = 2 â&#x2039;&#x2026; [FEC]. Temos: [ABE] = 2 â&#x2039;&#x2026; [ACE] (I) [BFE] = 2 â&#x2039;&#x2026; [FEC] (II) EUREKA! N°4, 1999
28
Sociedade Brasileira de Matemática
I – II ⇒ [ABE] – [BFE] = 2 ⋅ ([ACE] – [FEC]) ⇒ [ABF] = 2 ⋅ [AFC]. Só que: [ABF] = 2 ⋅ [ADF] (mesma altura relativa a BA e DA respectivamente e BA = 2 ⋅ DA) Donde 2 ⋅ [ADF] = 2 ⋅ [AFC] ⇒ [ADF] = [AFC]. Como esses possuem a mesma altura relativa a DF e FC respectivamente, ∧
∧
temos: DF = FC ⇒ F A C = A C F = θ ∧
∧
Mas α = A D C = B A E ⇒ DF = AF . Daí, DF = AF = FC e CDA
é
∧
retângulo em A . Prova: ∆ADC : α + α + θ + θ + θ = 180° ⇒ α + θ = 90° ⇒ DÂC = 90° Logo BÂC = 90° é o que queríamos achar.
. q q¡ . ¢ Duas pessoas disputam um jogo da maneira descrita a seguir. Inicialmente escolhem dois números naturais: n ≥ 2 (o número de rodadas) e t ≥ 1 (o incremento máximo). Na primeira rodada o jogador A escolhe um natural m1 > 0 e, posteriormente, o jogador B escolhe um natural positivo n1 ≠ m1. Para 2 ≤ k≤ n, na rodada k o jogador A escolhe um natural mk com mk – 1 < mk ≤ mk – 1 + t e posteriormente o jogador B escolhe um natural nk com nk – 1 < nk ≤ nk – 1 + t. Após essas escolhas, nessa k-ésima rodada, o jogador A ganha mdc (mk , nk – 1) pontos e o jogador B ganha mdc (mk , nk) pontos. Ganha o jogo o jogador com maior pontuação total ao fim das n rodadas. Em caso de pontuações totais iguais, o jogador A é considerado vencedor. Para cada escolha de n e t, determine qual dos jogadores possui estratégia vencedora.
£1¤3¥5¦3§ ¨8¤B©3ª «V¦3¬ 3¤3®3ª8¯ °1¬5± ²3±H 3¬@¥5³3¤3´ Nota: A solução usa a notação abreviada (m, n) = mcd(m,n). Seja S um super-número. Esse super-número é divisível por todos os naturais. Para B ganhar sempre, basta fazer nk = S + mk. Pois, na rodada k, B ganha (nk, mk) = mk pontos e A ganha (S + mk–1, mk) pontos. Assim A ganha no máximo mk pontos, temos que mkS + mk–1 ⇒ mkmk–1, mas mk > mk–1. Assim A nunca ganhará mk pontos. Ganhará sempre menos que mk. Portanto EUREKA! N°4, 1999
29
Sociedade Brasileira de Matemática
B no total ficará com mais pontos que A, já que ganha mais em todas as rodadas. Obs. É claro que não existe um super-número, mais para cada n, t e m1, B pode criar um número, que não chega a ser super, mas também serve. Basta fazer: S = ( 2⋅3⋅5⋅7⋅11⋅…⋅pθ)M, com θ e M suficientemente grandes!, Assim B sempre ganha! Explicando melhor: Após A escolher m1, B sabe que o maior número que A pode escolher é m1 + (n – 1) t B só precisa fazer S=
π
m1 + ( n −1) t j = m1
j e nk = S + mk
Justificativa: m1 < mi ≤ m1 + (n – 1) t ⇒ miS Na rodada k: • B ganha (mk, nk) = mk, já que mkS + mk •
A ganha (mk, nk – 1) = (mk, S + mk – 1)
Como m k > m k −1 m k não divide m k −1 ⇒ ⇒ m k não divide ( mk −1 + S ) ⇒ m k > (m k , n k −1 ) m S m S k k Portanto, a cada rodada B ganha mais pontos que A, e portanto B ganha o jogo. Obs. mk – 1 < mk ≤ mk – 1 + t ⇒ S + mk – 1 < S + mk ≤ S + mk – 1 + t ⇒ ⇒ nk –1 < nk ≤ nk – 1 + t Resposta: B sempre ganha (se for esperto)!!!
¶µ
¶µ
¶µ
} ~ 3
Segunda Prova.
. q q¡ . · Dois meninos jogam o seguinte jogo. O primeiro escolhe dois números inteiros diferentes de zero e o segundo monta uma equação do segundo grau usando como coeficientes os dois números escolhidos pelo primeiro jogador EUREKA! N°4, 1999
30
Sociedade Brasileira de MatemĂĄtica
e 1998, na ordem que quiser (ou seja, se o primeiro jogador escolhe a e b o segundo jogador pode montar a equação 1998x2 + ax + b = 0, ou bx2 + 1998x + a = 0, etc.) O primeiro jogador ĂŠ considerado vencedor se a equação tiver duas raĂzes racionais diferentes. Mostre que o primeiro jogador pode ganhar sempre.
ÂŁ1¤3ÂĽ5ÂŚ3§ ¨8¤BŠ3ÂŞ Â&#x17E;=Âą8¸3šJÂŹ5Âş ÂŹE¤hÂŁ1ÂŹ5Â&#x;3ÂŚ=ÂŞ8ÂŹ5šJÂąH CÂŞ ²3ÂŞ Âť ÂŹ5Š3ÂŞ8ÂŻ ´ Inicialmente veja que, se num polinĂ´mio p(x) = anxn + â&#x20AC;Ś + a1x + a0, tivermos an + â&#x20AC;Ś a1 + a0 = 0, teremos p (1) = 0 e 1 ĂŠ raiz de p. Dessa forma se o primeiro jogador escolhe b = â&#x20AC;&#x201C; (1998 + a), 1 serĂĄ raiz da equação do segundo grau que o seu oponente irĂĄ montar. Se um polinĂ´mio tem coeficientes inteiros (na verdade vale para coeficientes racionais) e possui uma raiz irracional do tipo a + b r (r nĂŁo ĂŠ quadrado perfeito), entĂŁo a â&#x20AC;&#x201C; b r tambĂŠm ĂŠ raiz. Ou seja, as raizes irracionais vĂŞm aos pares. No caso de uma equação de segundo grau, e coeficientes inteiros, ambas as raizes sĂŁo irracionais, ou ambas sĂŁo racionais. No nosso caso, como 1 jĂĄ ĂŠ raiz, a outra raiz serĂĄ racional. Basta ver entĂŁo, apenas se 1 nĂŁo ĂŠ raiz multipla (pois queremos que as raizes sejam distintas). Para isso basta escolher a adequadamente. Se o primeiro jogador escolher os nĂşmeros a = nâ&#x2039;&#x2026;1998, e b = â&#x20AC;&#x201C; (n + 1) 1998, n â&#x2030;Ľ 2 â&#x2C6;&#x2C6; Âź , ele ganha. (1 nĂŁo serĂĄ raiz mĂşltipla e a equação terĂĄ duas raizes racionais distintas.) Obs. Ă&#x2030; possivel obter soluçþes com a + b + 1998 â&#x2030; 0, por exemplo com {a, b} = {2040, â&#x20AC;&#x201C; 5478} (solução obtida com o auxĂlio de um computador.)
Â&#x2026;.Â&#x2020;qÂ&#x2021;Â&#x2030;Â&#x2C6;qÂĄÂ&#x152;Â&#x2039;.Â? Â&#x17D;Â?½ Determine todas as funçþes f : Âź â&#x2020;&#x2019; Âź que satisfazem f (2 f ( x)) = x + 1998 para todo x â&#x2C6;&#x2C6; ž = {0,1,2,...}.
ÂŁ1¤3ÂĽ5ÂŚ3§ ¨8¤BŠ3ÂŞ Âż1Ă&#x20AC;BÂą8²3ÂŚ.ÂŞ ÂĽ8Ă VÂŚ3Ă&#x201A;3ÂŚ=ÂŻ8Ă&#x192;"¤hŠ=ÂŞ ÂŁ1¤3ÂŚ.Ă&#x201E; ÂąHĂ&#x2026;CÂą šJ²3ÂŞ8ÂŹ5šJ¤3´ 1° Passo. f ĂŠ injetiva. Prova: Suponha que exista k â&#x2C6;&#x2C6; Ă&#x2020; tal que f (x) = f (y) = k para x, y â&#x2C6;&#x2C6; Ă&#x2020; e x â&#x2030; y. DaĂ f(x) = f(y) â&#x2021;&#x2019; 2f(x) = 2 f(y) â&#x2021;&#x2019; f(2f(x)) = f (2f(y)) â&#x2021;&#x2019; x + 1998 = y + 1998 â&#x2021;&#x2019; x = y. Absurdo! Logo f ĂŠ injetiva. EUREKA! N°4, 1999
31
Sociedade Brasileira de Matemática
2° Passo. f e sobrejetiva no contradomínio{k∈ Ç k ≥ 1998}, isto é, para todo k ∈ Ç , k ≥ 1998 ∃ α ∈ Ç tal que f(α) = k. Prova: Se k ≥ 1998 faça k = x + 1998 para x ≥ 0 e x ∈ Ç , logo f(2f(x)) = x + 1998 = k Tome então 2f(x) = α ∈ Ç . Logo f é sobrejetiva no contradomínio acima. 3° Passo. Então para todo k ∈ A = {k∈ Ç k ≥ 1998} existe um único natural x (que chamaremos f ' (k) = x) tal que f(x) = k. Obviamente existe um natural i ímpar tal que f (i) = w ≥ 1998 pois não podemos ter 0 ≤ f(i) ≤ 1997 para todos os naturais ímpares porque a função f é injetiva e de Ç em Ç (princípio de casa de pombos). Para esse w teremos f '(w) = i, pois f (i) = w. Por outro lado faça w = a + 1998 com a ∈ Ç Pela definição: f (2f(a)) = a + 1998 = w. Logo f '(w) = 2f (a). Como f '(w) é único, pela conclusão 1 ⇒ i = 2f(a) como f(a) ∈ Ç ⇒ i é par, absurdo! Conclusão 2. Não existe função desse tipo.
. q q¡ . È Dois matemáticos, perdidos em Berlim, chegam à esquina da rua Barbarossa com a rua Martin Luther, e precisam chegar à esquina da rua Meininger com a rua Martin Luther. Infelizmente eles não sabem para que lado fica a rua Meininger, nem a que distância ela está, logo são obrigados a ir e voltar ao longo da rua Martin Luther até chegaram à esquina desejada. Qual é o menor valor para o número positivo K tal que eles podem ter certeza de que se há N quarteirões (ou quadras) entre as ruas Barbarossa e Meininger então eles conseguem chegar ao destino andando no máximo KN quarteirões (ou quadras)?
£1¤3¥5¦3§ ¨8¤BÉ"©3¤3¯HÀh±8Ã*ª ÀhÊ8Ã"¬Eº8¤3¯ ®3ª8¹J©3¬5©=¤3¯ ª8ÀË Vª8¹J¥5¬EÀhÌ Este problema é baseado numa situação real, ocorrida com os matemáticos Nicolau Saldanha e Carlos Gustavo Moreira, que se encontravam em Berlim por ocasião do Congresso Internacional de Matemática de 1998. Era de noite, não havia ninguém na Martin-Luther-Strasse a quem pedir informações e eles queriam chegar rápido ao destino. A idéia do problema era que eles andassem juntos. Apareceram soluções em que os matemáticos se separavam (o que não havia sido previsto), as quais foram avaliadas caso a caso. Como os matemáticos não sabiam para que lado nem a que distância estava a Meiningerstrasse, deviam adotar uma estratégia do seguinte tipo: EUREKA! N°4, 1999
32
Sociedade Brasileira de Matemática
andar a1 quarteirões para um lado (digamos o direito), depois voltar ao ponto inicial e andar a2 quarteirões para a esquerda, depois a3 para a direita, depois a4 para a esquerda e assim sucessivamente, onde a1, a2, a7…são números inteiros positivos com a1 < a3 < a5 < … e a2 < a4 < a6 <… até encontrar a Meiningerstrasse. Os piores casos são quando a Meininger está a a2k + 1 + 1 quarteirões à direita ou a2k + 1 à esquerda da Barbarossastrasse, com k natural (convencionamos a0 = 0). Nesses casos, temos que entre o ponto inicial e o destino há an + 1 quarteirões, e os matemáticos andam no total 2a1 + 2a2 + …+ 2an + 2an + 1 + an + 1 quarteirões até chegarem ao destino (com n = 2k+1 ou n = 2k). Assim, devemos ter 2a1 + 2a2 + … + 2an + 2an + 1 + an + 1 ≤ k (an + 1), ou seja, k −1 S n +1 ≤ (a n + 1 ) para todo n ∈ Í , onde Sn = a1 + a2 + … + an.
2
Para k = 9 existem estratégias que satisfazem as condições do problema, por exemplo tomando am = 2m para todo m ∈ Í . De fato, teremos Sn + 1 = 21 + 22 k −1 +… + 2n + 1 = 2n + 2 – 2 < 4 (2 n + 1) = (a n + 1) para todo n ∈ Í . 2 k −1 Mostraremos que 9 é o menor k possível. Seja k < 9. Então c = é 2 menor que 4. Se k satisfaz as condições do problema, deve haver uma sequência (an) como acima com Sn + 1 ≤ c (an + 1) para todo n ∈ Í . Como an = Sn – Sn –1 teremos Sn + 1 ≤ c (Sn – Sn – 1 + 1) para todo n ∈ Í . Definimos Un = Sn – c, temos Un + 1 ≤ c (Un – Un – 1) para todo n ∈ Í . Como c < 4, Un > 0 para todo n ≥ 3, e, definindo Vn = Un + 1 /Un para todo n ≥ 3, teremos Vn ≤ c (1 – 1/Vn – 1) para todo n ≥ 4, onde Vn > 0 para todo n ≥ 3. Entretanto, Vn ≤ c (1 – 1/Vn – 1) implica Vn – Vn – 1 ≤ c ( 1 – 1/Vn –1) – Vn – 1 = 2
cV n −1 − c − V n V n −1 todo n ≥
2
c2 c − c − − V n −1 −1 4 2 ≤ c (c − 4) < 0 ⇒ V < V = n n −1 4V n −1 V n −1 4. Por outro lado, para todo n ≥ 4
para temos
c ( c − 4) c(c − 4) para todo n ≥ 4, donde ⇒ Vn − Vn −1 ≤ Vn − Vn −1 ≤ 4V n −1 4V3 (n − 3) c(c − 4) para todo n ≥ 4, absurdo, pois o lado direito é negativo V n ≤ V3 + 4V3 2
4V3 para n > 3 + c(4 − c) ❑ EUREKA! N°4, 1999
33
Sociedade Brasileira de Matemática
Î ÎÐÏÒÑ ÓÕÔ×Ö Ø@ÙÛÚ ÙÝÜßÞ ÙÛàHÓáÑ â ÓáÞ ÙÝÚßâãÔäÙBåaâ Ôäæ åçÓéè Ù Þ êìë í î@ïñð ò óõôçÖ ö ÷áø êì÷áö óúùßû@üCê îCý"þ ÿ =ê ÿ à ìö ÷5ê ë Nome Márcio Jun Hisamoto Fábio Dias Moreira Andressa Rissetti Paim Henry Wei Cheng Hsu Natália Argene Lovate Pereira Daniel Cherobini Luis Eduardo de Godoi Milton Eiji Kato Fabrício Henrique de Faria Davi Máximo Alexandrino Nogueira Caio Magno Castro de Paula Bruno Moreira de Souza Días Patrícia Akemi Komura Marcelo Li Koga Alberto Hikaru Shintani Diego Gomes Gripp Renato Mendes Coutinho Antônio Monteiro Guimarães Jr. Leonardo Luis Desideri Freitas Helder Seiji Kato Aline Galvão Rodrigo Miyashiro Nunes dos Santos João Marcos da Cunha Silva Thiago Mizuta Oliveiro Ribeiro Barbosa Jr. Jorge Peixoto Andréia Lúcio de Castro Léo Jaime Zandonai Pedro Junqueira de Barros Lucas Ikeda França Paulo Roberto Sampaio Santiago Rafael Marini Silva Eduardo Souza Cruz Breno Ignácio da Silva Diogo dos Santos Suyama Flavio Schiavini Abe
Cidade - Estado São Paulo - SP R. de Janeiro - RJ Santa Maria - RS São Paulo - SP Jundiaí - SP Santa Maria - RS SJ dos Campos - SP São Paulo - SP São Paulo - SP Fortaleza - CE Fortaleza - CE SJ dos Campos - SP São Paulo - SP São Paulo - SP São Paulo - SP Vitória - ES Americana - SP Campina G - PB Vitória - ES São Paulo - SP São Paulo - SP São Paulo - SP Fortaleza - CE São Paulo - SP Teresina - PI Goiânia - GO Goiânia - GO BentoGonçalves - RS São Paulo - SP São Paulo - SP Salvador - BA Vila Velha - ES Vitória - ES Sertãozinho - SP Belo H. - MG Vitória - ES
EUREKA! N°4, 1999
34
Prêmio Ouro Ouro Prata Prata Prata Prata Prata Prata Prata Prata Prata Prata Bronze Bronze Bronze Bronze Bronze Bronze Bronze Bronze Bronze Bronze Bronze Bronze Bronze Menção Honrosa Menção Honrosa Menção Honrosa Menção Honrosa Menção Honrosa Menção Honrosa Menção Honrosa Menção Honrosa Menção Honrosa Menção Honrosa Menção Honrosa
Sociedade Brasileira de MatemĂĄtica
Ă&#x17E; ĂŞĂŹĂŤÂ&#x2030;Ă ĂŽ@ïùð ò óþôçà ê à ò óúÚĂ&#x;Ăť@ĂźCĂŞ ĂŽVĂ˝ Ăż =ĂŞ Ăż CĂ ĂŹĂś á5ĂŞ ĂŤ Nome Thiago Barros Rodrigues Costa Humberto Silva Naves Daniel Pinheiro Sobreira Afonso de Paula Pinheiro Rocha Hugo Pinto Iwata Thiago da Silva Sobral JoĂŁo Alfredo Castellani Fajardo Freire Artur Duarte Nehmi Einstein do Nascimento Jr. Daniel PessĂ´a Martins Cunha Gustavo Alonso Daud Patavino Rafael de Holanda Barroso Eduardo Kunio Kuroda Abe Renata Lourenção Delamanha Ricardo de Castro PalĂĄcio Helen Wei Ling Hsu Frederico Pinto Eduardo Famini Silva Victor Marchesini Ferreira Thalita Basso Eduardo Suaiden Klein Mauricio Massao Soares Matsumoto Claudia Giacomin Bof Guilherme Silveira Barrozo Netto Thiago AraĂşjo Fiorio Pedro Fernando Almeida Di Donato Gustavo Modenesi Jaquellyne Gurgel Penaforte Jordan GuimarĂŁes Lombardi Tiago Monteiro Fernandes Marcelo Loula Novais de Paula
Cidade - Estado Fortaleza - CE Goiânia - GO Fortaleza - CE Fortaleza - CE SJ de Rio Preto - SP Fortaleza - CE Salvador - BA São Paulo - SP Fortaleza - CE Fortaleza - CE Santos - SP Fortaleza - CE São Paulo - SP Jundiaà - SP Fortaleza - CE São Paulo - SP São Paulo - SP Salvador - BA Salvador - BA Jundiaà - SP R. de Janeiro - RJ São Paulo - SP Aracruz - ES R. de Janeiro - RJ Fortaleza - CE SJ dos Campos - SP São Paulo - SP Fortaleza - CE SJ dos Campos - SP Piracicaba - SP Salvador - BA
EUREKA! N°4, 1999
35
Prêmio Ouro Ouro Ouro Prata Prata Prata Prata Prata Prata Prata Prata Bronze Bronze Bronze Bronze Bronze Bronze Bronze Bronze Bronze Menção Honrosa Menção Honrosa Menção Honrosa Menção Honrosa Menção Honrosa Menção Honrosa Menção Honrosa Menção Honrosa Menção Honrosa Menção Honrosa Menção Honrosa
Sociedade Brasileira de MatemĂĄtica
Ă&#x17E; ĂŞ ĂŤCĂ ĂŽ@ĂŻ ðÏò óþô ĂĽ ĂŞĂŹĂś =ĂŞ áÊÜ Ăł ĂšĂ&#x;Ăť5ĂźÂ&#x2030;ĂŞ ĂŽCýùâ ĂŤCá óúĂ&#x201D; ò áåó Nome Emanuel Augusto de Souza Carneiro Mauricio Pereira Carrari Rui Lopes Viana Filho Fernando Paz Cardoso Fabricio Siqueira Benevides Tony Calleri França JĂ´nathas DiĂłgenes Castello Branco Glauf Sidney Duarte Moreira Jr. Sergio Alvarez AraĂşjo Correia SĂŠrgio Tadao Martins Lucas Heitzmann Gabrielli Christian Iveson Bruno Gurgel Fernandes TĂĄvora Mila Lopes Viana Daniel Massaki Yamamoto Daniele VĂŠras de Andrade Mauricio Masayuki Honda Leonardo Cardoso Souza Daniel Nobuo Uno Daniel MourĂŁo Martins JoĂŁo Paulo de Tarso Ferreira Evandro Makiyama de Melo Christian Lyoiti Watanabe Fred Olavo A. Carneiro Bruno Da Cunha Raymundo Pedro Paulo de Simoni Gouveia Carlos Alexandre Rolim Fernandes MĂĄrcio Afonso Assad Cohen Matheus de Lima Faheina Rodrigo M. Gorgoll VĂtor Menezes Santana Alexandre Ferreira Terezan Ricardo Sallai Viciana Remo H. de M. Furtado Wayne Leonardo Silva de Paula Thiago Steiner Alfeu MĂĄrcio Reis Lopes Seong Ho Lee
Cidade - Estado Fortaleza - CE São Paulo - SP São Paulo - SP São Paulo - SP Fortaleza - CE Fortaleza - CE Fortaleza - CE Fortaleza - CE Fortaleza - CE São Paulo - SP São Paulo - SP São Paulo - SP Fortaleza - CE São Paulo - SP São Paulo - SP Fortaleza - CE São Paulo - SP Angra dos Reis - RJ São Paulo - SP Fortaleza - CE Angra dos Reis - RJ São Paulo - SP Angra dos Reis - RJ Fortaleza - CE Rio de Janeiro - RJ Fortaleza - CE Fortaleza - CE Rio de Janeiro - RJ Fortaleza - CE São Paulo - SP Goiânia - GO Rio de Janeiro - RJ São Paulo - SP Fortaleza - CE BelÊm - PA Fortaleza - CE Salvador - BA Santo AndrÊ-SP
EUREKA! N°4, 1999
36
Prêmio Ouro Prêmio especial Ouro Ouro Ouro Ouro Prata Prata Prata Prata Prata Bronze Bronze Bronze Bronze Bronze Bronze Bronze Bronze Bronze Bronze Bronze Bronze Bronze Menção Honrosa Menção Honrosa Menção Honrosa Menção Honrosa Menção Honrosa Menção Honrosa Menção Honrosa Menção Honrosa Menção Honrosa Menção Honrosa Menção Honrosa Menção Honrosa Menção Honrosa Menção Honrosa Menção Honrosa
Sociedade Brasileira de MatemĂĄtica
Ă&#x2013; Ă&#x17E;Â?Ă? Ă&#x153;Ă&#x;Ă&#x2018; âHĂ&#x201D;äĂ&#x2122; Ă ĂşĂ&#x2122; Ăš ĂĽaĂ&#x201C; Ă? Ă
â&#x2122;Ś
Eduardo Wagner
! "$#% & ' ( "$' )+*
VocĂŞ sabe quando foi realizada a primeira OlimpĂada de MatemĂĄtica? Foi no ano de 1894, na Hungria. Neste ano, a Sociedade de MatemĂĄtica e FĂsica da Hungria promoveu uma competição de MatemĂĄtica, envolvendo todos os alunos dos Ăşltimos anos das escolas, para homenagear seu presidente LorĂĄnd EĂśtvĂśs, eleito ministro da educação do paĂs. O evento foi um sucesso, e passou a ser realizado todos os anos. Vamos mostrar neste artigo alguns problemas dessas competiçþes com suas soluçþes resumidas. Os problemas escolhidos nĂŁo sĂŁo muito difĂceis, mas sĂŁo bastante interessantes. Recomendo aos leitores pensar um pouco em cada um deles antes de ver a solução. As ferramentas exigidas sĂŁo elementares (apenas no problema 2 a noção de congruĂŞncia ĂŠ adequada) mas as soluçþes necessitam de uma certa dose de criatividade. Aproveitem!
Â&#x2026;.Â&#x2020;qÂ&#x2021;Â&#x2030;Â&#x2C6;qÂĄÂ&#x152;Â&#x2039;.Â? Â&#x17D;Â?Â?
â&#x20AC;&#x201C; OlimpĂada de 1894 Prove que as expressĂľes 2x + 3y e 9x + 5y sĂŁo divisĂveis por 17 para os mesmos pares de valores dos inteiros x e y.
Â&#x2026;.Â&#x2020;qÂ&#x2021;Â&#x2030;Â&#x2C6;qÂĄÂ&#x152;Â&#x2039;.Â? Â&#x17D;-,
â&#x20AC;&#x201C; OlimpĂada de 1898 Determine todos os valores do natural n, para os quais 2 n + 1 ĂŠ mĂşltiplo de 3.
Â&#x2026;.Â&#x2020;qÂ&#x2021;Â&#x2030;Â&#x2C6;qÂĄÂ&#x152;Â&#x2039;.Â? Â&#x17D;Â?¢
â&#x20AC;&#x201C; OlimpĂada de 1905 Na figura a seguir, AM, BN e CP sĂŁo paralelos. N M P
A
B
C
Prove que
EUREKA! N°4, 1999
37
Sociedade Brasileira de MatemĂĄtica
1 1 1 + = AM BN CP
Â&#x2026;.Â&#x2020;qÂ&#x2021;Â&#x2030;Â&#x2C6;qÂĄÂ&#x152;Â&#x2039;.Â? Â&#x17D;-.
â&#x20AC;&#x201C; OlimpĂada de 1906 A seqßência a1, a2, a3, â&#x20AC;Ś, an representa uma arrumação arbitrĂĄria dos nĂşmeros 1, 2, 3, â&#x20AC;Ś, n. Prove que se n ĂŠ um nĂşmero Ămpar o produto (a1 â&#x20AC;&#x201C; 1)(a2 â&#x20AC;&#x201C; 2)(a3 â&#x20AC;&#x201C; 3) â&#x20AC;Ś (an â&#x20AC;&#x201C; n) ĂŠ um nĂşmero par.
Â&#x2026;.Â&#x2020;qÂ&#x2021;Â&#x2030;Â&#x2C6;qÂĄÂ&#x152;Â&#x2039;.Â? Â&#x17D;-/
â&#x20AC;&#x201C; OlimpĂada de 1910 Se a, b, c sĂŁo nĂşmeros reais tais que a 2 + b 2 + c 2 = 1 , prove que â&#x2C6;&#x2019;
Â&#x2026;.Â&#x2020;qÂ&#x2021;Â&#x2030;Â&#x2C6;qÂĄÂ&#x152;Â&#x2039;.Â? Â&#x17D;-0
1 â&#x2030;¤ ab + bc + ca â&#x2030;¤ 1 2
â&#x20AC;&#x201C; OlimpĂada de 1913 2 Prove que para todo natural n > 2, tem-se (1 â&#x2039;&#x2026; 2 â&#x2039;&#x2026; 3 â&#x2039;&#x2026; â&#x2039;&#x2026; â&#x2039;&#x2026; n ) > n n .
Â&#x2026;.Â&#x2020;qÂ&#x2021;Â&#x2030;Â&#x2C6;qÂĄÂ&#x152;Â&#x2039;.Â? Â&#x17D;-1
â&#x20AC;&#x201C; OlimpĂada de 1916 No triângulo ABC, AD ĂŠ a bissetriz do ângulo A. Prove que AD < AB â&#x2039;&#x2026; AC .
Â&#x2026;.Â&#x2020;qÂ&#x2021;Â&#x2030;Â&#x2C6;qÂĄÂ&#x152;Â&#x2039;.Â? Â&#x17D;-2
â&#x20AC;&#x201C; OlimpĂada de 1916 Divida os nĂşmeros 1, 2, 3, 4, 5 em dois conjuntos quaisquer. Prove que um dos conjuntos contĂŠm dois nĂşmeros e sua diferença.
Ă Ă? Ă&#x2018; 354%6 â Ă Â&#x2026;.Â&#x2020;qÂ&#x2021;Â&#x2030;Â&#x2C6;qÂĄÂ&#x152;Â&#x2039;.Â? Â&#x17D;Â?Â? Observe que 4(2x + 3y) + (9x + 5y) = 17(x + y). Portanto, se 2x + 3y for mĂşltiplo de 17, entĂŁo 9x + 5y tambĂŠm serĂĄ, e vice versa. OBS: Esta aparente â&#x20AC;&#x153;mĂĄgicaâ&#x20AC;? nĂŁo ĂŠ a Ăşnica forma de resolver este problema. Os leitores que conseguirem outra solução (para este ou para qualquer problema deste artigo) podem enviar suas descobertas para publicação nos prĂłximos nĂşmeros da
798;:<79=?> @
EUREKA! N°4, 1999
38
. q q¡ . -,
Sociedade Brasileira de Matemática
A solução mais natural para este problema utiliza congruências. Observe que 2 ≡ (–1) mód.3. Logo, 2n ≡ (–1)n mód.3 e, portanto, 2n + 1 ≡ (–1)n + 1 mód.3. Concluimos então que , 2n + 1 ≡ 0 mód.3 se, e somente se, n é ímpar. OBS: O leitor familiarizado com indução pode conseguir outra solução.
. q q¡ . ¢ N M P
A
B
C
Utilizando semelhança de triângulos na figura acima temos: CP CB = AM AB CP AC = BN AB Somando temos: CP CP AC + CB + = =1 AM BN AB Daí,
. q q¡ . -.
1 1 1 + = AM BN CP
O produto (a1 – 1)(a2 – 2)(a3 – 3) … (an – n) possui um número ímpar de termos porque n é ímpar. Mas, a soma desses termos é zero, que é par. Como a soma de uma quantidade ímpar de números ímpares não pode ser par, concluímos que um dos termos é par e, conseqüentemente, o produto é um número par.
EUREKA! N°4, 1999
39
Sociedade Brasileira de Matemática
. q q¡ . -/ Primeira parte:
(a + b + c )2 ≥ 0 a 2 + b 2 + c 2 + 2ab + 2bc + 2ac ≥ 0
1 + 2(ab + bc + ca ) ≥ 0 ab + bc + ca ≥ −
Segunda parte:
1 2
(a − b )2 + (b − c )2 + (c − a )2 ≥ 0 2(a 2 + b 2 + c 2 ) − 2(ab + bc + ca ) ≥ 0 1 − (ab + bc + ca ) ≥ 0 ab + bc + ca ≤ 1
. q BA . -0
A expressão do lado esquerdo da desigualdade pode ser escrita assim: 1.n.2.(n – 1).3.(n – 2). … .(n – 2).3.(n – 1).2.n.1 Considere agora separadamente os produtos: 1.n, 2.(n – 1), 3.(n – 2), … , (n – 2).3, (n – 1).2, n.1 O primeiro e o último são iguais a n, mas afirmamos que qualquer um dos outros é maior que n. De fato, os produtos “do meio” são da forma (k + 1)(n – k) onde k assume os valores: 0, 1, 2, …, n – 1. Como para eles, n – k é maior que 1, temos que (k + 1)(n – k) = k(n – k) + (n – k) > k.1 + (n – k) = n Logo, como n é maior que 2, o produto do lado esquerdo é maior que n.n.n. … .n = nn.
. q BA . -1 Considere a circunferência circunscrita ao triângulo ABC.
EUREKA! N°4, 1999
40
Sociedade Brasileira de Matemática
A
D C
B E
A bissetriz AD encontra a circunferência em E, ponto médio do arco BC. Como os ângulos ABC e AEC são iguais (cada um deles vale a metade do arco AC) e como os ângulos BAE e EAC são também iguais (porque AD é uma bissetriz), concluimos que os triângulos ABD e AEC são semelhantes. Daí, AB AD = AE AC ou seja, AD ⋅ AE = AB ⋅ AC Como AD é menor que AE temos que AD ⋅ AD < AB ⋅ AC ou seja, AD < AB ⋅ AC
. q BA . -2 Vamos tentar dividir 1, 2, 3, 4, 5, em dois conjuntos tais que nenhum deles contém a diferença de dois de seus elementos. O 2 não pode estar no mesmo conjunto que o 1 ou o 4 porque 2 – 1 = 1 e 4 – 2 = 2. Portanto, vamos colocar o 2 em um conjunto e o 1 e o 4 no outro. O 3 não pode ficar no segundo conjunto porque 4 – 3 = 1. Logo, o 3 deve ficar no primeiro conjunto, junto com o 2. Agora, o 5 não pode ficar no primeiro conjunto porque 5 – 3 = 2, e nem pode ficar no segundo porque 5 – 4 = 1. A divisão proposta é portanto impossível.
EUREKA! N°4, 1999
41
Sociedade Brasileira de MatemĂĄtica
Ă? Ă&#x2018; Ă?C Ă?hĂĽaĂ&#x201C;ĂŠĂ&#x2013; Ă?Ă?Ă&#x161; Ă&#x2122; Ă?Ă&#x2019;Ă&#x2018; Ă&#x201C;ĂĄĂ&#x201D;Ă&#x2014;Ă&#x2013;HĂ&#x2DC;@Ă&#x2122;Ă&#x203A;Ă&#x161; Ă&#x2122;Ă?Ă&#x153;Ă&#x;Ă&#x17E; Ă&#x2122;Ă&#x203A;Ă Ă&#x201C;ĂŠĂ&#x2018; âHĂ&#x201C;ĂĄĂ&#x17E; Ă&#x2122;Ă?Ă&#x161;Ă&#x;âãĂ&#x201D;äĂ&#x2122;Ă&#x;ĂĽaâ Ă&#x201D;äÌĂ&#x;üçĂ&#x201C;åè Ă&#x2122; â&#x2122;Ś
! "$#% & ' ( "$' )+*
Paulo Cezar Pinto Carvalho IMPA
VocĂŞ jĂĄ prestou atenção ao logotipo da OlimpĂada Brasileira de MatemĂĄtica, presente na capa da e (em sua versĂŁo animada) na pĂĄgina da Internet da OBM? Os cĂrculos coloridos sĂŁo uma referĂŞncia ao sĂmbolo dos Jogos OlĂmpicos, que ĂŠ formado por 5 anĂŠis entrelaçados representando os continentes. No logotipo da OBM, porĂŠm, estes anĂŠis estĂŁo dispostos de um modo tal que conhecimentos matemĂĄticos sĂŁo essenciais para sua construção. O que existe de difĂcil em dispor cinco anĂŠis de modo que cada um seja tangente a dois outros e, alĂŠm disso, tangente a dois cĂrculos adicionais, um interior e outro exterior? Vejamos.
798;:<79=?> @
Tomemos dois cĂrculos arbitrĂĄrios, um contido no outro e posicionemos um novo cĂrculo, de modo a ser tangente a ambos. A partir daĂ, os demais cĂrculos estĂŁo definidos e a Fig. 1 mostra o que ocorre no caso geral: quando tentamos colocar o Ăşltimo cĂrculo, vemos que a figura nĂŁo fecha, ou seja, nĂŁo ĂŠ possĂvel colocar um quinto cĂrculo tangente a dois dos quatro cĂrculos jĂĄ colocados e aos dois iniciais.
D E FHGJILK M NJOPE QJR$SUTWV XYTZOP[ SUQP\JS^]Z_ _JQ`TZaPE bZa SerĂĄ que ĂŠ possĂvel colocar o primeiro cĂrculo colorido em outra posição, de modo a fazer com que a figura se feche exatamente? Pode-se ter uma idĂŠia da resposta a esta pergunta observando a versĂŁo animada do logotipo. Observe que os cĂrculos interno e externo sĂŁo fixos, mas os coloridos assumem tamanhos e posiçþes variĂĄveis e parecem girar em torno deles (veja a Fig. 2 a seguir). Ou seja, a animação sugere que o fechamento da figura nĂŁo depende da posição ou tamanho do primeiro cĂrculo colorido, dependendo somente do tamanho e posição relativas dos cĂrculos interno e externo! EUREKA! N°4, 1999
42
Sociedade Brasileira de MatemĂĄtica
DcE FHGedUK M ] TZE QJT SUTWV X!TZOP[ S`] ]f_ _PQJTfa`E bfaPgihJaPXja NPOPaJ[ N`OJ_PX hJS`]ZE k \JS lJS^hJXjE gU_`E XjS A explicação para estes fatos estĂĄ em uma transformação geomĂŠtrica dos pontos do plano chamada de inversĂŁo e definida do seguinte modo. Definição: Seja O um ponto do plano e k um nĂşmero real positivo. A inversĂŁo de centro O e constante k associa a cada ponto P do plano, distinto de O, o ponto Pâ&#x20AC;&#x2122; (chamado de inverso de P) sobre a semi-reta OP tal que OP. OPâ&#x20AC;&#x2122; = k. A Fig. 3 a seguir ilustra o resultado de se aplicar uma transformação de inversĂŁo a um conjunto de pontos do plano. Como o produto OP. OPâ&#x20AC;&#x2122; deve ser constante, quanto mais prĂłximo um ponto estiver de O, mais distante o seu inverso estarĂĄ. Q' Q P
P'
O
DcE FBGemUK n Q`oZ_PXj]f\PS
R R'
O logotipo da OBM ĂŠ construĂdo com o auxĂlio deste tipo de transformação, explorando dois fatos fundamentais. a) InversĂľes sĂŁo tranformaçþes injetivas (isto ĂŠ, pontos distintos possuem inversos distintos). Para verificar este fato, basta observar que o ponto P cujo inverso ĂŠ um certo ponto Pâ&#x20AC;&#x2122; estĂĄ univocamente determinado e ĂŠ justamente o inverso de Pâ&#x20AC;&#x2122; (ou seja, a transformação inversa de uma inversĂŁo ĂŠ ela mesma).
EUREKA! N°4, 1999
43
Sociedade Brasileira de Matemática
b) O inverso de um círculo que não passa pelo centro de inversão é um outro círculo. Consideremos uma inversão de centro O e constante k e tomemos um círculo C que não passa por O. Seja P um ponto de C, P’ o seu inverso e Q o outro ponto em que a reta OP corta C.
P
Q
P'
O
C
DcE FHGqp K M E Q`of_JXj]fS lP_UO`grTWV XjT OP[ S Uma propriedade fundamental do círculo é que o produto OP. OQ é igual a uma constante p (a potência de O em relação a C) para qualquer posição de P. Assim, OP ' OP.OP ' k = = OQ OP.OQ p Portanto, o inverso de C pode ser obtido assim: para cada ponto Q de C, tomamos o ponto P' sobre a semi-reta OQ tal que OP' = (k/p) OQ. Este tipo de transformação é chamado de homotetia e sempre transforma uma figura em outra semelhante (ela faz uma ampliação ou redução da figura, conforme k/p seja maior ou menor que 1). Em particular, o transformado de um círculo por homotetia é sempre um outro círculo. Em resumo: o inverso de um círculo (que não passa pelo centro de inversão O) é um outro círculo, obtido através de uma homotetia de centro O (para você pensar: como será o inverso de um círculo que passa por O?). Agora estamos em condições de entender como é construído o logotipo da OBM. O ponto de partida é a figura abaixo: dois círculos concêntricos, com cinco círculos de raios iguais encaixados entre eles.
DcE FBGesUt^M hJS`QPR$SUlP_ hPaJXjR$E l`aBueTZE Q`TZS^T V XjTZO`[ SJ] E FPOJa`E ] _PQJR$XY_ T V XYTZOP[ SJ] TZS`QJTfvJQPR$XYE TZS`] EUREKA! N°4, 1999
44
Sociedade Brasileira de Matemática
Só é possível encaixar estes 5 círculos para um determinado valor da razão R/r entre os raios dos círculos externo e interno. De modo mais geral, vejamos qual deve ser esta razão para que n círculos possam ser encaixados entre os dois círculos concêntricos. O diâmetro de cada um dos círculos iguais é a diferença R − r entre os raios dos círculos concêntricos. Por outro lado, seus centros formam um polígono regular de n lados, inscrito em um R+r círculo de raio concêntrico aos dois círculos iniciais, como mostra a 2 figura abaixo.
M
A
O
D E FHGqw K x OJaPQPlPS yUhPS`]Z] V of_J[f_`QJTfaPE bZaPX z TWV XjTfOP[ SP]{_PQPR|Xj_^T V XjTfOP[ SP] TfSPQPTfvPQJR$XjE TZSP]Z} No triângulo retângulo OAM , a hipotenusa OA mede
R+r eo 2
R−r e é oposto a um ângulo igual a 180°/n. Assim: 2 R−r R+r = sen(180° / n) 2 2 ou, desenvolvendo: R 1 + sen(180°/ n) = r 1 − sen(180° / n) No nosso caso, em que n = 5, devemos ter R 1 + sen(36° ) = r 1 − sen(36° ) Quando os raios R e r estão nessa proporção, é possível encaixar cinco círculos iguais entre os dois círculos concêntricos. cateto AM mede
EUREKA! N°4, 1999
45
Sociedade Brasileira de MatemĂĄtica
Para terminar de formar o logotipo, tomamos o conjunto formado pelos dois cĂrculos concĂŞntricos e pelos cinco cĂrculos de raios iguais encaixados entre eles e aplicamos uma transformação de inversĂŁo.
O
~c Â&#x20AC;H eÂ&#x201A;Ut^Â&#x192; Â&#x201E; Â&#x2026;`Â&#x20AC;PÂ&#x2026;JÂ&#x2020;$ hPÂ&#x2026; lJa^Â&#x192; Â&#x2021;?Â&#x2C6;5Â&#x2030;qÂ&#x2026;PÂ&#x160;`Â&#x2020;| lPÂ&#x2026; hPÂ&#x2026;JÂ&#x2039; Â&#x152;`oZ_PÂ&#x2039;jÂ?ZÂ&#x17D;`Â&#x2026; A menos que o centro de inversĂŁo seja o seu centro comum, os inversos dos cĂrculos concĂŞntricos nĂŁo sĂŁo concĂŞntricos. AlĂŠm disso, os inversos dos cinco cĂrculos iguais nĂŁo tem mais raios iguais, dando o aspecto irregular do logotipo. Os raios dos cĂrculos tornam-se mais desiguais quanto mais o centro de inversĂŁo se afasta do centro dos cĂrculos concĂŞntricos. Note, no entanto, que as propriedades de tangĂŞncia sĂŁo preservadas, em virtude da injetividade da inversĂŁo, que faz com que o nĂşmero de pontos de interseção de figuras seja preservado atravĂŠs da transformação. Para produzir a animação do logotipo, basta girar o conjunto de cinco cĂrculos de raios iguais na figura original: seus transformados por inversĂŁo mudarĂŁo de tamanho e posição Ă medida que isto ocorre. Se vocĂŞ quiser, pode experimentar com as propriedades dessa transformação visitando a pĂĄgina da OBM na Internet. Basta clicar sobre o sĂmbolo animado da OBM, ou ir diretamente ao seguinte endereço: http: //www.obm.org.br/logotipo.htm. VocĂŞ encontrarĂĄ uma pĂĄgina interativa que permite variar as proporçþes do sĂmbolo atravĂŠs da escolha do centro de inversĂŁo. VocĂŞ atĂŠ poderĂĄ criar sĂmbolos diferentes mudando o nĂşmero de anĂŠis! Na verdade, os "designers" que criaram o logotipo da OBM utilizaram um programa parecido, para ajustar o tamanho e posição relativa dos anĂŠis de modo a produzir uma figura agradĂĄvel do ponto de vista visual. Este ĂŠ um bom exemplo do emprego da MatemĂĄtica em artes visuais. HĂĄ casos notĂĄveis de artistas, como Escher, que usaram a MatemĂĄtica como ferramenta essencial em seu processo criativo. Em futuros nĂşmeros da voltaremos a falar de inversĂŁo, estudando suas propriedades em mais detalhe e mostrando outras aplicaçþes. Aguardem!
Â?9Â?;Â&#x2018;<Â?9Â&#x2019;?Â&#x201C; Â&#x201D;
EUREKA! N°4, 1999
46
Sociedade Brasileira de MatemĂĄtica
Â&#x2022;Â&#x2014;Â&#x2013;Â&#x2122;Â&#x2DC; Â&#x161;5Â&#x203A;%Â&#x153;Â&#x2122;Â?Â&#x2014;Â&#x2022;Â&#x;Â&#x17E;5Â?ÂĄÂ Â&#x2014;¢-Â&#x2013;Â&#x2122;ÂŁ-Â&#x2DC; Â?U¤ ÂĽÂŚÂ&#x2022;ÂĄÂ Â&#x2014;¢%Â&#x2013;Â&#x2122; UÂ&#x2013;Â&#x2122;Â&#x2022; §Â&#x2014;Â&#x2013;Â&#x2122;Â&#x2022; ¨ Š ÂŞZÂŤfÂŹ  Ž°¯ZÂą ²ZÂł ÂŻZ´ZÂŞf°¯ZÂŹ ÂľfÂŞZÂą ÂŻZÂł ÂśZÂŻZÂł ¡!¸f³ºš ²f³° ÂŻZÂł ¸ZÂź ½ž ¯ZÂśZÂŻfÂł šW²Z¡PÂźZ²Z³¿³°²ZÂł ÂŹ ¸c  ²Z¡Ă&#x20AC;¸Z³°à Determine todas as soluçþes de xy = yx com x e y racionais positivos.
11)
Ă&#x201A;?Ă&#x192; Ă&#x201E;$Ă&#x2026; Ă&#x2020;JĂ&#x2021;PĂ&#x192;%Ă&#x2C6; Ă&#x2030; Ă&#x160;<Ă&#x2039;JĂ&#x152;Ă?Ă&#x201E;|Ă&#x192; Ă&#x17D; Ă?<Ă&#x201E;$Ă? Ă&#x2030;PĂ&#x152;Ă&#x2019;Ă&#x2018;jĂ&#x192;Ă&#x201C;Ă&#x2C6; Ă&#x2039; Ă&#x201A;?Ă&#x201D;$Ă&#x201E;$Ă&#x2022;JĂ&#x2039; Ă&#x2013;?Ă&#x201D;$Ă&#x2014;JĂ&#x2018;jĂ&#x192; Ă&#x152;qĂ&#x2DC; Ă&#x2122;
i)
Ă&#x2030; evidente que x = y â&#x2C6;&#x2C6;
ii)
Suponha x â&#x2030; y, seja entĂŁo
ay = y â&#x2020;&#x2019; y = a
1 a a â&#x2C6;&#x2019;1
+
satisfaz a equação.
Ă&#x2122;
x = a (a â&#x2C6;&#x2C6; y
e, fazendo a â&#x2C6;&#x2019; 1 =
+),
logo:
p p+q ,a = , temos : q q
q   y =  p + q  p  q    ďŁ ďŁ¸  q+ p   p+q p x =    ďŁŹďŁ q 
JĂĄ que p e q sĂŁo primos entre si, p + q e q tambĂŠm sĂŁo primos entre si, e portanto devemos garantir que p p + q e p q sejam inteiros. Necessariamente devemos ter p = 1; se nĂŁo vejamos: suponha que p â&#x2030;Ľ 2 e que q = sp para algum inteiro positivo s; daĂ s p < p + q < ( s + 1) p (pois (s + 1)p â&#x20AC;&#x201C; sp > p . sp â&#x20AC;&#x201C; 1 â&#x2030;Ľ p) e nĂŁo teremos que nos obriga fazer p = 1.
Ă&#x161;<Ă&#x203A; Ă&#x153; Ă?PĂ&#x17E;|Ă&#x; Ă PĂĄJĂ&#x203A; â
  1 + q  q +1  x =    ďŁ q  Para x â&#x2030; y, teremos:  q  1 + q    = y   q  ďŁ ďŁ¸  EUREKA! N°4, 1999
47
p
p + q sendo um inteiro, o
Sociedade Brasileira de MatemĂĄtica
onde q ĂŠ um inteiro maior do que ou igual a 1. Ă&#x2030; fĂĄcil verificar que tais x e y sĂŁo racionais e sĂŁo soluçþes do problema, para todo q â&#x2030;Ľ 1 natural.
ĂŁ
ĂŁ
Dado n â&#x2C6;&#x2C6; determine determine o maior k â&#x2C6;&#x2C6; tal que existam conjuntos A1, A2,â&#x20AC;Ś, Ak contidos em {1, 2, â&#x20AC;Ś, n} de forma que Ai â&#x160;&#x201E; Aj para todo i â&#x2030; j.
13)
ä?Ă&#x203A; Ă&#x17E;$Ă&#x; ĂĽJĂĄPĂ&#x203A;%ĂŚ ç è Ă&#x203A; ĂŠĂ&#x2019;Ă&#x203A; ĂŞPĂ JĂŤjĂŠĂ?Ă&#x203A;%ĂŹ<Ă ĂŞPĂŽ%ĂŻ Ă&#x; Ă°$ĂŞUĂą<çPĂŤjĂ&#x203A; â Observemos inicialmente que C nk =
n! â&#x2030;¤ C n[n / 2 ] para todo k com k! (n â&#x2C6;&#x2019; k )!
0 â&#x2030;¤ k â&#x2030;¤ n (onde [n/2] ĂŠ o Ăşnico inteiro tal que [n/2] â&#x2030;¤ n/2 â&#x2030;¤ [n/2] + 1). De fato, C nk +1 n â&#x2C6;&#x2019; k = â&#x2030;Ľ 1 pois n > 2k e portanto se k < n / 2, k +1 C nk n â&#x2030;Ľ 2k + 1, ou seja n â&#x20AC;&#x201C; k â&#x2030;Ľ k + 1. Portanto, se k < n / 2, C nk â&#x2030;¤ C n[n / 2 ] e se
k > n / 2, C nk = C nn â&#x2C6;&#x2019; k â&#x2030;¤ C n[n / 2 ] , pois n â&#x2C6;&#x2019; k < n / 2. Seja A â&#x160;&#x201A; {1, 2,â&#x20AC;Ś, n} e P = (a1, a2, â&#x20AC;Ś, an) uma permutação de {1, 2, â&#x20AC;Ś, n}. Dizemos que P passa por A se existe m â&#x2030;¤ n com A = {a1, a2, â&#x20AC;Ś, am}. Se A tem m elementos existem exatamente m! (n â&#x20AC;&#x201C; m)! permutaçþes que passam por A. Como n! n! = C nm â&#x2030;¤ C n[n / 2 ] = , donde [n / 2]!(n â&#x2C6;&#x2019; [n / 2])! m!(n â&#x2C6;&#x2019; m)! m! (n â&#x20AC;&#x201C; m)! â&#x2030;Ľ [n / 2]!(n â&#x2C6;&#x2019; [n / 2])! para todo m â&#x2030;¤ n.
Note agora que se A1, A2, â&#x20AC;Ś, Ak â&#x160;&#x201A; {1, 2, â&#x20AC;Ś, n} sĂŁo tais que, para todo i â&#x2030; j Ai â&#x160;&#x201E; Aj , entĂŁo, se i â&#x2030; j, nenhuma permutação passa por Ai e Aj ao mesmo tempo. Se mi ĂŠ o nĂşmero de elementos do conjunto Ai (1 â&#x2030;¤ i â&#x2030;¤ k ), podemos concluir k
que
â&#x2C6;&#x2018; m !(n â&#x2C6;&#x2019; m)!â&#x2030;¤ n!â&#x2021;&#x2019; k.[n / 2]! (n â&#x2C6;&#x2019; [n / 2])!â&#x2030;¤ n!â&#x2021;&#x2019; k â&#x2030;¤ C [
n / 2] . n
i
i =1
Por outro lado, hĂĄ C n[n / 2 ] subconjuntos de {1, 2, â&#x20AC;Ś, n} com exatamente [n / 2] elementos e, obviamente, se A e B sĂŁo dois subconjuntos distintos de [n / 2] elementos de {1, 2,â&#x20AC;Ś, n} entĂŁo A â&#x160;&#x201E; B, de modo que o maior k que satisfaz as condiçþes do enunciado ĂŠ C n[n / 2 ].
EUREKA! N°4, 1999
48
Sociedade Brasileira de MatemĂĄtica
15)
Considere uma seqßência de triângulos retângulos AnBnCn no plano cuja hipotenusa seja BnCn, com as seguintes condiçþes:
i) ii) iii)
A1B1 = A1C1 = 1; Bn+1 = Bn e An+1 = Cn para todo n â&#x2C6;&#x2C6; ; An+1Cn+1 ĂŠ congruente Ă altura de An em relação a BnCn.
ò
Mostre que qualquer ponto do plano pertence a infinitos triângulos AnBnCn.
Ăł<Ă´ ĂľPĂś ájĂľJĂ´ ĂľUĂ´ ø Ăš ĂşĂ&#x201C;Ăľ^ĂťJĂź Ă˝$Ăš ĂžJĂżPĂź ø ĂľPĂ´ ĂľUĂś Ăź Ăź Ă&#x2019;ø +ĂľJĂşĂ&#x201C;Ăś Ăź ĂťUĂ´ Ăľ Ă˝ Ăľ Ăľ ĂľPĂť
Sejam an, bn e cn os comprimentos dos lados BnCn, AnCn e AnBn do triângulo AnBnCn e Îąn o ângulo An BË&#x2020; n C n . Ă&#x2030; suficiente mostrarmos que as hipotenusas dos triângulos crescem infinitamente e que os triângulos dĂŁo infinitas voltas em torno do ponto B1. Em outras palavras, devemos mostrar que a seqßência (an) ĂŠ ilimitada e que Sn =
n
â&#x2C6;&#x2018;Îą
k
tambĂŠm cresce ilimitadamente.
k =1
Seja hn a altura relativa Ă hipotenusa BnCn. Ă&#x2030; imediato que anhn = bncn. AlĂŠm disso, pelas condiçþes (ii) e (iii), segue que an = cn+1 e bn+1 = hn, donde concluĂmos que bn+1cn+1 = bncn = ... = b1c1 = 1. DaĂ, anhn = 1, para todo natural n, e como an+12 = bn+12 + cn+12, vem 1 a n2+1 = a n2 + 2 , a12 = 2. an 2 Seja pn = an . EntĂŁo, para mostrar que an ĂŠ ilimitada, ĂŠ suficiente mostrarmos que (pn) ĂŠ ilimitada. Note que an+1 > an, para todo natural n, de modo que (pn) ĂŠ crescente. Temos 1 , p1 = 2. p n +1 = p n + pn Vamos mostrar por indução que sempre temos Pn â&#x2030;Ľ n + 1. De fato, 1 1 n+2 = â&#x2030;Ľ n + 2. P1 = 2 â&#x2030;Ľ 2, e Pn â&#x2030;Ľ n +1 implica Pn+1 = Pn + â&#x2030;Ľ n +1 + Pn n +1 n +1 Assim, Pn (e portanto an) ĂŠ ilimitada. Vamos agora provar que (Sn) ĂŠ EUREKA! N°4, 1999
49
Sociedade Brasileira de MatemĂĄtica
ilimitada. Temos Îą n > senÎą n =
bn 1 = bn bn +1 > bn2+1 (pois bn = decresce an cn
quando n cresce). Como bn2+1 = a n2+1 â&#x2C6;&#x2019; a n2 , Îą 1 + Îą 2 + ... + Îą n > (a 22 â&#x2C6;&#x2019; a12 ) + (a 32 â&#x2C6;&#x2019; a 22 ) + ... + (a n2+1 â&#x2C6;&#x2019; a n2 ) = a n2+1 â&#x2C6;&#x2019; a12 â&#x2030;Ľ n + 2 â&#x2C6;&#x2019; 2, e portanto S n =
18)
n
â&#x2C6;&#x2018;Îą
k ĂŠ ilimitado.
k =1
Seja Îą a maior raiz real da equação x3 â&#x20AC;&#x201C; 3x2 + 1 = 0. Prove que [Îą2004] ĂŠ divisĂvel por 17. Obs: [y] ĂŠ o Ăşnico inteiro tal que [y] â&#x2030;¤ y < [y] + 1.
!#"%$ & $ ' $ ( ") $+* -,
Considere Îą, β e Îł as raĂzes de f(x) = x3 â&#x20AC;&#x201C; 3x2 + 1, Îą a maior delas, e S(n) = Îąn + βn + Îłn . (Obs. Ă&#x2030; fĂĄcil de se ver que todas as raĂzes sĂŁo reais. De fato, f (â&#x2C6;&#x2019;1) < 0 < f (0), e f (1) < 0 < f (3)). Com estas consideraçþes e as relaçþes de Girard para f(x)= 0, tem-se que: (I)
e S(0) = Îą0 + β 0 + Îł 0 = 3, S(1) = Îą1 + β 1 + Îł1 = 3 S(2) = Îą2 + β 2 + Îł 2 = (Îą1 + β 1 + Îł1)2 â&#x20AC;&#x201C; 2 (ιβ + ιγ + βγ ) = 9 â&#x20AC;&#x201C; 0 = 9
(II)
f(x) = x2 . ( x â&#x20AC;&#x201C; 3 ) +1 > 0 para todo x â&#x2030;Ľ 3. Como f ĂŠ contĂnua e f( 2,87 ) â&#x2030;&#x2026; â&#x20AC;&#x201C; 0,07 < 0, segue-se que 2,87 < Îą < 3. Por outro lado, de (I) tem-se Îą 2 + β 2 + Îł 2 = 9 . Assim, β 2 + Îł 2 < 9 â&#x20AC;&#x201C; (2,87)2 < 1. Sendo f(0) = 1, conclui-se que Îą e β sĂŁo nĂŁo nulas . Logo, 0 < β 2 + Îł 2 < 1 e conseqĂźentemente , 0 < β 2 < 1 e 0 < Îł 2 < 1.
(III)
P(x) = xn â&#x20AC;&#x201C; 3 . f(x) = x n â&#x20AC;&#x201C; 3x n â&#x20AC;&#x201C; 1 + x n â&#x20AC;&#x201C; 3 ( n â&#x2030;Ľ 3) ĂŠ um polinĂ´mio, tal que:
EUREKA! N°4, 1999
50
Sociedade Brasileira de Matemática
P (α ) = α n − 3α n −1 +α n −3= 0 n n −1 + β n −3 = 0 P ( β ) = β − 3β n n −1 + γ n −3 = 0 P (γ ) = γ − 3γ Adicionando-se membro a membro , obtém-se para n ≥ 3, a seguinte relação de recorrência: S( n ) – 3. S ( n –1 ) + S ( n – 3 ) = 0 ,
S(n) = 3. S (n –1) – S (n – 3) (IV)
ou, melhor ainda
(n ≥ 3)
Sendo S(0), S(1) e S(2) inteiros por (I), podemos concluir, através da relação de recorrência acima e indução sobre n, que S(n) será um inteiro para qualquer natural n. Assim, S(2004) = α 2004 + β 2004 + γ 2004 é um inteiro. Como 0 < β 2 < 1 e 0 < γ 2 < 1 ....... por (II), segue-se que 0 < β2004 + γ2004 = (β 2) 1002 + (γ2 )1002 < β 2 + γ 2 < 1. Logo, S(2004) – 1 < α 2004 < S(2004), ou seja, α 2004 = S(2004) – 1.
(V)
A relação de recorrência obtida em (III) implica em particular que S (n) = 3S (n − 1) − S (n − 3) (mod 17) para todo n ≥ 3, o que permite construir a tabela seguinte de S(n) (mod 17): S(0) ≡ 3 (mod 17), S(1) ≡ 3 (mod 17), S(2) ≡ 9 (mod 17), S(3) ≡ 7 (mod 17), S(4) ≡ 1 (mod 17), S(5) ≡ 11(mod 17), S(6) ≡ 9 (mod 17), S(7) ≡ 9 (mod 17), S(8) ≡ 16 (mod 17), S(9) ≡ 5 (mod 17), S(10) ≡ 6 (mod 17), S(11) ≡ 2 (mod 17), S(12) ≡ 1 (mod 17), S(13) ≡ 14 (mod 17), S(14) ≡ 6 (mod 17), S(15) ≡ 0 (mod 17), S(16) ≡ 3 (mod 17), S(17) ≡ 3 (mod 17), S(18) ≡ 9 (mod 17).
(VI)
Note que S(16) ≡ S(0) (mod 17), S(17) ≡ S(1) (mod 17) e S(18) ≡ S(2) (mod 17). Isso permite mostrar que S(n +16) ≡ S(n) (mod 17) para todo n ∈ . . De fato, por indução, S(n + 16) = 3S(n + 15) – S (n + 13) ≡ 3S(n – 1) – S(n – 3) (mod 17) = S(n)( mod 17), se n ≥ 3 (usamos como base de indução os casos
EUREKA! N°4, 1999
51
Sociedade Brasileira de MatemĂĄtica
n = 0, n = 1 e n = 2). Como conseqßência, concluimos que S(n) â&#x2030;Ą S(n + 16p) (mod 17), para todo p â&#x2C6;&#x2C6; / . (VII)
Como 2004 = 4 + 16.p , com p = 125, temos de (IV), (V) e (VI): S(2004) â&#x2030;Ą S(4) â&#x2030;Ą 1 (mod 17). Portanto,  Îą2004 = S(2004) â&#x20AC;&#x201C; 1 ĂŠ divisĂvel por 17.
Nota: A demonstração acima foi baseada na resolução de um problema de enunciado similar a este, proposto pela França e não utilizado na IMO de 1988. 19)
a) Determine o número måximo de regiþes em que n retas podem dividir o plano. b) Determine o número måximo de regiþes em que n planos podem dividir o espaço.
0)1 23 4 * ") 5 26& 7 1 8 1 9 ( & 2:,
a)
Observe inicialmente os casos abaixo:
i) ii)
Uma reta divide o plano em duas regiĂľes: 1 + 1. Uma segunda reta ĂŠ dividida pela anterior no mĂĄximo em duas partes e mais duas regiĂľes sĂŁo acrescentadas, ou seja: com 2 retas temos: ( 1 + 1 + 2) regiĂľes (no mĂĄximo.) iii) Uma terceira reta ĂŠ dividida pelas duas retas anteriores no mĂĄximo em trĂŞs partes e acrescentando entĂŁo mais trĂŞs regiĂľes, ou seja: com 3 retas temos: (1 +1 +2 +3) regiĂľes (no mĂĄximo.) iv) Suponha agora que tenhamos n retas; a n-ĂŠsima reta ĂŠ dividida pelas (n â&#x20AC;&#x201C; 1) outras retas no mĂĄximo em n partes e evidentemente acrescentando n regiĂľes, o que nos darĂĄ: (1 + 1 + 2 + 3 +â&#x20AC;Ś+ n) n2 + n regiĂľes ou + 1 regiĂľes. 2 Obs: Se as retas estĂŁo em posição geral todas as desigualdades acima sĂŁo igualdades. b)
Observe que:
EUREKA! N°4, 1999
52
Sociedade Brasileira de MatemĂĄtica
i)
ii) iii)
Quando temos dois planos, o segundo plano intersecta o primeiro plano no mĂĄximo atravĂŠs de uma reta e o segundo plano ĂŠ dividido em duas partes. Um terceiro plano intersecta os planos anteriores em no mĂĄximo duas retas e o terceiro plano ĂŠ dividido em 4 partes. Um quarto plano intersecta os planos anteriores em no mĂĄximo 3 retas e o quarto plano ĂŠ dividido em 7 partes.
Em geral, o k-ĂŠsimo plano intersecta os anteriores em no mĂĄximo k â&#x20AC;&#x201C; 1 retas, (k â&#x2C6;&#x2019; 1)2 + (k â&#x2C6;&#x2019; 1) + 1 = k 2 â&#x2C6;&#x2019; k + 2 regiĂľes que o dividem em no mĂĄximo 2 2 (pelo Ătem a), ou seja, ao ser acrescentado o k-ĂŠsimo plano sĂŁo criadas no k2 â&#x2C6;&#x2019; k + 2 mĂĄximo novas regiĂľes do espaço. 2 Como um plano divide o espaço em duas regiĂľes temos no mĂĄximo n k2 â&#x2C6;&#x2019; k + 2 regiĂľes em que k planos dividem o espaço. 2+â&#x2C6;&#x2018; 2 k =2 n(n + 1)(2n + 1) Sabendo que 12 + 2 2 + 3 2 + ... + n 2 = , temos um total de 6 n 3 + 5n + 6 regiĂľes (no mĂĄximo). 6 Obs: Se os planos estĂŁo em posição geral todas as desigualdades acima sĂŁo igualdades. ;=< >-? @ >-ACB D7< EGF E H F I6BJ>-KC<LB EME < NOAQP R F ESKT< EMH I6< U N F D BJEWV:XZY[V]\^FWV]_`KQB E`F KT@ P R F E B >-? F I6@ < I6F E a bdcOegf)h ikjml nobpcOegq e)n r3e)nos=eutdlvc3r6l w x)y z { $ 4( | $ &
Maria convidou 9 garotos e 8 garotas para sua festa de aniversårio. Ela preparou camisetas com os números de 1 a 18 e ficou com a de número 1 e distribuiu as demais para seus convidados. Em determinado momento, em que todos estavam dançando, a soma dos números de cada casal era um quadrado perfeito. Quais pares estavam dançando? -,
A maior soma possĂvel dos nĂşmeros de um casal ĂŠ 18 + 17, que ĂŠ menor que 62. Assim, os valores das somas dos nĂşmeros de cada casal devem valer 4, 9, EUREKA! N°4, 1999
53
Sociedade Brasileira de MatemĂĄtica
16 ou 25, de modo que os pares de 18, 17 e 16 devem ser 7, 8 e 9, respectivamente. Portanto o par de 2 deve ser 14 (pois nĂŁo pode ser o prĂłprio 2 e o 7 jĂĄ ĂŠ par do 18), o do 11 deve ser 5 (pois 14 jĂĄ ĂŠ par de 2), o de 4 deve deve ser 12 (pois 5 jĂĄ ĂŠ par de 11), o de 13 deve ser 3 (pois 12 jĂĄ ĂŠ par de 4), o de 1 deve ser 15 (pois 3 jĂĄ ĂŠ par de 13 e 8 jĂĄ ĂŠ par de 17) e o de 10 deve ser 6 (pois 15 jĂĄ ĂŠ par de 1). Assim, os pares sĂŁo 18 e 7, 17 e 8, 16 e 9, 15 e 1, 14 e 2, 13 e 3, 12 e 4, 11 e 5, 10 e 6. x & | , "%$ |~} | 6 |Â Â&#x20AC;)Â&#x201A;Â&#x201E;Â&#x192;# + Â&#x2026;7| 6 *~ Â&#x2021;Â&#x2020;=| 3 4$ |~ ~Â&#x2C6; Â&#x201A;Â&#x201E;} Â&#x201A;d z |~ |SÂ&#x2030; * ( !|SÂ&#x2030; * & | | ' 6 Â&#x160; * * Â&#x2C6; Â&#x2039; Â&#x152; 6 Â? | * $ z | 6| Â&#x2026;Â&#x17D;* + Â? * ( 3 6 & |J* * & Â? 6 Â&#x152; ! Â&#x2026;7| Â&#x201A; x)y z { $ & 6Â&#x2026;7 Â&#x2039; 6
AD Ê a bissetriz interna do ângulo A do triângulo ABC, com D sobre o lado BC. As bissetrizes dos ângulos ADB e ACB concorrem em E, com E sobre o lado AB. Determine a medida do ângulo BAC. O problema Ê equivalente ao problema 5 da pågina 3 da resolvido na pågina 6 da mesma.
Â?9Â?;Â&#x2018;<Â?9Â&#x2019;?Â&#x201C; Â&#x201D;
No.3,
x)y z " z | $ |
SĂŁo dadas 13 moedas, das quais 12 tĂŞm o mesmo peso. NĂŁo se sabe se a dĂŠcima terceira moeda ĂŠ mais leve ou mais pesada que as demais. Mostre que ĂŠ possĂvel determinar a moeda diferente empregando trĂŞs pesagens em uma balança de braços. Isto ainda seria possĂvel com 14 moedas? -,
Das 13 moedas selecionaremos dois grupos de 4 moedas A1, A2, A3, A4 e B1, B2, B3, B4 e as pesamos. Sobram 5 moedas, C1, C2, C3, C4 e C5. Temos duas possibilidades: i)
i.1) i.2)
A balança fica equilibrada. Neste caso a moeda diferente estĂĄ entre as 5 restantes. Pesamos agora A1, A2, A3 e C1, C2, C3 (A1, A2 e A3 sĂŁo padrĂŁo). Temos mais duas possibilidades EquilĂbrio. A moeda diferente ĂŠ C4 ou C5. Pesamos A1 e C4. Se der diferente a moeda diferente ĂŠ C4, e se houver equilĂbrio ĂŠ C5. DesequilĂbrio. Vamos supor sem perda de generalidade que o grupo C1C2C3 ĂŠ mais pesado que A1A2A3. Nesse caso a moeda diferente ĂŠ C1, C2 ou C3 e ĂŠ mais pesada que as outras. Pesamos C1 e C2. Se houver desequilĂbrio a mais pesada ĂŠ a diferente. Se houver equilĂbrio ĂŠ a C3.
EUREKA! N°4, 1999
54
Sociedade Brasileira de MatemĂĄtica
ii)
ii.1)
ii.2)
ii.3)
DesequilĂbrio. Vamos supor sem perda de generalidade que o grupo A1A2A3A4 ĂŠ mais pesado que B1B2B3B4. Pesamos agora A1A2B1 e A3B2C1. Temos trĂŞs possibilidades: EquilĂbrio. Nesse caso a moeda diferente ĂŠ A4, B3 ou B4. Se for A4 ĂŠ mais pesada e se for B3 ou B4 ĂŠ mais leve. Pesamos B3 e B4. Se houver equilĂbrio a diferente ĂŠ A4. Se nĂŁo a mais leve das duas ĂŠ a diferente. O grupo A1A2B1 ĂŠ mais pesado. Nesse caso a moeda diferente ĂŠ A1, A2 ou B2. Pesamos A1 e A2. Se houver equilĂbrio a diferente ĂŠ B2, se nĂŁo ĂŠ a mais pesada das duas. O grupo A1A2B1 ĂŠ mais leve. Nesse caso a moeda diferente ĂŠ B1 ou A3. Pesamos B1 e C1. Se houver equilĂbrio a moeda diferente ĂŠ A3, se nĂŁo ĂŠ B1.
Se tivermos 14 moedas nĂŁo ĂŠ possĂvel determinar sempre a moeda diferente. Se na primeira pesagem pesamos dois grupos de 5 ou mais moedas e nĂŁo houver equilĂbrio, a moeda diferente pode ser qualquer uma das pelo menos 10 envolvidas na pesagem. Como cada pesagem tem apenas 3 resultados possĂveis, as duas Ăşltimas pesagens dĂŁo no total no mĂĄximo 9 resultados diferentes, que nĂŁo permitem distinguir todas as (pelo menos 10) possibilidades de moeda diferente. Se pesarmos dois grupos de 4 ou menos moedas na primeira pesagem e houver equilĂbrio sobram pelo menos 6 moedas para anĂĄlise. Se na segunda pesagem usarmos 4 ou mais dessas moedas de situação desconhecida e nĂŁo houver equilĂbrio, qualquer uma delas pode ser a diferente e a Ăşltima pesagem (que sĂł tem 3 resultados possĂveis) nĂŁo pode determinĂĄ-la com segurança. Se na pesagem usamos 3 ou menos das moedas de situação desconhecida sobram pelo menos 3 em situação desconhecida. Se na terceira pesagem usamos duas delas ou todas as 3 e houver desequilibrio qualquer uma dessas moedas pode ser a diferente, e nĂŁo concluĂmos nossa tarefa. Se usamos uma ou nemhuma e houver equilĂbrio sobram pelo menos duas de situação desconhecida, e em qualquer caso nĂŁo ĂŠ sempre possĂvel determinar a moeda diferente.
Â?9Â?;Â&#x2018;<Â?9Â&#x2019;?Â&#x201C; Â&#x201D;
Nota: O problema 10, proposto na pågina 59 da No. 2 generaliza este problema. Tente resolvê-lo agora, adaptando para a situação geral os argumentos desta solução!
EUREKA! N°4, 1999
55
Sociedade Brasileira de MatemĂĄtica
 Â&#x2014;¢-Â&#x2013;Â&#x2122;ÂŁ5Â&#x2DC; Â?U¤ ÂĽÂ&#x2122;Â&#x2022;  U¢-Â&#x2013;Â&#x2122; UÂ&#x2013;Â&#x2122;Â&#x2022; § Â&#x2013;Â&#x2122;Â&#x2022; Â&#x2018;
Â&#x2019;Â&#x201D;Â&#x201C;Â&#x2013;Â&#x2022;+Â&#x2014;OÂ&#x2DC; Â&#x2122;Â&#x2013;Â&#x161;TÂ&#x203A;Â&#x201E;Â&#x201C;TÂ&#x153;%Â&#x201C;Â&#x201E;Â? Â&#x17E;TÂ&#x2DC; Â&#x; Â&#x201C;T Â&#x161;ÂĄÂ&#x17E;Â&#x2013;Â&#x2022;+Â&#x2014;OÂ&#x2DC; Â&#x161;Â&#x2013; Â&#x153;]Â&#x201C;Â&#x2013;Â? ¢TÂŁ]¤Â&#x2013;Â&#x17E;QÂ&#x153;%Â&#x2122;Â&#x2013;Â&#x201C;TÂ&#x153; ÂĽ+ ŒÂ&#x201C;Â&#x2013;§TÂ? Â&#x17E;Â&#x2013;Â&#x203A;ÂĄÂ&#x161;Â&#x2013;Â&#x153; ÂĽ+ ¨Â&#x201C;Š¼ªÂ&#x201C;TÂ&#x153;]Â&#x; Â&#x201C;Â&#x2013;Â&#x153;%Â&#x17E;Â&#x201E;Â&#x153;]¢TÂŤÂ&#x2013;Â&#x17E;TÂ&#x153;]Â&#x; ¤TÂ&#x17E;QÂ&#x153;-Â&#x2122;TÂ&#x17E;Â&#x201E;Â&#x2022;TÂ&#x201C;+Â&#x2014;ÂŹÂ&#x201C;TÂ&#x153; ÂĽÂŞÂ ÂŚÂ&#x201C;T§Â&#x2013;Â? Â&#x17E;TÂ&#x203A;Â&#x201E;Â&#x161;TÂ&#x153; ÂĽ+Â&#x161;T ¨Â&#x161;ÂĄÂ&#x201C;Â&#x2013;Â&#x153;^ÂĽÂŞÂ ÂŚÂÂŻÂŽJÂ&#x2DC; Â&#x203A;ÂĄÂ&#x201C;Â&#x2013;Â&#x153;%Â&#x2022;Â&#x2013;°TÂ&#x203A;Â&#x201E;Â&#x17E;T ŒÂ&#x201C;Â&#x2013;Â&#x153; .
20)
Diga se existe uma função polinomial de Âą o intervalo (0,+â&#x2C6;&#x17E;) = {x â&#x2C6;&#x2C6; ²
21)
2
em Âą cuja imagem seja
x > 0}. a + b = c.
a) Encontre todas as soluçþes inteiras da equação b) Encontre todas as soluçþes inteiras da equação
3
a + 3 b = 3 c.
22)
Sejam Îą , β , Îł , δ os ângulos de um quadrilĂĄtero, nessa ordem. Prove que esse quadrilĂĄtero ĂŠ inscritĂvel se, e somente se, a relação ιβ + ιδ + γβ + γδ = Ď&#x20AC; 2 ocorre.
23)
Seja ABC um triângulo qualquer de ortocentro H e sejam ha, hb, hc os comprimentos das alturas relativas a A, B, C respectivamente. Prove ____ ____ ____ 1 que ha . AH + hb . BH + hc CH = (a 2 + b 2 + c 2 ). 2 Na loteria de Truchilândia, cada bilhete tem um nĂşmero de trĂŞs algarismos que usa somente os algarismos 1, 2, 3, 4 (ĂŠ permitido repetir os dĂgitos). Um bilhete ĂŠ ganhador se coincide em pelo menos duas posiçþes com o nĂşmero sorteado. Um apostador quer compar vĂĄrios bilhetes, de maneira que um deles ganhe com certeza, mas gastando o mĂnimo possĂvel. Determinar quantos bilhetes deve comprar e quais bilhetes deve comprar.
24)
Obs. Se o bilhete sorteado for o 423 entĂŁo 123 ĂŠ um bilhete ganhador, mas 243 nĂŁo ĂŠ. 25)
Durante o ano de 1998, uma pequena livraria, que abria nos sete dias da semana, vendeu no mĂnimo um livro por dia e um total de 600 livros no ano todo. Diga, justificando, se existiu, obrigatĂłriamente, um perĂodo de dias consecutivos onde foram vendidos exatamente 129 livros.
Nota:
Os problemas 21, 22, 23 e 24 foram propostos na 1a. lista de preparação para a X OlimpĂada de MatemĂĄtica do Cone Sul. O problema 25 foi proposto na IX OlimpĂada de MatemĂĄtica do Rio Grande do Norte.
EUREKA! N°4, 1999
56
Sociedade Brasileira de MatemĂĄtica
Â?Â&#x2014;¢5¢ ÂĽ5§{ÂĽ
A solução do problema 2 da 9a. OlimpĂada de MatemĂĄtica do Cone Sul, (cujo enunciado estĂĄ na pĂĄgina 22 da No.2) estĂĄ errada. Na verdade publicamos a solução de outro problema do banco, cujo enunciado era:
Â?9Â?;Â&#x2018;<Â?9Â&#x2019;?Â&#x201C; Â&#x201D;
Sejam H o ortocentro do triângulo ABC , nĂŁo retângulo, e M o ponto mĂŠdio do lado BC. A circunferĂŞncia circunscrita em P. Mostre que P, H, M sĂŁo colineares. A solução correta do problema 2 da 9a. OlimpĂada de MatemĂĄtica do Cone Sul ĂŠ como segue: Âł+´Â&#x201D;Âľ-ÂśÂ&#x201D;¡š¸+ÂşÂ&#x201E;½Ÿ
: Sejam H o ortocentro (interseção das alturas) do triângulo acutângulo ABC e M o ponto mÊdio do lado BC. Seja X o ponto em que a reta HM intersecta o arco BC (que não contÊm A) da circunsferência circunscrita a ABC. Seja Y o ponto de interseção da reta BH com a circunsferência, distinto de B. Demonstre que XY = BC. Solução:
Seja X' o simĂŠtrico de H em relação ao ponto M. Vamos mostrar que X â&#x2030;Ą X'. O quadrilĂĄtero HBX'C ĂŠ um paralelogramo, pois os pontos mĂŠdios de suas diagonais coincidem. EntĂŁo â&#x2C6; BX'C = â&#x2C6; BHC = 180° â&#x20AC;&#x201C; â&#x2C6; BAC. Segue que X' deve pertencer ao arco BC que nĂŁo contĂŠm A, donde X â&#x2030;Ą X'. Observe agora que o quadrilĂĄtero BYCX ĂŠ um trapĂŠzio inscritĂvel (pois BY X'C = XC ), donde BC = XY.
EUREKA! N°4, 1999
57
Sociedade Brasileira de Matemática
¥¦¢ § ¥ - C [¿ § ¢5 ¾ À
Á+¢ §T Â] ¡ % ÃT¢T Ä Q «T¢ ¡ %Â] ¦ % T ª O T Q ¥ª T T ] ] T - T ¦ ]Å Æ º¡» ¸ Ç+È µ%· »É³+Ê ¶ · Ë È » Ì »½¸+Æ ´ ¸ Í » Î
Eureka! Lembrei-me de Arquimedes quando tive a luz para a idéia que mexeu comigo e um punhado de alunos que até então estavam inertes face à necessidade de aprender Matemática. Descobri! Diria Arquimedes, radiante de felicidade, no bom português. E tal foi a emoção que vivi quando atinei para a solução que revolucionaria o ensino da Matemática numa tão carente escola pública. Tal luz, que me veio à mente, iluminou alguns alunos que poderiam hoje estar mergulhados, como muitos, nas trevas do descaso e do abandono do ensino público. Descobri! Que o saber dos números independe das classes sociais. É democrático! E nasce no espirito daqueles a quem seja dada a fagulha e uma palavra de confiança, de incentivo. A função da Matemática é profunda, é humanística, pois o bem estar da coletividade é o fim de tudo. Cabe, à Matemática a criação de espíritos disciplinados, mentes sadias e aprimoradas. E esse é o nosso escopo: estimulando, propiciando e melhorando o ensino dessa matéria nas Escolas Brasileiras.
Prof. Paulo Araripe Fortaleza-CE
EUREKA! N°4, 1999
58
Sociedade Brasileira de MatemĂĄtica
Â&#x2122;¤
žĂ&#x2030;Ă? Ă?Ă?
ÂĽĂ&#x2019;Ă&#x2018; Ă&#x2018;[ÂżOĂ&#x201C;^¼Œ¢ ÂĽ Â?[Ă&#x201D;5¢5Â?[Ă&#x2022;^ÂĽĂ&#x2019;Ă&#x2013;
Se vocĂŞ ĂŠ fanĂĄtico por MatemĂĄtica e deseja receber na sua casa a revista Ă&#x2014;šĂ&#x2DC;gĂ&#x2122;)Ă&#x2014;šĂ&#x161; Ă&#x203A;Â&#x201E;Ă&#x153; , faça o seu pedido escrevendo para: Secretaria da OlimpĂada Brasileira de MatemĂĄtica, Estrada Dona Castorina, 110 Jardim Botânico - Rio de Janeiro, RJ - CEP: 22460-320. O custo de cada exemplar avulso ou atrasado ĂŠ de R$4,00. VocĂŞ pode fazer uma assinatura anual o que darĂĄ direito a receber as publicaçþes do referido ano (mĂnimo 3 exemplares) por um valor promocional de R$10,00. Para isso, faça um depĂłsito no Banco do Brasil - AgĂŞncia 0598-3 - Conta N°52208-2 em nome do professor Eduardo Wagner. Envie-nos a fotocopia do depĂłsito e faça referĂŞncia aos nĂşmeros desejados. NĂŁo esqueça de colocar seu nome e endereço completos e nĂłs remeteremos a(s) revista(s) pelo correio. Pedidos podem ser feitos tambĂŠm por e-mail e comprovantes de depĂłsito poderĂŁo ser enviados pelo fax. Se tiver qualquer dĂşvida entre em contato conosco. Telefone: 021-5295077 / Fax: 021-5295023 e-mail: obm@impa.br Home-Page: http//www.obm.org.br/
Ă?kĂ&#x17E; Ă&#x;ªà âåCĂŁÂ&#x201D;äÌü ãèç ĂŠ ĂŞĂŹĂŤ Ă ĂŽ ĂŻ Ă°Â&#x2013;ĂąZà òQĂłoĂ´ Ăľ Ăś ádøCĂš!Ăş ݚú ĂźĂ&#x2030;ýšÞ Ăż QúšáÂ&#x201E;Ăż Â&#x2013;Ăś CĂş TĂş Ăź ݪÿ ^Ăş :Ăż á TĂś QĂş
!" #$ % & '( # ) *,+ ' + # - . %/0 1 '2 32 ' + 4# 57698 :<;>=9?>@ ?BA C.? D 8E6 FG8 C2?H@ FBIB?>J2F :<K>J 8 L ? M NPO Q.R2STS UWV X O Q Y Q XTV X,O X Z>V X,[
-
EUREKA! N°4, 1999
59
Sociedade Brasileira de MatemĂĄtica
ÂĽ]\ Â?
Ă&#x201C;
Â&#x17E; ÂĽ
<¤ Â
Ă?-^`_
¿)ž
ÂĽ
a b Â&#x201A; c# { } '0d "fe%"ge)Â&#x2030;G} "){9c
8 de maio (sĂĄbado) â&#x2122;Ś h,i b Â&#x201A;,c# { } '0d "fe)"gejclkfc x Â&#x2030;G 0m)
17 a 24 de maio Argentina â&#x2122;Ś n,h b Â&#x201A;,c# { } '0d "fe)"LÂ&#x20AC;fo%") { Â&#x2030; {9o)"ge%Â&#x2030;M} "fp Â&#x2030; }rqfp {9k)"
Primeira Fase â&#x20AC;&#x201C; 12 de junho (sĂĄbado) Segunda Fase â&#x20AC;&#x201C; 28 de agosto (sĂĄbado) Terceira Fase â&#x20AC;&#x201C; 23 de outubro (sĂĄbado) e 24 de outubro (domingo) â&#x2122;Ś s,i b Â&#x201A;,c# { } '0d "fe)"L{ x p Â&#x2030;0o x "fk){9c x ") te)Â&#x2030;G} "jp Â&#x2030; }Tqfp {9k)"
10 a 22 de julho Bucharest, RomĂŞnia. â&#x2122;Ś h,s b Â&#x201A;,c# {4} '0d "fe)"L{ Â&#x20AC;)Â&#x2030;0ofc#"%} Â&#x2030;0o){9k)" x "ge)Â&#x2030;G} "fp Â&#x2030; }rqjp {9k)"
setembro Cuba. uwv x y z {}| ~Â&#x20AC;{0 Â&#x201A;Â&#x201E;Â&#x192;&Â&#x2026;Â&#x2021;Â&#x2020; Â&#x2C6; Â&#x2C6; Â&#x2030;TÂ&#x160;9Â&#x2039; Â&#x152;<Â? Â&#x17D;Â&#x20AC;Â?,Â?$Â? Â&#x2018;GÂ&#x2019; Â?,Â&#x201C; Â&#x2039;Â&#x20AC;Â&#x160; Â&#x201D;$Â&#x2039;Â&#x20AC;Â&#x2019; Â?Â&#x2022;Â?$Â&#x201D; Â&#x2013;fÂ&#x2014;.Â&#x2DC;&Â&#x2122; Â&#x161; Â&#x203A;>Â&#x161; Â&#x153;Â&#x17E;Â? Â&#x;  ¥ ¢.ÂĄ ÂŁ ¤¼£ ÂĄ ÂŚ §9¨ Š ÂŚ ÂŁ.ÂŞÂŹÂŤ ÂĄ4ÂĄ ¯Ž¹° ² Âł ´%Âľ ÂŁ2Âś ¡¸¨ š´¼¥$²&Âş.ÂŁ ÂĄ Âť Âź ½ ž&Âż Ă&#x20AC;Ă&#x201A;Ă ÂżĂ&#x201E;Ă&#x192; ž&Ă&#x20AC;,Ă&#x2026; ž.ÂżrĂ&#x2020;>½ ž Ă Âť Ă&#x2021;Ă&#x2C6;Ă&#x192; ž Ă&#x2030; Âť$Âź Ă&#x192;7Ă&#x2021;šĂ&#x160; Ă }Ă&#x2026; ÂżĂ&#x201E;Ă $Ă&#x192;7Ă&#x2030; Âż Ă ÂżĂ&#x201E;ÂżÂ&#x201E;Ă&#x2039; Ă&#x152;2Âť>Ă&#x20AC;,Ă&#x2026;EÂź Ă? Ă&#x17D;7Ă? Ă? Ă&#x2018;Ă&#x201A;Ă&#x2019; Ă&#x2018;>Ă&#x201C; Ă&#x201D; Ă? Ă&#x2022;2Ă&#x2019; Ă&#x201C; Ă? Ă&#x2013;šĂ&#x2019; Ă&#x2014; Ă&#x2DC;2Ă&#x2122; Ă&#x2018;TĂ&#x161; Ă&#x2122; Ă&#x203A; Ă&#x2019;$Ă&#x2013;Ă&#x153;Ă&#x2018;&Ă&#x2019; Ă&#x2022;<Ă&#x2122; Ă?,Ă&#x2DC; Ă? Ă&#x203A; Ă&#x2122; Ă&#x2018;HĂ? Ă&#x2DC;.Ă&#x2022;2Ă?>Ă&#x17E;&Ă&#x;,Ă&#x2018;Ă&#x201E;Ă&#x203A; Ă&#x2122;TĂ&#x2018; Ă&#x2019; Ă ,ĂĄ Ă? Ă&#x2014; Ă&#x2DC;2Ă&#x2019; â ĂŁ ä â ĂĽ2â ĂŚ ç$è ĂŠ ĂŞ2ĂŤ ĂŹ ĂŤ ĂŻ ĂŤÂ&#x201E;Ă°,Ăą ò Ăł Ă´ ò.ĂąHĂľ0Ăą ĂŞ&Ăś,Ăł ĂąĂ&#x201E;á$Ăą ø Ăą$Ăš9Ăš Ăş Ă ĂŽ Ăť$Ăź Ăź Ă˝ Ăž Ăż Ăž Ăź Ă˝<Ăž
! " #%$'&)(+* ,.-+/ 0 1 2 3.4 57698;:=<+> ?)?.8;< 5@- <9> ?)?)A)A ? B CEDGF HJILK M F N F I I B O@P B QSRUT V Q
EUREKA! N°4, 1999
60
Sociedade Brasileira de Matemática
¾ÉÏLÏXWZY@[[Ó]\^Y ÏXWZ[[Ñ_WZ[r\L¿ ÏLÓ]\L¿OÑ Amarisio da Silva Araújo Alberto Hassen Raad Antônio C. Rodrigues Monteiro Angela Camargo Ariosto de Oliveira Lima Benedito T. Vasconcelos Freire Claudio Arconcher Egnilson Miranda de Moura Élio Mega Florêncio F. Guimarães F. Francisco Dutenhefner Gisele de A. Prateado G. Ivanilde H. Fernandes Saad João B. de Melo Neto João F. Melo Libonati Jorge Ferreira José Carlos Pinto Leivas José Luis Rosas Pinho José Paulo Carneiro José Vieira Alves Leonardo Matteo D'orio Licio Hernandes Bezerra Luzinalva M. de Amorim Marco Polo Marcondes Cavalcante França Mario Jorge Dias Carneiro Pablo Rodrigo Ganassim Paulo H. Cruz Neiva de L. Jr. Reinaldo Gen Ichiro Arakaki Ricardo Amorim Roberto Vizeu Barros Sergio Claudio Ramos Seme Gebara Neto Tadeu Ferreira Gomes Valdenberg Araújo da Silva Wagner Pereira Lopes Waldemar M. Canalli
(UFV) Viçosa - MG (UFJF) Juiz de Fora - MG (UFPE) Recife - PE (Centro de Educação de Adultos CEA) Blumenau - SC (UFPI) Parnaíba - PI (UFRN) Natal - RN (Col. Leonardo da Vinci) Jundiaí - SP (Col. Agrícola do Bom Jesus) Bom Jesus - PI (Col. ETAPA) São Paulo - SP (UFES) Vitória - ES (UFMG) Belo Horizonte - MG (UFGO) Goiânia - GO (U. Católica Dom Bosco) Campo Grande - MS (UFPI) Teresina - PI (Grupo Educ. IDEAL) Belém - PA (UEM) Maringá - PR (URG) Rio Grande - RS (UFSC) Florianópolis - SC (USU) Rio de Janeiro - RJ (UFPB) Campina Grande - PB (Parque de Material Aeronáutico de Belém) Belém - PA (UFSC) Florianópolis - SC (UFBA) Salvador - BA (Colégio Singular) Santo André - SP (UF Ceará) Fortaleza - CE (UFMG) Belo Horizonte - MG (L. Albert Einstein) Piracicaba - SP (Esc. Tec.Everardo Passos) SJ dos Campos - SP (INPE) SJ dos Campos - SP (Centro Educ. Logos) Nova Iguaçu - RJ (Colégio ACAE) Volta Redonda - RJ (IM-UFRGS) Porto Alegre - RS (UFMG) Belo Horizonte - MG (U. do Estado da Bahia) Juazeiro - BA (U. Federal de Sergipe) São Cristovão - SE (Esc. Tec. Fed. de Goiás) Jataí - GO (P.M. S. João de Meriti) S. João de Meriti - RJ
EUREKA! N°4, 1999
61
Sociedade Brasileira de Matemática
EUREKA! N°4, 1999
62
CONTEÚDO AOS LEITORES
2
5a. OLIMPÍADA DE MAIO Problemas
3
5a. OLIMPÍADA DE MAIO Resultados
5
10a. OLIMPÍADA DE MATEMÁTICA DO CONE SUL Problemas e Soluções
7
40a. OLIMPÍADA INTERNACIONAL DE MATEMÁTICA Problemas e Resultados
15
ARTIGOS ADEDANHA OU "DE COMO OS DEUSES MATEMÁTICOS TROUXERAM A PAZ AO MUNDO" Pablo Emanuel
17
QUADRILÁTEROS E TRIÂNGULOS Marcelo Mendes
23
O PRINCÍPIO DAS GAVETAS Paulo Cezar Pinto Carvalho
27
DESIGUALDADES ELEMENTARES Antonio Caminha Muniz Neto
34
40a. OLIMPÍADA INTERNACIONAL E 14a. OLIMPÍADA IBEROAMERICANA DE MATEMÁTICA Primeiro teste de Seleção
50
SOLUÇÕES DE PROBLEMAS PROPOSTOS
51
PROBLEMAS PROPOSTOS
57
COMO ASSINAR A EUREKA!
59
AGENDA OLÍMPICA
60
COORDENADORES REGIONAIS
61
Sociedade Brasileira de Matemática
AOS LEITORES Realizamos a primeira fase da XXI Olimpíada Brasileira de Matemática em mais de 3.000 colégios do nosso país. Através dos relatórios enviados pelas escolas aos Coordenadores Regionais, estabelecemos as notas de corte para a promoção dos alunos à segunda fase que se realizará em agosto. Notas de corte para promoção a segunda fase: Primeiro nível (5a. e 6a. séries) : Segundo nível (7a. e 8a. séries) : Terceiro nível (Ensino médio) :
09 acertos ou mais. 10 acertos ou mais. 12 acertos ou mais.
A prova da segunda fase será discursiva e ainda será aplicada nos colégios. Os enunciados, as soluções e os criterios de pontuação serão enviados a todas as escolas participantes. A Comissão de Olimpíadas agradece a participação de alunos e professores na primeira fase e deseja sucesso a todos nas fases seguintes. Agradecemos às pessoas que colaboraram com este número propondo problemas e enviando soluções de problemas propostos. Aproveitamos para continuar estimulando nossos leitores a contribuir para nossa revista com artigos, problemas e soluções. Lembramos da existência da lista de discussão de problemas de matemática da OBM, cujo endereço eletrônico mudou para: obm-l@mat.puc-rio.br . Para maiores informações escreva para obm@impa.br ou para o Prof. Nicolau Saldanha, administrador desta lista em nicolau@mat.pucrio.br .
Comitê Editorial.
EUREKA! N°5, 1999
2
Sociedade Brasileira de Matemática
5a. OLIMPÍADA DE MAIO Primeiro Nível PROBLEMA 1
São escolhidos 2 números inteiros entre 1 e 100 inclusive, tais que a diferença é 7 e o produto é múltiplo de 5. De quantas maneiras pode ser feita a escolha? PROBLEMA 2
Num paralelogramo ABCD, BD é a diagonal maior. Ao fazer coincidir B com D, mediante uma dobra, se forma um pentágono regular. Calcular as medidas dos ângulos que a diagonal BD forma com cada um dos lados do paralelogramo. PROBLEMA 3
Em cada um dos 10 degraus de uma escada existe uma rã. Cada rã pode, de um pulo, colocar-se em outro degrau, mas quando uma rã faz isso, ao mesmo tempo, uma outra rã pula a mesma quantidade de degraus em sentido contrário: uma sobe e outra desce. Conseguirão as rãs colocar-se todas juntas num mesmo degrau? PROBLEMA 4 2
1
3
Dez cartões quadrados de 3 centímetros de lado são cortados por uma línha, como mostra a figura. Depois dos cortes tem-se 20 peças: 10 triângulos e 10 trapézios. Forme um quadrado que utilize as 20 peças sem superposições nem espaços.
PROBLEMA 5
Ana, Beatriz, Carlos, Diego e Emilia jogam um torneio de xadrez. Cada jogador enfrenta uma vez só cada um dos outros quatro jogadores. Cada jogador consegue 2 pontos se ganha a partida, 1 ponto se empata e 0 pontos se perde a partida. Ao finalizar o torneio, as pontuações dos 5 jogadores são todas diferentes. Encontre o máximo número de empates que pode ter tido o torneio e justifique por que não pode ter havido um número maior de empates.
EUREKA! N°5, 1999
3
Sociedade Brasileira de Matemática
SEGUNDO NÍVEL PROBLEMA 1
Um número natural de três algarismos é chamado de tricúbico se é igual à soma dos cubos dos seus dígitos. Encontre todos os pares de números consecutivos tais que ambos sejam tricúbicos. PROBLEMA 2
A figura representa a quarta parte de um círculo de raio 1. No arco AB, se consideram pontos P e Q de forma tal que a reta PQ seja paralela à reta AB. Sejam X e Y os pontos de interseção da reta PQ com as _____2
B
____ 2
retas OA e OB respectivamente. Calcular PX + PY .
O
A
PROBLEMA 3
A primeira fileira da tabela ao lado é preenchida com os números de 1 a 10, em ordem crescente. A segunda fileira é preenchida com os números de 1 a 10, em qualquer ordem. Em cada casa da terceira fileira se escreve a soma dos dois números escritos nas casas acima. Existe alguma maneira de preencher a segunda fileira de modo que os algarismos das unidades dos números da terceira fileira sejam todos distintos? PROBLEMA 4
Seja ABC um triângulo equilátero. M é o ponto médio do segmento AB e N é o ponto médio do segmento BC. Seja P o ponto exterior a ABC tal que o triângulo ACP é isósceles e retângulo em P. PM e AN cortam-se em I. Prove que CI é a bissetriz do ângulo MCA. PROBLEMA 5
São dados 12 pontos que são os vértices de um polígono regular de 12 lados. Rafael deve traçar segmentos que tenham seus dois extremos em dois dos pontos desenhados. É permitido que cada ponto seja extremo de mais de um segmento e que os segmentos se cruzem, mas é proibido traçar três segmentos que sejam os três lados de um triângulo em que cada vértice é um dos 12 pontos iniciais. Encontre o número máximo de segmentos que pode traçar Rafael e justifique por que não é possivel traçar um número maior de segmentos.
EUREKA! N°5, 1999
4
Sociedade Brasileira de Matemática
5a. OLIMPÍADA DE MAIO Resultado Brasileiro Primeiro Nível Fábio Dias Moreira André de Carvalho Amaro Diego Costa de Almeida Daniel Haanwinckel Junqueira Raúl M. Alexandrino Nogueira Bruna Griguol Felipe Oliveira de Sousa Guilherme Finkelfarb Lichand Leonardo Luis Desideri Freitas Zilma K. Barbosa Bezerra Thiago Augusto Caldas Bello Mateus Gomes Filgueiras
Ouro Prata Prata Bronze Bronze Bronze Bronze Menção Honrosa Menção Honrosa Menção Honrosa Menção Honrosa Menção Honrosa
Rio de Janeiro - RJ São Paulo - SP Fortaleza - CE Salvador - BA Fortaleza - CE Cafelândia - PR Fortaleza - CE São Paulo - SP Vitória - ES Fortaleza - CE Salvador - BA Fortaleza - CE
Segundo Nível João Alfredo Castellani F. Freire Arthur Duarte Nehmi Davi M. Alexandrino Nogueira Luiz Brizeno Firmeza Neto Luciana Andretta do Nascimento Thiago Barros Rodrigues Costa Daniel Pinheiro Sobreira Maurício Massão Soares Matsumoto Fabio S. Toniolo Einstein do Nascimento Jr. Hugo Pinto Iwata Rodrigo Roque Dias
Ouro Prata Prata Bronze Bronze Bronze Bronze Menção Honrosa Menção Honrosa Menção Honrosa Menção Honrosa Menção Honrosa
Salvador - BA São Paulo - SP Fortaleza - CE Fortaleza - CE Cafelândia - PR Fortaleza - CE Fortaleza - CE São Paulo - SP São Paulo - SP Fortaleza - CE S J do Rio Preto - SP São Paulo - SP
O Brasil teve uma excelente participação na 5a. Olimpíada de Maio na qual participaram 13 paises, sendo o país com maior pontuação nos dois níveis em que é realizada esta competição.
EUREKA! N°5, 1999
5
Sociedade Brasileira de Matemática
5a. OLIMPÍADA DE MAIO Clasificação por paises
PRIMEIRO NÍVEL País Brasil Argentina Espanha México Peru Colombia Cuba Costa Rica Uruguai Chile Bolívia Venezuela Equador
SEGUNDO NÍVEL Pontuação 286 272 206 168 157 154 152 143 142 121 118 95 55
País Brasil Peru Argentina Cuba Colombia México Espanha Uruguai Chile Bolívia Costa Rica Venezuela Equador
Pontuação 405 341 264 219 188 175 136 125 123 108 57 57 22
!!!!
Você sabia…
Que o maior número primo
6972593
–1, que tem 2.098.960 dígitos e conhecido é 2 foi descoberto em 1/6/99 por Nayan Hafratwala, um participante do GIMPS, um projeto cooperativo para procurar primos de mersenne? Consulte na Internet a página Http://www.mersenne.org/prime.htm
EUREKA! N°5, 1999
6
Sociedade Brasileira de Matemática
10a. OLIMPÍADA DE MATEMÁTICA DO CONE SUL Problemas e Soluções A 10a. Olimpíada de Matemática do Cone Sul foi realizada na cidade de Córdoba, Argentina no período de 17 a 24 de maio de 1999. Dela participaram alunos de até 15 anos dos seguintes países: Argentina, Brasil, Bolívia, Chile, Paraguai, Peru e Uruguai. A equipe brasileira foi selecionada através de provas realizadas em março e maio deste ano e foi liderada pelos professores Florêncio Ferreira Guimarães Filho da UFES, e Antônio Caminha Muniz Neto, da UFCE. O Resultado da Equipe Brasileira BRA 1 BRA 2 BRA 3 BRA 4
Daniel Massaki Yamamoto Daniel Pinheiro Sobreira Fabrício Siqueira Benevides Humberto Silva Naves
BRONZE BRONZE PRATA PRATA
Primeiro Dia Duração da prova: 4 horas PROBLEMA 1
Achar o menor inteiro positivo n tal que as 73 frações
19 20 21 91 , , ,........., n + 21 n + 22 n + 23 n + 93 sejam todas irredutíveis. SOLUÇÃO
a a é irredutível se e só se é irredutível ( se a e b tem um fator b b−a comum, então a e b – a têm um fator comum, e reciprocamente). O problema se transforma em achar o menor valor de n tais que as frações 19 20 91 ,..., , n+2 n+2 n+2 sejam todas irredutíveis.
A fração
EUREKA! N°5, 1999
7
Sociedade Brasileira de Matemática
Se n + 2 é primo, maior que 91, todas as frações são irredutíveis. Assim, um valor possível de n é 95. Verifiquemos que é o menor possível. 20 Se n + 2 < 97 e n + 2 é par (n é par) há frações redutíveis, por exemplo . n+2 Se 19 ≤ n + 2 ≤ 91, obviamente há uma fração redutível. Se n + 2 < 19, então n + 2 tem um múltiplo entre 19 e 91, e portanto, há uma fração redutível. 31 é redutível. Se n + 2 = 93 = 3 . 31, então n+2 19 Se n + 2 = 95 = 5 . 19, então é redutível. n+2 Então, o valor mínimo de n + 2 é 97, que corresponde a n = 95. PROBLEMA 2
Seja ABC um triângulo retângulo em A. Construir o ponto P sobre a hipotenusa BC, tal que se Q for o pé da perpendicular traçada desde P ao cateto AC, então a área do quadrado de lado PQ é igual à área do retângulo de lados iguais a PB e PC. Mostrar os passos da construção. SOLUÇÃO B H P
A
C
Q
____ 2
____
____ ____
P ∈ BC satisfaz as condições do enunciado se e só se PQ = PB ⋅ PC (*). Como ____
∆PQC ≅ ∆BAC , vem que
PQ ____
PC ____
PC
____
PB
=
____ 2
BC
____
___
=
____
AB
____
___
AB
. Daí, PQ = PC ⋅ ____ , e segue de (*) que BC BC ____
____
____ ____
____
. Sendo H o pé da altura relativa a BC , temos BH ⋅ BC = AB 2 , donde
AB 2
EUREKA! N°5, 1999
8
Sociedade Brasileira de Matemática ____
PC
____
PB
____
=
BC
____
.
BH
Temos então a seguinte construção: B' B H'
P H
A
C r ↔
i)
Traçe, por C, a reta r tal que r r⊥ AC .
ii)
Marque em r o ponto B' tal que B' C = BC e B, B' estejam num mesmo
iii) iv) v)
semi-plano dos determinados por BC . Trace a altura AH relativa à hipotenusa BC. Marque H' ∈ AB tal que BH' = BH. BC ∩ B'H' = {P}
____
____
↔
PROBLEMA 3
Há 1999 bolinhas em uma reta; algumas são vermelhas e as demais azuis (poderiam ser todas vermelhas ou todas azuis). Debaixo de cada bolinha escrevemos o número igual à soma da quantidade de bolinhas vermelhas à direita dela mais a quantidade de bolinhas azuis à esquerda dela. Se, na sequência de números assim obtida, houver exatamente três números que aparecem uma quantidade ímpar de vezes, quais podem ser estes três números? SOLUÇÃO
Se as 1999 bolinhas são de uma mesma cor, a sucessão de números é crescente ou decrescente. Cada número aparece uma vez só e há 1999 (portanto, não há EUREKA! N°5, 1999
9
Sociedade Brasileira de Matemática
exatamente 3 números que se repetem um número ímpar de vezes (1 é ímpar). Logo, há bolinhas das duas cores. Dada uma distribuição das bolinhas que tem em certa posição uma bolinha azul A e na posição seguinte uma bolinha vermelha R, se há a bolinhas azuis à esquerda de A e r bolinhas vermelhas à sua direita, então há a + 1 bolinhas azuis à esquerda de R e r – 1 bolinhas vermelhas à sua direita. O número escrito embaixo de A é n = a + r e o número escrito embaixo de R é a + 1 + r – 1 = n. Se trocamos de lugar A e R, e não mexemos em nenhuma outra bolinha, na nova distribuição há a bolinhas azuis à esquerda de R e r – 1 bolinhas vermelhas à sua direita, enquanto que à esquerda de A há a bolinhas azuis e, à sua direita, r – 1 bolinhas vermelhas. Os números escritos embaixo de R e A são a + r – 1= n – 1 e a + r – 1 = n – 1. Os números escritos embaixo das outras bolinhas não mudam. Então, depois da troca, o número n se repete duas vezes menos e o número n – 1 se repete duas vezes mais. Os números que se repetem uma quantidade ímpar de vezes serão os mesmos em ambas configurações. Portanto, basta estudar a configuração na qual todas as bolinhas vermelhas são consecutivas, a partir da primeira, e todas as azuis são consecutivas, a partir da última vermelha. Sejam α, β, as quantidades de bolinhas vermelhas e azuis, respectivamente; então α + β = 1999. Embaixo da primeira bolinha (é vermelha) está o número α – 1, na seguinte, α – 2, depois α – 3, e assim por diante, até ter 0 na última bolinha vermelha (na posição α). Então, embaixo da primeira bolinha azul há 0, na segunda 1 e assim por diante, até a última, que tem β – 1 embaixo. Se α < β, os números 0, 1, 2, …, α – 1 aparecem duas vezes (quantidade par) e os números α, α + 1, α + 2, …, β – 1 aparecem uma vez (quantidade ímpar). Se há exatamente 3 números que aparecem uma quantidade ímpar de vezes, estes são α, α + 1 e α + 2 = β – 1. Portanto, α + β = 2α + 3, donde α = 998, e os três números que se repetem uma quantidade ímpar de vezes são 998, 999 e 1000. Se α > β, os três números que aparecem uma quantidade ímpar de vezes são β, β +1 e β + 2 = α – 1, donde α + β = 2β + 3 e os tres números são, novamente, 998, 999 e 1000.
Segundo Dia Duração da prova 4 horas. PROBLEMA 4
Seja A um número de seis algarismos, três dos quais estão coloridos e são iguais a 1, 2 e 4. EUREKA! N°5, 1999
10
Sociedade Brasileira de Matemática
Demonstrar que é sempre possível obter um número que é múltiplo de 7, efetuando uma só das seguintes operações: ou suprimir os três algarismos coloridos, ou escrever todos os algarismos de A em alguma ordem. SOLUÇÃO
Provaremos o seguinte resultado mais geral: Seja A um número de mais de 3 algarismos, três dos quais são 1, 2, 4. Prove que é sempre possível permutarmos os algarismos de A de modo que o número resultante seja um múltiplo de 7. Prova: Seja B = (ak…a1)10, k ≥ 1, o número obtido a partir de A ao suprimirmos uma ocorrência de cada um dos algarismos 1, 2, 4 e C o número que queremos obter a partir de A. i) ii)
B = 7 : tome C = 2471 B = 7 ...... ... 7 : tome C = 7 72471
. Analogamente tratamos o caso em que k >1
iii)
iv)
k −1
só há algarismos 0 e 7 em B. Suponhamos, de agora em diante, que nem todos os algarismos de B sejam iguais a 7 ou zero. B não é equivalente a 0 (mod 7): Como {0, 124, 142, 214, 241, 412, 421} é um sistema completo de restos, módulo 7, (isto é, esses números, quando divididos por 7 deixam todos os restos possíveis: 0, 1, 2, 3, 4, 5 e 6) obtemos C justapondo, à direita de B, uma permutação conveniente de 124. B ≡ 0 (mod. 7): Se a1 ≠ 7,0 então B' = (ak…a2 0 a1)10 não é múltiplo de 7, pois 10B –B' = 9a1. Como {0, 1024, 1042, 2014, 2041, 4012, 4021} também é um sistema completo de restos, módulo 7, obtemos C como em (iii) (isto é, somando 100B' a um dos seis números 1024,…,4021).
PROBLEMA 5
É dado um quadrado de lado 1. Demonstrar que, para cada conjunto finito de pontos no bordo do quadrado, é possível achar um vértice do quadrado com a seguinte propriedade: a média aritmética dos quadrados das distâncias de tal 3 . vértice aos pontos do conjunto é maior ou igual a 4
EUREKA! N°5, 1999
11
Sociedade Brasileira de Matemática
SOLUÇÃO
Sejam A1A2A3A4 o quadrado e X1, X2, …, Xn os pontos do perímetro. Devemos _____ 2
_____ 2
_____ 2
provar que uma das quatro somas Ai X 1 + Ai X 2 + ... + Ai X n , i = 1, 2, 3, 4, é maior 3n ou igual que . 4 Somamos entre si estas quatro somas _____ 2
_____ 2
_____ 2
______ 2
______ 2
_____ 2
_____ 2
______ 2
______ 2
______ 2
_____ 2
_____ 2
_____ 2
_____ 2
_____ 2
_____ 2
( A1 X 1 + A1 X 2 + ... + A1 X n ) + ( A2 X 1 + A2 X 2 + ... + A2 X n ) + ______ 2
_____ 2
_____ 2
_____ 2
( A3 X 1 + A3 X 2 + ... + A3 X n ) + ( A4 X 1 + A4 X 2 + ... + A4 X n ) e reagrupamos os somandos em n grupos, um por cada ponto Xi _____ 2
_____ 2
_____ 2
______ 2
_____ 2
_____ 2
( A1 X 1 + A2 X 1 + A3 X 1 + A4 X 1 ) + ( A1 X 2 + A2 X 2 + A3 X 2 + A4 X 2 ) + ... + _____ 2
( A1 X n + A2 X n + A3 X n + A4 X n ). Demostraremos que, se X é um ponto do perímetro do quadrado, então _____ 2
( A1 X + A2 X + A3 X + A4 X ) ≥ 3. Sejam x e 1 – x as distâncias X aos extremos do lado ao que pertencem. Então as distâncias de X aos outros dois vértices do quadrado são, em alguma ordem,
1 + x 2 e 1 + (1 − x) 2 , e temos _____2
_____2
______2
______2
A1 X + A2 X + A3 X + A4 X
= x 2 + (1 − x) 2 + (1 + x 2 ) + (1 + (1 − x) 2 ) = 4( x 2 − x +1).
Devemos provar que, para todo x ∈ [0,1] ,
4( x 2 − x + 1) ≥ 3, mas isto equivale a
1 3 3 ( x − ) 2 + ≥ , que é claramente 2 4 4
verdadeira. Temos assim que _____ 2
_____ 2
_____ 2
_____ 2
_____ 2
_____ 2
( A1 X 1 + A1 X 2 + ... + A1 X n ) + ( A2 X 1 + A2 X 2 + ... + A2 X n ) + _____ 2
______ 2
_____ 2
____ 2
_____ 2
_____ 2
+ ( A3 X 1 + A3 X 2 + ... + A3 X n ) + ( A4 X 1 + A4 X 2 + ... + A4 X n ) ≥ 3n, 3n portanto, um dos quatro somandos é maior ou igual que , c.q.d. 4
EUREKA! N°5, 1999
12
Sociedade Brasileira de Matemática
PROBLEMA 6
Uma formiga caminha pelo piso de um pátio circular de raio r e avança em linha reta, mas às vezes se detém. Cada vez que se detém, antes de continuar a caminhar, gira 60o alternando o sentido (se da última vez ela girou 60o para a direita da próxima vez gira 60o para a esquerda, e vice-versa). Achar o maior comprimento possível do caminho percorrido pela formiga. Demonstrar que o comprimento assim obtido é efetivamente, o máximo possível.
60o
Giro de 60o à direita. SOLUÇÃO
Podemos supor que a formiga só se detém uma vez, pois, caso se detenha mais vezes podemos substituir seu caminho por outro de mesmo comprimento onde ela só se detém uma vez, como na figura abaixo
Sejam A e B respectivamente os pontos inicial e final do caminho percorrido pela formiga. Se AB C' não é diâmetro, traçamos o diâmetro A'B' // AB. Se C' é tal que A'C'B' ≅ ACB, temos C' dentro do B círculo (pois A'C'B' = 120°) e A'C' + C'B' = B' A O A' B' A' ( AC + CB ) > AC + CB. AB Logo, para o caminho ser máximo AB deve ser diâmetro, ou seja, AB = 2r Temos agora que maximizar AC + CB sobre todos os triângulos ACB. tais que C
∧
que ACB = 120° e AB = 2r. Seguem duas demonstrações de que AC + CB é máxima quando AC = CB:
EUREKA! N°5, 1999
13
Sociedade Brasileira de Matemática
1a. demonstração
a 2 + b 2 + ab = 4r 2 ⇒ (a + b) 2 − ab = 4r 2 ⇒ C
( a + b) 2 −
a
120°
b B
a+b≤
2r
A
4r 3
( a + b) 2 ≤ (a + b) 2 − ab = 4r 2 ⇒ 4 , com igualdade se e só se a = b.
2a. demonstração Seja C ' ' sobre AC tal que C ' ' C = CB e C ' ' ≠ A. Temos AC ' ' = AC + CB e AC ' ' B = 60°.
C''
C 120°
A
B
C'' C ≡ O'
A
Então devemos determinar a maior corda AC'', com C'' sobre o arco capaz do ângulo 60° sobre AB. Se O' é o centro da circunferência do arco capaz, a corda é o diâmetro por O', Daí, C ≡ O' e AC = CC'' = CB
B
Assim, a resposta do problema é
4r 3
.
EUREKA! N°5, 1999
14
Sociedade Brasileira de Matemática
40a. OLIMPÍADA INTERNACIONAL DE MATEMÁTICA Problemas e Resultados A 40a. Olimpíada Internacional de Matemática foi realizada na cidade de Bucharest, Romênia, no período de 10 a 22 de julho de 1999. A equipe brasileira foi liderada pelos professores Nicolau Corção Saldanha, da PUC - Rio, e Carlos Gustavo T. de A. Moreira, do IMPA.
O Resultado da Equipe Brasileira BRA 1 BRA 2 BRA 3 BRA 4 BRA 5 BRA 6
Fabrício Siqueira Benevides Pedro Paulo de Simoni Gouveia Daniel Massaki Yamamoto Sérgio Tadao Martins Daniel Nobuo Uno Humberto Silva Naves
13 pontos 12 pontos 08 pontos 14 pontos 11 pontos 17 pontos
(Medalha de Bronze) (Medalha de Bronze) (Medalha de Bronze) (Medalha de Bronze)
A prova deste ano foi considerada difícil. Pela primeira vez em muitos anos nenhum participante obteve a pontuação máxima (42 pontos). A maior pontuação obtida este ano foi de 39 pontos.
Primeiro Dia Duração da prova: 4 horas e 30 minutos PROBLEMA 1
Determine todos os conjuntos finitos S de pontos do plano com pelo menos três elementos que satisfazem a seguinte condição: Para quaisquer dois pontos distintos A e B de S, a mediatriz do segmento AB é um eixo de simetria de S. PROBLEMA 2
Seja n ≥ 2 um inteiro fixo. a) Determinar a menor constante C para a qual a desigualdade
∑
x i x j ( xi2 + x 2j ) ≤ C (
1≤ i < j ≤ n
b)
∑x ) i
4
1≤i ≤ n
é válida para quaisquer números reais x1, …, xn ≥ 0. Para esta constante C, determine quando ocorre a igualdade.
EUREKA! N°5, 1999
15
Sociedade Brasileira de Matemática
PROBLEMA 3
Considere um tabuleiro quadrado n × n, onde n é um inteiro positivo par fixo. O tabuleiro está dividido em n2 quadrados unitários. Dizemos que dois quadrados distintos do tabuleiro são adjacentes se eles têm um lado comum. Marcam-se N quadrados unitários do tabuleiro de tal forma que qualquer quadrado (marcado ou não) é adjacente a pelo menos um quadrado marcado. Determine o menor valor possível para N.
Segundo Dia Duração da prova: 4 horas e 30 minutos PROBLEMA 4
Determine todos os pares (n, p) de inteiros estritamente positivos tais que
• p é primo, • n ≤ 2p, e • (p – 1)n + 1 é divisível por np–1. PROBLEMA 5
Duas circunferências Γ1 e Γ2 estão contidas no interior de uma circunferência Γ e são tangentes a Γ em pontos distintos M e N, respectivamente. A circunferência Γ1 passa pelo centro de Γ2 . A reta que passa pelos dois pontos de interseção de Γ1 e Γ2 intersecta Γ em A e B. As retas MA e MB intersectam Γ1 respectivamente em C e D. Prove que CD é tangente a Γ2 . PROBLEMA 6
Determine todas as funções f : R → R tais que
f ( x − f ( y )) = f ( f ( y )) + x f ( y ) + f ( x) − 1 para quaisquer x, y ∈ R.
EUREKA! N°5, 1999
16
Sociedade Brasileira de Matemática
ADEDANHA OU “DE COMO OS DEUSES MATEMÁTICOS TROUXERAM A PAZ AO MUNDO” ♦ Nível Iniciante
Pablo Emanuel - IMPA
Diz a lenda que, há muitos milênios, o mundo vivia em guerra constante, pois as pessoas não sabiam como resolver as suas discordâncias, a não ser pela força bruta. Um dia, os deuses (que são exímios matemáticos), para resolver esta situação, enviaram um mensageiro à Terra, com a missão de ensinar os homens a resolverem as suas disputas. O anjo se dirigiu então aos homens, dizendo: - Quando dois entre vós precisarem chegar a um acordo, que se faça como vos digo: que um escolha par e o outro escolha ímpar, então que ambos mostrem ao mesmo tempo a mão exibindo uma certa quantidade de dedos. Serão então somadas estas quantidades. Se a soma for um número par declara-se vencedor o jogador que escolheu par e, caso contrário, declara-se vencedor aquele que escolheu ímpar. Os homens ficaram maravilhados com a sabedoria dos deuses e, deste dia em diante, houve um grande período de paz, pois todas as questões eram resolvidas com o jogo que eles haviam aprendido dos deuses. Um dia, porém, esta paz foi abalada. Três reis disputavam um pedaço de terra, que ficava exatamente na divisa entre os três países. Eles estavam prontos a utilizar o jogo divino do par-ou-ímpar, mas o rei que sabia mais matemática entre os três se levantou e disse: - Caros colegas, nós todos sabemos que um número só pode ser par ou ímpar, não existindo uma terceira opção. Como somos três, algum de nós não vai ter opção alguma. Este era realmente um problema muito sério. Para resolvê-lo, foi chamado o melhor matemático da Terra na época, chamado Zerinhoum. Ele pensou durante várias semanas em como resolver o problema dos reis, e finalmente chegou a uma solução: - Majestades, encontrei a solução para o vosso problema. Ao mesmo tempo, vós estendereis vossas mãos, mantendo-as ou com a palma para cima ou com a palma para baixo. Aquele dentre vós que tiver a mão em posição diferente dos demais ganha a disputa.
EUREKA! N°5, 1999
17
Sociedade Brasileira de Matemática
- E se todos nós tivermos as palmas das mãos viradas para o mesmo lado? -indagaram os reis. - Neste caso, majestades, vós jogareis novamente, até que algum entre vós vença. Como a disputa era muito urgente, os reis aceitaram a sugestão do eminente matemático. Houve mais um período de paz, desta vez muito mais curto. Em pouco tempo, as pessoas perceberam que o jogo de Zerinhoum podia se alongar indefinidamente, e que era possível se fazer alianças para prejudicar adversários políticos. Então as pessoas rezaram aos deuses, pedindo um novo jogo, que trouxesse de novo a paz à Terra. Os deuses então enviaram novamente um mensageiro. Quando ele chegou, os homens lhe cercaram dizendo: - Mensageiro dos deuses, atendeste as nossas preces. Vivíamos em guerra, e os deuses nos enviaram o sagrado jogo do par-ou-ímpar, que nos trouxe a paz. Mas este jogo só podia ser jogado por dois jogadores, e as trevas se abateram de novo sobre nós. Então um grande homem nos ensinou um novo jogo, que chamamos Zerinhoum em sua homenagem. Mas este jogo tinha problemas, e a guerra voltou a nos assolar. Por favor, ó grande sábio, que vem em nome dos deuses, ensina-nos um novo jogo, que possa nos trazer de volta nossa paz. E o anjo assim respondeu: - Eu vos ensinarei um novo jogo. Zerinhoum era um grande matemático, mas não conhecia os segredos dos deuses. Eu vos revelarei estes segredos. Para isto, o melhor é começar pelo antigo jogo do par-ou-ímpar. Como se decide se um número é par ou é ímpar? Basta dividi-lo por 2. Se o resto for igual a 0, o número será par, se for igual a 1, o número será ímpar. Estas são as únicas duas opções, porque o resto sempre é menor do que o dividendo (2). Reparai que se dividirmos o número por 3, passam a existir 3 opções para o resto, pois ele pode ser 0, 1 ou 2. Na divisão por 4, existem 4 restos possíveis ( 0, 1, 2 e 3). Em geral, quando dividimos um número por n , existem n restos possíveis ( 0, 1, 2, …, n – 2 e n – 1 ). E o que isto tem a ver com o jogo? Tudo, eu vos digo. Se n pessoas estiverem em uma disputa, vós fareis como eu vos digo: As pessoas escolherão, cada uma, um número entre 0 e n – 1 diferente. Depois, ao mesmo tempo, elas mostrarão as mãos, exibindo uma quantidade qualquer de dedos. As quantidades serão somadas, e o número resultante será dividido por n. A pessoa que escolheu o resto desta divisão será a vencedora.
EUREKA! N°5, 1999
18
Sociedade Brasileira de Matemática
Esta é a forma que os deuses jogam. Mas vós da Terra sois muito desorganizados para poder escolher tantos números de forma tranqüila. Portanto, eu vos ensinarei uma forma alternativa de jogar este jogo. Vós vos arrumareis em um círculo. Uma pessoa será designada a contar. Então vós gritareis a palavra mágica “Adedanha” e todos mostrarão as mãos. Os resultados serão somados, e aquele que havia sido designado fará o seguinte procedimento: Em primeiro lugar falará “Um”, e apontará para o céu, para que nunca vos esqueçais de que foram os deuses que vos ensinaram este jogo. Então apontará para si mesmo e falará “Dois”. Depois apontará para o jogador à sua esquerda e falará “Três”, e depois seguirá apontando para o jogador à esquerda deste e assim por diante, sempre acrescentando um ao número que havia falado anteriormente, até chegar à soma que havia sido calculada. O jogador que estiver sendo apontado neste momento será o vencedor. Se a soma for 1, o jogador que estiver à direita do que estiver contando será declarado vencedor. Se for 0, será o que estiver à direita deste. Os homens entenderam as determinações do mensageiro, mas ainda não entendiam porque o segundo jogo era equivalente ao primeiro. O anjo então lhes explicou: - A pessoa que está contando vai apontar para si mesma quando estiver falando “2”. Depois vai dar uma volta completa no círculo e vai apontar para si mesma novamente quando estiver no “2 + n”, e novamente no “2 + 2n”. Ou seja, ela vai estar apontando para si mesma se e somente se estiver falando um número cujo resto na divisão por n seja 2. Da mesma forma, vai estar apontando para o jogador à sua esquerda se e somente se estiver falando um número que deixa resto 3 ao ser dividido por n. E assim por diante, de forma que cada jogador terá associado a si um número entre 0 e n – 1 tal que ele é o vencedor se e somente se o resultado da soma deixa aquele resto quando dividido por n. Os homens estavam maravilhados com a explicação do mensageiro, mas um sábio ancião levantou uma questão: - Ó, mensageiro divino, sem dúvida és sábio e sagaz. No entanto, uma dúvida me corrói o espírito. Tendo cada jogador 10 dedos, esta soma pode atingir números muito elevados, fazendo com que o responsável pela contagem passe um tempo enorme falando e apontando até que se descubra o vencedor. - Tens toda a razão, sábio homem. Mas em verdade vos digo que é tolice que um jogador exiba uma quantidade de dedos maior ou igual à quantidade de jogadores. Com efeito, suponde que um jogador coloque um número maior ou EUREKA! N°5, 1999
19
Sociedade Brasileira de Matemática
igual a n. Os primeiros n dedos só vão ter o efeito de fazer com que a contagem dê uma volta completa no círculo, sem alterar em nada quem será o vencedor. Portanto, ele pode subtrair n da sua quantidade sem que isto altere o resultado. Se o número persistir maior ou igual a n, basta voltar a subtrair, até que o número fique entre 0 e n – 1. - Isto de fato diminui sobremaneira o esforço requerido- replicou o ancião. Mas ainda assim o resultado pode chegar a n(n – 1), que ainda é bastante grande. - És de fato perspicaz, meu nobre homem. Mas não penseis que a sabedoria dos deuses possui limite. O mesmo processo que foi aplicado a cada número individualmente pode ser aplicado à soma. Por exemplo, considerai um jogo com 4 jogadores. Suponde que um dos jogadores exibe 3 dedos e outro exibe 2 dedos. Por que considerar a sua soma como sendo 5, se o efeito de somar 4 é apenas fazer com que o responsável pela contagem dê uma volta a mais? Em vez disto, é muito mais sensato considerar a sua soma como sendo 5 – 4 = 1. Mais geralmente, considere um jogo com n jogadores. Em primeiro lugar diminui-se n dos valores jogados por cada um, de forma que todos eles estejam entre 0 e n – 1 (se todos os jogadores dessem ouvidos às palavras dos deuses, não jogariam além destes limites). Depois procede-se a soma, da seguinte forma. Soma-se o primeiro valor com o segundo. Caso esta soma seja um valor maior ou igual a n, subtrai-se n do resultado ( Sede espertos e sabereis que fazendo isto sempre obtereis um número entre 0 e n – 1 ). Depois, a este resultado, soma-se o terceiro valor, tomando-se o cuidado de se subtrair n caso a soma exceda n – 1. Prossegue-se desta forma até que todos os valores tenham sido somados. Se seguistes o meu raciocínio até este ponto, não deveria ser-vos surpresa o fato que o resultado de uma tal operação está sempre entre 0 e n – 1, e portanto o jogador responsável pela contagem nunca precisará dar mais de uma volta. E então todos os habitantes se ajoelharam aos pés do anjo, reconhecendo a sua suprema sabedoria, e o mundo conheceu enfim a paz. Até hoje os homens jogam os jogos de par-ou-ímpar e adedanha da forma como foram ensinados pelos deuses, embora, infelizmente, a maioria tenha se esquecido da lição final e continue se extenuando em uma interminável contagem que dá voltas e mais voltas.
EUREKA! N°5, 1999
20
Sociedade Brasileira de Matemática
E foi assim que a lenda me foi contada pela minha avó, que ouviu de sua avó, que ouviu de sua própria avó, e assim por diante, até o princípio dos tempos. Você deve estar achando meio esquisita a maneira de somar que foi ensinada pelos deuses. No entanto, eles a usaram em várias outras coisas que nos são muito familiares. Se você não acredita, responda rápido a estas perguntas: a) Se uma coisa começa em uma segunda-feira e dura 7 dias, em que dia ela termina? E se durar 14 dias? E se durar 701 dias? b) Se uma coisa começa às 8 horas da manhã e dura 24 horas, a que horas ela acaba? E se durar 48 horas? E se durar 4804 horas? c) Se o ponteiro dos minutos de um relógio está apontando 23 minutos, para onde ele estará apontando daqui a 60 minutos? e daqui a 120 minutos? e daqui a 66681 minutos? Garanto que, se você respondeu à terceira pergunta dos 3 ítens, não contou de um em um (ou então já estamos no terceiro milênio ☺). Você percebeu que os dias da semana se repetem de 7 em 7 dias, que as horas do dia se repetem de 24 em 24 horas e que o ponteiro do relógio volta a apontar para o mesmo ponto de 60 em 60 minutos. Garanto também que você, sem se dar conta, já pensou várias vezes coisas como “5 horas depois das 21 horas são 2 horas da manhã”, ou seja, fez a conta 21 + 5 = 2 ! E, por incrível que pareça, esta conta está certa!!! Está certa, porque você está pensando a menos de múltiplos de 24 (ou, como preferem os matemáticos, módulo 24) , ou seja: 21 + 5 = 2 ( + um múltiplo de 24 ) , ou, como preferem os matemáticos, 21 + 5 = 2 (mod 24) . Desta forma, a terceira pergunta do item c) pode ser reescrita como “Quanto é 23 + 66681 (mod 60)” . Se você foi esperto(a) o suficiente para responder àquela pergunta, você já deve ter percebido que 66681 = 21 (mod 60), e que 23 + 66681 = 23 + 21 (mod 60), ou seja, 23 + 66681 = 44 (mod 60), logo o ponteiro estará apontando para o minuto 44. Só para ver se você entendeu até agora, preencha estas lacunas: EUREKA! N°5, 1999
21
Sociedade Brasileira de Matemática
2 + 2 = 1 (mod __ ) 2 +__ = 0 (mod 17) 26 = 3 (mod __ ) Não se esqueça que a expressão “mod n” é só uma forma abreviada de “+ um múltiplo de n”. Lembrando-se disto, veja quantas coisas você sabia, mas não sabia que sabia: 3 × 3 = 1 (mod 4) 1 = – 1 (mod 2) 2 × 2 × 2 × 2 = 1 (mod 5) 3 × 3 × 3 × 3 = 1 (mod 5) (esta talvez você não saiba, mas n × n × n × n = 1 (mod 5), sempre que n não é múltiplo de 5. Você pode ver isto e muito mais no artigo do professor Carlos Gustavo Moreira, na EUREKA! No. 2. Pergunta: se n é múltiplo de 5, quanto é n × n × n × n (mod 5)? ) Agora que você já sabe o segredo dos deuses matemáticos, já pode jogar adedanha da forma original, como os deuses a conceberam, e manter a paz no mundo sem fazer esforço.
EUREKA! N°5, 1999
22
Sociedade Brasileira de Matemática
QUADRILÁTEROS E TRIÂNGULOS Marcelo Mendes
♦ Nível Intermediário Apresentamos a seguir alguns resultados que servem de ferramenta para resolução de problemas de geometria elementar envolvendo quadriláteros e triângulos, bastante freqüentes em problemas de olimpíada.
QUADRILÁTEROS INSCRITÍVEIS
Os ângulos opostos de um quadrilátero inscritível são suplementares. Reciprocamente, se os ângulos opostos de um quadrilátero são suplementares, então esse quadrilátero é inscritível (cíclico). D C O
A
B
Além disso, se ocorrer uma situação onde dois ângulos iguais “olham” para um mesmo segmento, então os extremos desse segmento e os vértices dos dois ângulos formam um quadrilátero inscritível. D α
C α
A
B
Exemplo: Seja AB o diâmetro de um semicírculo. Um ponto M é marcado no semicírculo e um ponto K é marcado sobre AB. Um círculo com o centro P passa EUREKA! N°5, 1999
23
Sociedade Brasileira de Matemática
por A, M, K e um círculo com centro Q passa por M, K, B. Prove que M, K, P, Q pertencem a um mesmo círculo.
Solução: No círculo circunscrito de AMK, ∠MPK = 2∠MAK; e no círculo circunscrito de BMK, ∠MQK = 2∠MBK. Como AB é diâmetro do semicírculo, ∠AMB = 90o e ∠MAK+∠MBK = 90o. Daí, ∠MPK+∠MQK = 180o e MPKQ é inscritível. TEOREMA DE PTOLOMEU
Se ABCD é um quadrilátero inscritível de diagonais AC e BD, então: AB × CD + AD × BC = AC × BD. A
d
D
a
c E
B
b
C
Prova: Seja x = BD e y = AC e a, b, c, d, os comprimentos dos lados. Construa ∠CDE = ∠ABD, E ∈ AC. Daí, ∆CDE ∼ ∆ADB e ∆ADE ∼ ∆BCD, dando, respectivamente, EC⋅x = ac e AE⋅x = bd. Somando essas duas últimas equações, temos xy = ac + bd, como queríamos provar ❑ Há também uma extensão para esse teorema que vale para quadriláteros não inscritíveis: AB × CD + AD × BC > AC × BD, isto é, numa situação geral vale AB × CD + AD × BC ≥ AC × BD.
Exemplo: Prove que, se ABCDEFG é um heptágono regular convexo, então: 1 1 1 . = + AB AC AD
Aplicando o Teorema de Ptolomeu no quadrilátero inscritível ACDE, onde EUREKA! N°5, 1999
24
Sociedade Brasileira de Matemática
CD = DE = a = AB, AC = CE = b e AD = AE = c, temos bc = ac + ab. Dividindo essa última equação por abc, segue o resultado. A RELAÇÃO ENTRE A DISTÂNCIA DO ORTOCENTRO A UM VÉRTICE E DO CIRCUNCENTRO AO LADO OPOSTO
Sejam H e O respectivamente o ortocentro e o circuncentro, do ∆ABC e M, o ponto médio do lado BC. Então AH = 2⋅OM.
A Y H
B
X
O
N
M
C
Prova: Sejam AX e BY alturas e N, o ponto médio de AC. Como MN é base média, MN // AB e MN = ½AB. Daí, ∆ABH ∼ ∆OMN pois têm lados paralelos entre si (e razão 2:1). Portanto, AH = 2⋅OM ❑ Exemplo: Prove que o ortocentro, o baricentro e o circuncentro de um triângulo qualquer são colineares. (Reta de Euler) Seja G a interseção de AM e HO (na figura acima). Então, ∆AHG ∼ ∆GOM na razão 2:1. Daí, AG = 2⋅GM. Portanto, G é o baricentro e pertence à reta HO. PROBLEMAS
1.
Seja P um ponto sobre o menor arco AC da circunferência circunscrita a um triângulo equilátero ABC. Calcule a medida do ângulo ∠APC.
2.
Prove que um trapézio é inscritível se, e somente se, ele for isósceles (lados não paralelos iguais).
3.
Sejam AX e BY alturas de um triângulo isósceles ABC (AC = BC) de ortocentro H. Prove que 2⋅HX⋅XC = XY⋅HC.
EUREKA! N°5, 1999
25
Sociedade Brasileira de Matemática
4.
Seja ABCD um losango inscritível de lado 1 e P, um ponto sobre o menor arco CD. Prove que PD2 + PC⋅PA = 1.
5.
Seja P um ponto sobre o menor arco AC da circunferência circunscrita a um triângulo equilátero ABC. Prove que PB = PA + PC.
6.
Seja H o ortocentro de um triângulo ABC e P, o ponto diametralmente oposto a B na circunferência circunscrita a ABC. Prove que AHCP é um paralelogramo.
7.
ABCD é um paralelogramo. H é o ortocentro do ∆ABC e O, o circuncentro do ∆ACD. Prove que H, O, D são colineares. 1 1 1 Seja A1A2…An um polígono regular de n lados. Se , = + A1 A2 A1 A3 A1 A4 calcule n.
8.
9.
Sejam M, N, P os pontos médios dos lados de um ∆ABC acutângulo de circuncentro O. Prolongue MO, NO, PO, a partir de O, até X, Y, Z, respectivamente, tais que MX = 2 ⋅ OM , NY = 2 ⋅ ON , PZ = 2 ⋅ OP. Prove que ∆XYZ é semelhante ao ∆ABC .
10.
Sejam M, N, P os pontos médios dos lados de um ∆ABC acutângulo de circuncentro O. Prolongue MO, NO, PO, a partir de O, até X, Y, Z, respectivamente, tais que MX, NY, PZ tenham comprimentos respectivamente iguais às metades das alturas do triângulo a partir dos vértices A, B, C. Prove que ∆XYZ é semelhante ao triângulo órtico de ABC (triângulo formado pelos pés das alturas do ∆ABC).
EUREKA! N°5, 1999
26
Sociedade Brasileira de Matemática
O PRINCÍPIO DAS GAVETAS ♦ Nível Intermediário
Paulo Cezar Pinto Carvalho - IMPA
Muitos problemas atraentes de matemática elementar exploram relações entre conjuntos finitos, expressas em linguagem coloquial. Parte de sua atração vem justamente do fato de que podem ser formulados e, muitas vezes, resolvidos sem recorrer a fórmulas ou a técnicas complicadas. Vejamos um exemplo simples.
Exemplo 1. Qual é o número mínimo de pessoas que devemos reunir para que tenhamos certeza de que entre elas há duas que fazem aniversário no mesmo mês? Solução: A resposta é 13. Se houvesse apenas 12 pessoas, seria possível que cada uma delas fizesse aniversário em um mês diferente. Com 13 pessoas, há , obrigatoriamente, pelo menos um mês com mais de um aniversário (se houvesse, no máximo, um aniversário por mês, o número de pessoas presentes seria, no máximo, 12). O argumento empregado acima é conhecido como Princípio das Gavetas de Dirichlet ou Princípio das Casas do Pombos. Um possível enunciado para este princípio é o seguinte:
Se n objetos forem colocados em, no máximo, n – 1 gavetas, então pelo menos uma delas conterá pelo menos dois objetos. (Uma maneira um pouco mais formal de dizer o mesmo é: se o número de elementos de um conjunto finito A é maior do que o número de elementos de um outro conjunto B, então uma função de A em B não pode ser injetiva.) Embora trate-se de um fato extremamente elementar, ele é útil para resolver problemas que, pelo menos à primeira vista, não são imediatos. Para aplicá-lo, devemos identificar, na situação dada, quem faz o papel dos objetos e quem faz o papel das gavetas.
Exemplo 2. Uma prova de concurso possui 10 questões de múltipla escolha, com cinco alternativas cada. Qual é o menor número de candidatos para o qual podemos garantir que pelo menos dois deles deram exatamente as mesmas respostas para todas as questões? EUREKA! N°5, 1999
27
Sociedade Brasileira de Matemática
Solução: Neste caso, os objetos são os alunos e as gavetas são as possíveis seqüências de respostas . Como cada questão pode ser respondida de 5 modos, a prova pode ser preenchida de 5 × 5 × 5 × … 5 = 510 = 9 765 625 modos. Logo, só se pode ter a certeza de que dois candidatos fornecem exatamente as mesmas respostas se houver pelo menos 9 765 626 candidatos. Exemplo 3. Em uma reuniao há n pessoas. Mostre que existem duas pessoas que conhecem exatamente o mesmo número de outros participantes (admitimos que “conhecer”seja uma relação simétrica, ou seja, se a conhece b, então b conhece a). Solução: Os objetos são as pessoas. As gavetas, naturalmente, são as quantidades de pessoas conhecidas. Temos, no entanto, uma dificuldade: as possíveis quantidades de conhecidos são 0, 1, 2, …, n – 1. Assim, à primeira vista, temos n gavetas para n objetos, o que nos impede de usar o princípio das gavetas. Note, porém, que as gavetas 0 e n – 1 não podem ser usadas simultaneamente: se existir uma pessoa que não conhece nenhum participante, então não pode existir um participante que conheça todos! Assim, uma das gavetas 0 ou n – 1 permanece desocupada e os n objetos devem ser, portanto, distribuídos em n – 1 gavetas. Portanto, uma delas será ocupada por pelo menos dois objetos, o que mostra que há duas pessoas que conhecem exatamente o mesmo número de participantes. Nos casos anteriores, foi bastante simples identificar as gavetas. Nem sempre é assim. Os exemplos a seguir ilustram situações em que é necessário “construir” as gavetas a serem usadas.
Exemplo 4: Escolhem-se 5 pontos ao acaso sobre a superfície de um quadrado de lado 2. Mostre que pelo menos um dos segmentos que eles determinam tem comprimento menor ou igual a 2 . Solução: Neste caso, está claro que os objetos são os 5 pontos. O ponto chave da resolução está na identificação das gavetas. Devemos subdividir o quadrado dado em 4 partes de modo tal que a distância entre dois pontos situados em uma destas partes nunca seja maior que 2 . A Fig. 1 mostra como fazê-lo: basta dividi-lo nos quatro quadrados determinados pelas retas que unem os pontos médios dos lados opostos. Em cada uma destas quatro “gavetas”, a distância máxima entre dois pontos é igual à sua diagonal, que mede 2 . Portanto, dados 5 pontos, pelo menos 2 estarão em uma mesma “gaveta” e, assim, determinam um segmento de comprimento menor ou igual a 2 . EUREKA! N°5, 1999
28
Sociedade Brasileira de Matemática 1 1
1
2
1
Figura 1
Exemplo 5. Escolha 101 números dentre os elementos do conjunto {1, 2, 3, …, 200}. Mostre que, dentre os números escolhidos, há sempre dois números tais que um divide o outro. Solução: Antes de mais nada, observe que podemos escolher 100 números do conjunto sem que exista um par onde um número divide o outro: basta tomar os números 101, 102, …, 200. É claro que se acrescentamos mais um número p (obrigatoriamente menor ou igual a 100) a essa coleção, um múltiplo seu já estará lá. Na verdade, podemos garantir que esse múltiplo é da forma 2rp (basta tomar p e multiplicá-lo sucessivamente por 2 até que ele se torne maior do que 100). Mostraremos que isso ocorre para qualquer conjunto de 101 elementos. Ou seja, todo subconjunto com 101 dos números de 1 a 200 sempre contém um número e um múltiplo seu obtido através de multiplicação por uma potência de 2. Note que esta afirmativa é mais forte do que a dada do enunciado, mas, como veremos, nos permite estruturar uma demonstração. Isto ocorre com frequência nos problemas envolvendo o Princípio das Gavetas: parte do sucesso nas soluções depende da habilidade em perceber o que deve ser demonstrado. Voltando à solução, observemos que todo inteiro n se escreve, de modo único, na forma n = 2rb, onde r é um inteiro não negativo e b é um número ímpar. Por exemplo, 18 = 21.9 , 36 = 22.9 e 125 = 20.125. Para os números de 1 a 200, os valores possíveis de b são os ímpares de 1 a 199, que são 100. Aqui estão nossas gavetas! Já que há 100 valores possíveis de b, qualquer coleção de 101 números de 1 a 200 possui dois números x = 2rb e y = 2sb com o mesmo b (isto é, temos dois objetos que serão colocados na mesma gaveta). Se r < s, então x divide y; senão, y divide x, o que conclui a demonstração. O último exemplo requer um argumento um pouco mais sofisticado.
Exemplo 6: Em uma reunião, há 6 pessoas. Mostre que necessariamente existem 3 pessoas que se conhecem mutuamente ou 3 pessoas que não se conhecem mutuamente (como no exemplo 3 admitimos que, se a conhece b, então b conhece a). EUREKA! N°5, 1999
29
Sociedade Brasileira de Matemática
Solução: Usaremos o diagrama da Fig. 2 abaixo para ilustrar a situção. Cada pessoa é representada por um vértice do hexágono. Quando duas pessoas se conhecem, ligamos os vértices correspondentes por um segmento contínuo; senão, usamos um segmento tracejado. O que devemos mostrar é que, nesta figura, necessariamente existe um triângulo formado por linhas contínuas ou um triângulo formado por linhas tracejadas.
Figura 2
Consideremos os segmentos que incidem em um dos vértices p1. Como eles são 5, há pelo menos 3 deles que são contínuos ou pelo menos 3 que são tracejados. Admitamos que haja 3 contínuos (o argumento seria análogo no outro caso). Denotemos por p2, p3 e p4 vértices ligados a p1 por segmentos contínuos (veja a Fig. 3). Se algum dos segmentos p2p3, p2p4 ou p3p4 é contínuo, este segmento, juntamente com os que ligam seus extremos a p1, formam um triângulo contínuo. Por outro lado, se nenhum deles é contínuo, eles formam um triângulo tracejado, o que completa a demonstração. p2
p3
p1
p4
Figura 3
Este exemplo é um caso particular de um teorema mais geral, chamado de Teorema de Ramsey. Dado qualquer inteiro k ≥ 3, existe um inteiro R(k) tal que, em uma reunião de R(k) pessoas, sempre existem k que se conhecem EUREKA! N°5, 1999
30
Sociedade Brasileira de Matemática
mutuamente ou k que não se conhecem mutuamente. Este resultado normalmente é expresso usando a linguagem de grafos: ao se colorir, com duas cores, as arestas de um grafo completo com R(k) vértices, há sempre um subgrafo completo com k vértices onde todas as arestas têm a mesma cor. (Na realidade, o Teorema de Ramsey aborda situações mais gerais; veja, por exemplo os problemas 8 e 9 abaixo). Aproveitamos para mencionar a seguinte generalização do princípio das gavetas: Se n objetos são colocados em m gavetas e n > mk (onde m, n e k são números naturais) então alguma gaveta conterá pelo menos k + 1 objetos. Terminamos com uma lista de problemas que podem ser resolvidos com as técnicas aqui ilustradas. As soluções serão publicadas nos próximos números da EUREKA!. PROBLEMAS
1) Numa gaveta há 6 meias pretas e 6 meias brancas. Qual é o número mínimo de meias a se retirar (no escuro) para garantir que: a) b)
As meias retiradas contenham um par da mesma cor? As meias retiradas contenham um para de cor branca?
2) Sejam n um natural ímpar e A uma matriz simétrica em que cada linha e coluna seja uma permutação dos inteiros 1, 2,…, n. Mostre que cada um destes números aparece uma vez na diagonal de A. 3) Mostre que se um subconjunto com n + 1 elementos é escolhido do conjunto {1, 2, 3,…, 2n} então este subconjunto necessariamente contém um par de números primos entre si. 4) Considere 9 pontos de coordenadas inteiras no R3. Mostre que o ponto médio de um dos segmentos de reta definidos por estes pontos também tem coordenadas inteiras. 5) Mostre que se n é ímpar e a1, a2,…,an é uma permutação de 1, 2,…,n, então o produto (a1 – 1) (a2 – 2)…(an – n) é par. 6) Mostre que em qualquer coleção de n inteiros há um subconjunto cuja soma dos elementos é divisível por n. EUREKA! N°5, 1999
31
Sociedade Brasileira de Matemática
7) Mostre que em qualquer coleção de n inteiros existe um par cuja soma ou diferença é divisível por n. 8) Mostre que em toda reunião com 10 pessoas existem 3 que se conhecem mutuamente ou 4 que se desconhecem mutuamente. Mostre que, na realidade, o resultado vale mesmo que na reunião só existam 9 pessoas. 9) Dados inteiros a, b ≥ 2, seja N (a, b) o menor número para o qual, dado um conjunto com N (a, b) pessoas, sempre existam a que se conheçam mutuamente ou b que se desconheçam mutuamente (se existir tal número). Os problemas anteriores implicam que N (3, 3) ≤ 6 e N (3, 4) ≤ 9. Mostre que: a) b) c)
N(a, 2) = a; N(a, b) = N (b, a); N(a, b) ≤ N (a – 1, b) + N (a, b – 1); observe que, em consequência, N(a, b) existe para todo par (a, b).
10) Dois discos A e B são divididos em 2n setores iguais. No disco A, n setores são pintados de azul e n de vermelho. No disco B, os setores são pintados de azul ou vermelho de forma completamente arbitrária. Mostre que A e B podem ser superpostos de modo que pelo menos n setores tenham cores coincidentes. 11) Sejam A1, A2,…, A100 subconjuntos distintos de um conjunto X satisfazendo a propriedade de que cada Ai possua mais da metade dos elementos de X. Mostre que existem 6 elementos x1, x2,…x6 de X tais que cada Ai contenha pelo menos um destes 6 elementos. 12) Considere um conjunto A com n elementos. Seja F uma família de subconjuntos de A tal que: -
Quaisquer dois elementos de F têm interseção não vazia. Nenhum outro subconjunto de A intersecta todos os elementos de F.
a) Dê exemplo de uma família F satisfazendo a estas condições. b) Mostre que F possui 2n – 1 elementos.
13) Uma fábrica produz pelo menos uma unidade de um produto X por dia e no máximo 10 unidades deste produto por semana. Mostre que dado qualquer EUREKA! N°5, 1999
32
Sociedade Brasileira de Matemática
inteiro positivo n existe um conjunto de dias consecutivos em que a produção total é igual a n [ Sugestão: mostre que existe um número k (dependente de n) suficientemente grande para o qual os conjuntos {S1, S2,…Sk} e {S1 + n, S2 + n, …, Sk + n} tem pelo menos um elemento comum, onde Si é a soma das produções nos dias 1, 2, …, i.].
14) Mostre que toda sequência com n2 + 1 elementos possui uma subsequência crescente com n + 1 elementos ou uma subsequência decrescente com n + 1 elementos. 15) Sejam mn + 1 elementos tais que a1 < a2 < …< amn + 1. Mostre que ou existem m + 1 destes números tais que nenhum é divisor de um outro ou existem n + 1 deles tais que cada um é divisor do seguinte. 16) Prove que se o conjunto {1, 2, 3, …, 1978} é partido em 6 subconjuntos, em algum destes subconjuntos existe um elemento que é igual à soma de dois elementos, não necessariamente distintos, do mesmo subconjunto. 17) Considere um conjunto com 2n pontos. a) Mostre que é possível conectar estes pontos com n2 segmentos de reta sem que um triângulo de vértices nos pontos dados seja formado. b) Mostre que se os pontos são conectados por n2 + 1 segmentos de reta, então pelo menos um triângulo é formado.
18) Considere um conjunto de n pontos 1, 2, …, n. Para cada par de pontos é escolhida uma orientação para o segmento de reta que os une. Se o segmento ij é orientado de i para j dizemos que i → j. Mostre que existe uma permutação a1, a2, … an de 1, 2, …, n tais que a1 → a2 → … → an. 19) São dados n pontos azuis e n pontos vermelhos no plano. Mostre que é possível formar n pares de pontos (um azul e um vermelho em cada par) de modo que os n segmentos de reta definidos por estes pares não se cruzem. 20) Mostre que dados 5 pontos do plano em posição geral há 4 que formam um quadrilátero convexo.
EUREKA! N°5, 1999
33
Sociedade Brasileira de Matemática
DESIGUALDADES ELEMENTARES Antonio Caminha Muniz Neto ♦ Nível Avançado. Pretendemos neste artigo desenvolver ferramentas básicas a fim de que o leitor se torne apto a resolver uma vasta gama de problemas de competições matemáticas que envolvam desigualdades. Tentamos tornar nossa exposição a mais auto-suficiente possível. Em certas passagens, contudo, algum conhecimento de cálculo é útil, ainda que não imprescindível. Em tais ocasiões indicamos ao leitor a referência [3] como bibliografia auxiliar. Antes de discutirmos qualquer desigualdade em especial, consideremos um exemplo preliminar.
Exemplo
1:
Para
todo
1 + + + +... + ≥ 1 2
1 3
1 4
1 n
positivo
n,
1 1 1 k + k + ...+ k 2 2 2
=
inteiro
1 2
(log
2
)
prove
que
n +1 .
Solução : Veja que, para todo inteiro k > 1, 1 2 k −1 +1
+
1 2 k −1 + 2
+ ... +
1 2k
>
1 2
2 k −1 vezes
Portanto, sendo 2 k a maior potência de 2 menor ou igual a n, temos
1 + 12
n
+∑
j =3
1 j
≥ 1 + 12 +
∑ 1j = 1 + 12 + ∑( 2 1 +1 + ... + 21 ) >1 + 12 + ∑ 12 = 1 + k2 2k
k
j =3
j =2
k
j −1
Mas 2 k ≤ n < 2 k +1 ⇒ k ≤ log 2 n < k + 1 ⇒ 1 + k2 >
j
1 2
(log
2
j =2
)
n + 1 , e a desigualdade
do enunciado é imediata. ❏ O exemplo acima foi colocado de propósito. Ele chama atenção para o fato de que nem sempre precisamos de algo mais que raciocínio para resolver problemas envolvendo desigualdades. A proposição abaixo mostra um pouco mais sobre como podemos derivar desigualdades interessantes com muito pouca matemática.
EUREKA! N°5, 1999
34
Sociedade Brasileira de Matemática
Proposição 1 (Desigualdade do Rearranjo): Sejam a1 < a 2 < ... < a n reais positivos dados, e considere a expressão S = a1b1 + a 2 b2 + ...+ a n bn , onde ( b1 , b2 ,..., bn ) é uma reordenação de ( a1 , a2 ,..., a n ) . Então
a1a n + a 2 a n −1 + ...+ a n a1 ≤ S ≤ a12 + a 22 + ...+ a n2 Prova : Vamos primeiro tornar S a maior possível. Como só há um número finito (n fatorial) de possíveis reordenações ( b1 , b2 ,..., bn ) , há uma delas que torna S máxima. Suponha então que estamos com a reordenação ( b1 , b2 ,..., bn ) que torna S máxima. Queremos mostrar que essa reordenação é exatamente ( a1 , a2 ,..., a n ) . Para isso, basta mostrarmos que deve ser b1 < b2 < ... < bn . Suponha o contrário, i.e., que existam índices i < j tais que bi > b j . Trocando as posições de bi e b j (i.e.,
bi ao lado de a j e b j ao lado de ai ), S varia de a i b j + a j bi − ( a i bi + a j b j ) = ( a i − a j )( b j − bi ) > 0 , quer dizer, S aumenta. Mas isso contraria o fato de ser a reordenação ( b1 , b2 ,..., bn ) aquela que torna S máxima. Logo, b1 < b2 < ... < bn e daí bi = ai para todo i, donde o maior valor pondo
possível de S é a12 + a 22 + ...+ a n2 . O raciocínio para minimizar S é análogo. ❏ Passemos agora a nosso principal objetivo, o estudo de desigualdades especiais. A mais importante destas é a dada pela proposição 2 abaixo. Antes, uma definição.
Definição 1 : Dados n > 1 reais positivos a1 , a2 ,..., a n , definimos
i. A média aritmética de a1 , a2 ,..., a n como o número ii. A média geométrica de a1 , a2 ,..., a n como o número
a1 + a 2 + ...+ a n n n
.
a1a2 ... a n .
Proposição 2 (Desigualdade Entre as Médias Aritmética e Geométrica) : Dados n > 1 reais positivos a1 , a2 ,..., a n , sua média aritmética é sempre maior ou igual que a média geométrica, ocorrendo a igualdade se e só se a1 , a2 ,..., a n forem todos iguais. Em símbolos: EUREKA! N°5, 1999
35
Sociedade Brasileira de Matemática a1 + a 2 + ...+ a n n
≥ n a1a 2 ... a n
Prova : Façamos a prova em dois passos:
i. A desigualdade é verdadeira quando n for uma potência de 2, ocorrendo a igualdade se e só se todos os números forem iguais.
ii. A desigualdade é verdadeira em geral, e a igualdade ocorre se e só se os números forem todos iguais.
i. Façamos indução sobre k ≥ 1, sendo n = 2 k : Para k = 1, temos a1 + a 2 2
≥ a1a2 ⇔ a1 − 2 a1a2 + a2 ≥ 0 ⇔ ( a1 − a2 ) 2 ≥ 0 , o que é
verdade. Há igualdade se e só se ( a1 − a 2 )2 = 0 , i.e., se e só se a1 = a2 . Se já provamos que
a1 + a 2 + ...+ a n n k
≥ n a1a 2 ... a n , com igualdade se e só se
a1 = ... = a n para n = 2 , então ( a1 +...+ a n ) + ( a n +1 +...+ a 2 n ) 2n
≥
n
[
1 a1 +...+ a n 2 n
=
n
a1... a n
+
a n +1 +...+ a 2 n n
]
≥
n
a1 ...a n +
n
a n +1 ...a 2 n
2
≥
a n +1... a2 n = 2 n a1... a n a n +1... a2 n
Para haver igualdade, devemos ter igualdade em todas as passagens. Então, deve ser a1 +...+ a n n n
= n a1... a n ,
a1 ...a n +
n
2
a n +1 ...a 2 n
a n +1 +...+ a 2 n n
= n a n +1... a2 n e
= 2 n a1... a n a n +1... a2 n
Para as duas primeiras igualdades, segue da hipóteses de indução que deve ser a1 = ... = a n e a n +1 = ... = a2 n A última igualdade ocorre se e só se n a1... a n = n a n +1... a 2 n . Estas duas condições juntas implicam que devemos ter a1 = ... = a n = a n +1 = ... = a 2 n . É também evidente que se os números forem todos iguais a igualdade ocorre. EUREKA! N°5, 1999
36
Sociedade Brasileira de Matemática
ii. Seja agora n > 1 um natural qualquer e a1 ,a2 ,...,an reais positivos. Tome k natural tal que 2 k > n. Usando a desigualdade entre as médias para os
2 k números a1 , a2 ,..., a n e 2 k − n cópias de a = n a1a 2 ... a n , obtemos a1 + ...+ a n + a + ...+ a 2
k
k
≥ 2 a1... a n a 2
k
−n
=
2k
a na 2
k
−n
=
a1 + ...+ a n n
e daí a1 + ...+ a n + ( 2 k − n )a ≥ 2 k a , ou ainda
2k
k
a2 = a ,
≥ a = n a1 ... a n , que
era a desigualdade desejada. Para haver igualdade, segue do item i que deve ser a1 = ... = a n = a = ... = a . Em particular, todos os números a1 , a2 ,..., a n devem ser iguais. É fácil ver que se esses números forem todos iguais então há igualdade. ❏
Corolário 2.1 : Dados n > 1 reais positivos a1 , a2 ,..., a n , temos
(
( a1 + a2 + ...+ a n )
1 a1
+
1 a2
)
+ ...+ a1 ≥ n 2 , n
com igualdade se e só se a1 , a2 ,..., a n forem todos iguais.
Prova : Basta aplicarmos duas vezes a proposição 2 e multiplicarmos os resultados:
a1 + a2 + ...+ a n ≥ n n a1a 2 ... a n e
1 a1
+
1 a2
+ ...+ a1 ≥ n n n
1 a1a 2 ...a n ❏
Exemplo 2 : (Olimpíada Israelense) Sejam k e n inteiros positivos, n > 1. Prove que 1 kn
+
1 kn +1
+ ...+ kn +1n −1 > n
∑(
1 kn + j
+1 =
(
n k +1 k
)
−1
Prova : Basta ver que
n −1 ∑ 1 + n = kn + j j=0
n −1 j =0
)
n −1
∑
j=0
EUREKA! N°5, 1999
37
kn + j +1 kn + j
n −1
> nn ∏
j =0
kn + j +1 kn + j
=nn
k +1 k
,
Sociedade Brasileira de Matemática
onde aplicamos a desigualdade entre as médias aritmética e geométrica uma vez. Note que, como os números
kn + j +1 kn + j
são dois a dois distintos, não há igualdade,
razão do sinal > acima. ❏ Dentre todas as desigualdades especiais que temos oportunidade de usar em problemas de competições matemáticas, a desigualdade a seguir se constitui, juntamente com a desigualdade entre as médias aritmética e geométrica, num dos dois mais importantes resultados a serem guardados.
Proposição 3 (Desigualdade de Cauchy): Sejam a1 , ..., a n , b1 , ..., bn reais dados, não todos nulos (n > 1). Então
| a1b1 + ... + a n bn | ≤ a12 +... + a n2
b12 +... + bn2
Além disso, teremos a igualdade se e só se os ai e os bi forem proporcionais, i.e., se e só se existir um real positivo λ tal que bi = λ ai para todo i.
Prova : Considere o seguinte trinômio do segundo grau
f ( x ) = ( a1 x − b1 )2 + ( a2 x − b2 )2 + ...+( a n x − bn ) 2 Desenvolvendo os parênteses, chegamos a
f ( x ) = (a12 + a22 + ...+an2 )x 2 − 2(a1b1 + a2b2 + ...+anbn )x + (b12 + b22 + ...+bn2 ) Por ser uma soma de quadrados, temos f ( x ) ≥ 0 para todo real x, e daí deve ser ∆ ≤ 0 , i.e., 4( a1b1 + a2 b2 + ...+ a n bn )2 ≤ 4( a12 + a22 + ...+ a n2 )( b12 + b22 + ...+ bn2 ) Cancelando o fator 4 e extraindo a raiz quadrada de ambos os membros, chegamos na desigualdade de Cauchy. Examinemos agora a igualdade. Se houver igualdade, quer dizer, se for ∆ = 0 , então o trinômio tem uma raiz real λ :
( a1λ − b1 )2 + ( a2 λ − b2 )2 + ...+( a n λ − bn )2 = 0
Mas aí todos os parênteses devem ser nulos, i.e., bi = λ ai para todo i. Então, havendo igualdade os a i e bi devem ser proporcionais. É evidente que se eles forem proporcionais a igualdade ocorre. ❏ Temos a seguir alguns corolários importantes. EUREKA! N°5, 1999
38
Sociedade Brasileira de Matemática
Corolário 3.1 (Desigualdade entre as Médias Quadrática e Aritmética) : Dados reais positivos a1 , ..., a n , temos
a12 + a 22 + ...+ a n2 n
≥
a1 + a 2 +..+ a n n
, com igualdade
se e só se a1 = ... = a n . Prova Fazendo b1 = b2 = ... = bn = 1 na desigualdade de Cauchy, obtemos
a1 + a2 + ... + a n ≤ a12 +... + a n2
n,
com igualdade se e só se existir um real positivo λ tal que ai = λ para todo i, quer dizer, se e só se os ai forem todos iguais. Para obter a desigualdade do enunciado, basta dividir ambos os membros da desigualdade acima por n. ❏
Corolário 3.2 : Se n > 1 é inteiro e a1 , ..., a n , b1 , ..., bn são reais positivos, então
(
a1 b1
)
+ ...+ bn (a1b1 +... + a n bn ) ≥ (a1 +... + a n ) , a
n
2
com igualdade se e só se b1 = ... = bn . ai bi
Prova : Faça x i =
, yi = a i bi e aplique a desigualdade de Cauchy para os
números x1 ,..., x n , y1 ,..., y n . ❏ Exemplo 3 : (Teste de Seleção da Romênia para IMO) Sejam x1 , x 2 ,..., x n +1 reais positivos tais que x1 + x 2 + ...+ x n = x n +1 . Prove que
x1( xn+1 − x1) + ...+ xn ( xn+1 − xn ) ≤ xn+1( xn+1 − x1 ) + ...+xn+1( xn+1 − xn ) Prova : Para 1 ≤ j ≤ n , seja y j = x n +1 − x j . Pela desigualdade de Cauchy temos
x1 y1 + ...+ x n y n ≤ =
x1 + ...+ x n
y1 + ...+ y n =
x n +1 ( x n +1 − x1 ) + ...+ ( x n +1 − x n ) ❏
EUREKA! N°5, 1999
39
Sociedade Brasileira de Matemática
Temos mais duas desigualdades importantes.
4
(Chebychev): Sejam a1 , ..., a n , b1 ,..., bn a1 ≤ a 2 ≤ ... ≤ a n e b1 ≤ b2 ≤ ... ≤ bn . Então
Proposição
(
a1 + a 2 + ...+ a n n
)(
b1 + b2 + ...+ b n n
)≤
reais,
com
a1 b1 + a 2 b2 + ...+ a n b n , n
com igualdade se e só se a1 = a2 = ... = a n ou b1 = b2 = ... = bn .
Prova : a1 b1 + a 2 b2 + ...+ a n b n n
=
1 n2
−
(
a1 + a 2 + ...+ a n n
)(
b1 + b2 + ...+ bn n
)=
[ n( a1b1 + a2b2 + ...+an bn ) − ( a1 + a2 + ...+an )( b1 + b2 + ...+bn )] = = 12 n
n
∑ (ai − a j )(bi − b j ) ≥ 0 ,
i , j =1
já que os ai , bi são igualmente ordenados. Note que a condição do enunciado é suficiente para haver igualdade. Por outro lado, suponha que tenhamos a igualdade. Como ( a i − a j )( bi − b j ) ≥ 0 para todos i, j, devemos ter ( a i − a j )( bi − b j ) = 0 para todos os i, j. Suponha que existisse um índice k com bk < bk +1 . Então b1 ≤ ... ≤ bk < bk +1 ≤ ... ≤ bn , e de ( a i − a k +1 )( bi − bk +1 ) = 0 segue que a i = a k +1 para i ≤ k. Portanto a1 = a2 = ... = a k = a k +1 . De ( a i − a k )( bi − bk ) = 0 e i > k concluímos que
a k +1 = ... = a n . Logo, todos os ai devem ser iguais. ❏
Corolário 4.1 : Sejam a1 , a2 ,..., a n reais positivos e k um natural. Então a1k + a 2k + ...+ a nk n
≥
(
),
a1 + a 2 + ...+ a n k n
com igualdade se e só se todos os ai forem iguais ou k ∈ {0, 1}. EUREKA! N°5, 1999
40
Sociedade Brasileira de Matemática
Prova : Por indução, o resultado acima é trivialmente verdadeiro para k = 1. Suponha k > 1 e o resultado válido para k - 1. Como ambos os membros da desigualdade acima são invariantes por permutações dos índices 1, 2, ..., n, podemos supor sem perda de generalidade que a1 ≤ a2 ≤ ... ≤ a n . Daí,
a1k −1 ≤ a 2k −1 ≤ ... ≤ a nk −1 , a1k
+ a 2k
+ ...+ a nk n
≥
(
e
a1 + a 2 + ...+ a n n
)
da
desigualdade
a1k −1 + a 2k −1 + ... + a nk −1 n
Pela hipótese de indução, vem que
de
Chebychev
obtemos
.
a1k −1 + a 2k −1 + ... + a nk −1 n
≥
(
)
a1 + a 2 + ... + a n k −1 . n
Combinando as duas desigualdades acima segue o resultado. A condição de igualdade é óbvia a partir da desigualdade de Chebychev. ❏ Por fim, vejamos algo um pouco mais sofisticado.
Definição 2 : Seja I um intervalo da reta e f : I → R uma função. A função f é dita
( )≤ ii. Côncava se f ( )≥ i. Convexa se f
x+ y 2
f ( x )+ f ( y ) 2
para todos os x, y em I.
x+ y 2
f ( x )+ f ( y ) 2
para todos os x, y em I.
Nas aplicações, quase sempre lidamos com funções contínuas (se você não sabe o que vem a ser uma função contínua, pense na mesma como uma função cujo gráfico não sofre interrupções ou saltos ao longo de seu domínio). Se f for contínua a proposição a seguir é geometricamente evidente. A partir de agora, sempre que nos referirmos a uma função estaremos sempre supondo ser seu domínio um intervalo da reta e a função contínua nesse intervalo.
Proposição 5 : Sejam f : I → R uma função. Então:
i. f é convexa se e só se, para todos x, y em I e todo t∈ [0, 1] tivermos
f ((1 − t ) x + ty ) ≤ (1 − t ) f ( x ) + tf ( y ) ii. f é convexa se e só se, para todos x, y em I e todo t ∈ [0,1] tivermos f ((1 − t ) x + ty ) ≥ (1 − t ) f ( x ) + tf ( y )
Façamos o caso em que f é convexa. O outro caso é análogo. Observe que
(1 − t ) x + ty ∈ [ x, y ] ⊆ I , e que, no trapézio abaixo, (1 − t ) f ( x ) + tf ( y )
EUREKA! N°5, 1999
41
Sociedade Brasileira de Matemática
é o comprimento da paralela às bases pelo ponto (1 − t ) x + ty
f (z)
x
z = ( 1 – t ) x + ty
y
Prova : Suponha primeiro que f satisfaz a condição do item i. Tomando t = 12 concluímos que f é convexa. Reciprocamente, suponha que f seja convexa. Dados x, y em I, temos
f
( ) x +3y 4
x +2 y + y = f 2 ≤
f
( )+ f ( y) ≤ x+ y 2
2
f ( x )+ f ( y ) + 2
f ( y)
2
=
1 4
f ( x) +
3 4
f ( y ) Tro
cando x por y e raciocinando como acima segue que, para
{
}
t ∈ 0, 14 , 12 , 43 ,1 , f ((1 − t ) x + ty ) ≤ (1 − t ) f ( x ) + tf ( y ) (*) Por indução sobre k inteiro positivo podemos concluir de maneira análoga que (*) continua válida para todo t da forma mk , onde 0 ≤ m ≤ 2 k é inteiro. Como todo 2
real em [0,1] é limite de uma seqüência de números dessa forma, segue que (*) continua válida para todo t em [0, 1]. ❏ As afirmações a seguir são agora bastante evidentes, e vão ser nosso principal guia quando quisermos decidir se uma dada função é ou não convexa ou côncava.
EUREKA! N°5, 1999
42
Sociedade Brasileira de Matemática
i. Se para todos a < b em I o gráfico de f entre as retas x = a e x = b estiver abaixo da reta que passa por ( a , f ( a )), ( b, f ( b )) , então f é convexa, e reciprocamente.
ii. Se para todos a < b em I o gráfico de f entre as retas x = a e x = b estiver acima da reta que passa por ( a , f ( a )), ( b, f ( b)) , então f é côncava, e reciprocamente. A figura abaixo para se convencer da validez desse resultado no caso em que f é convexa.
y = f(x)
e
(c, e)
d
(c, d)
a
c
b
x
( )
f ( a ) + f ( b) . Daí, Nele, c = a +2 b . É evidente que d = f ( c ) = f a +2 b e e = 2
f
( a +2 b ) ≤ f (a )+2 f (b) e f é convexa.
Para nós, a importância dessa discussão sobre funções côncavas e convexas reside na seguinte:
Proposição 6 (Desigualdade de Jensen): Sejam I um intervalo da reta e f : I → R uma função. Se x1 ,..., x n ∈ I e t1 ,..., t n ∈ [0,1] , com t1 + ... + t n = 1 , então t1 x1 + ... + t n x n ∈ I e
EUREKA! N°5, 1999
43
Sociedade Brasileira de Matemática
(
)
( )
( )
i. f convexa ⇒ f t1 x1 + ... + t n x n ≤ t1 f x1 + ... + t n f x n
ii. f côncava ⇒ f ( t1 x1 + ... + t n x n ) ≥ t1 f ( x1 ) + ... + t n f ( x n ) Prova : Façamos a prova, por indução sobre n > 1, para o caso em que f é convexa, sendo o outro caso análogo. O caso n = 2 é nossa hipótese. Suponha agora que para um certo n > 1 e todos x1 ,..., x n ∈ I e t1 ,..., t n ∈ [0,1] , com t1 + ... + t n = 1 , tenhamos
(
)
( )
( )
t1 x1 +...+ t n x n ∈ I e f t1 x1 + ... + t n x n ≤ t1 f x1 + ... + t n f x n Considere agora x1 ,..., x n +1 ∈ I e t1 ,..., t n +1 ∈ [0,1] , com t1 + ... + t n +1 = 1 . Se t n +1 = 1 então t1 = ... = t n = 0 e nada há a fazer. Senão, defina
y=
t1 x1 + ... + t n x n 1− t n +1
= s1 x1 + ... + sn x n ,
tj
onde s j = 1− t . Como s1 + ...+ sn = 1 , segue da hipótese de indução que n +1 y ∈ I . Daí,
(
f (t1 x1 + ... + t n +1 x n +1 ) = f (1 − t n +1 )
(
t1 x1 + ... + t n x n 1− t n +1
)+t
=
n +1 x n +1
)
= f (1 − t n +1 ) y + t n +1 x n +1 ≤ (1 − t n +1 ) f ( y ) + t n +1 f ( x n +1 ) , já que f é convexa. Aplicando a outra metade da hipótese de indução, obtemos
f ( y) = f ( s1x1 +... + sn xn ) ≤ s1 f ( x1) + ...+sn f ( xn ) = 1−t1 t
f ( x1) + ...+ 1−tn t
n+1
n+1
f ( xn )
Juntando essas duas desigualdades, obtemos a desigualdade de Jensen. ❏ Vejamos agora um exemplo de como aplicar a desigualdade de Jensen.
Exemplo 5: (Olimpíada Balcânica) Sejam n > 1 e a1 ,..., a n reais positivos com soma a1 1+ b1
1. a
Para
cada
a
+ 1+ 2b + ...+ 1+ bn ≥ 2
n
i,
seja
n 2 n −1 .
EUREKA! N°5, 1999
44
bi =
n
∑
aj .
j =1, j ≠1
Prove
que
Sociedade Brasileira de Matemática
(
)
Prova : Veja que b j = 1 + 1 − a j = 2 − a j , e então temos de provar que a1 2 − a1
+
a2 2− a2
a
+ ...+ 2 − na ≥ n
n 2 n −1
Afirmamos que a função f : (− ∞,2 ) →R dada por f ( x ) = 2 −x x é convexa. Para ver isso, basta escrever f ( x ) = 2 −2 x − 1 , e esboçar o gráfico de f, como abaixo. y
f
2
x
Portanto, temos pela desigualdade de Jensen que n
∑ f (a j ) j =1
≥ n f 1n
n
j =1
∑ a j = nf ( 1n ) =
n 2 n −1
❏
Exemplo 6 : Utilizando a função logaritmo natural e a desigualdade de Jensen, vamos dar outra prova da desigualdade entre as médias aritmética e geométrica. Prova : Sejam a1 ,..., a n reais positivos. Existem reais x1 ,..., x n tais que
a j = ln x j para todo j. Como f ( x ) = ln x é uma função côncava, vem que f ( x1 ) + ... + f ( x n ) n
≤ f
ou seja,
ln n x1 ... x n ≤ ln EUREKA! N°5, 1999
45
(
(
x1 + ... + x n n
x1 + ...+ x n n
)
),
Sociedade Brasileira de Matemática
Como f é crescente, chegamos ao resultado desejado. ❏ Vale notar, para quem tem familiaridade com derivadas, que é possível provar que, se f ' ' existe, então f é convexa se e só se f ' ' ( x ) ≥ 0 para todo x em I e f é côncava se e só se f ' ' ( x ) ≤ 0 para todo x em I. Finalizamos este artigo com alguns problemas onde procuramos oferecer oportunidade de exercitar o que foi aprendido no texto, além de desenvolver um pouco mais a teoria. É bom salientar que em alguns deles mais de uma desigualdade pode ser usada.
Problemas onde não precisamos das desigualdades acima, mas de criatividade 1. (Olimpíada Americana): Prove que, para todos a, b, c reais positivos, temos 1 a 3 + b 3 + abc
+
1 b 3 + c 3 + abc
+
1 c 3 + a 3 + abc
≤
1 abc
2. (Desigualdade de Abel): Sejam a1 , ..., a n , b1 ,..., bn reais dados (n > 1), com a1 ≥ a 2 ≥ ... ≥ a n ≥ 0 . Se M e m são respectivamente o máximo e o mínimo do conjunto {b1 , b1 + b2 ,..., b1 + ... + bn } , prove que
ma1 ≤ a1b1 + a2 b2 + ...+ a n bn ≤ Ma1 ,
com igualdade se e só se a1 = a 2 = ... = a n .
3. (Teste de Seleção de Singapura para IMO): Prove que, quaisquer que sejam os reais positivos a, b e c, tem-se
c a 2 − ab + b 2 + a b 2 − bc + c 2 ≥ b a 2 + ac + c 2 . 4. (Banco IMO): Sejam n > 1 um inteiro dado. Determine o maior valor possível da soma
∑
(
)
x i x j x i + x j sobre todas as n-uplas x1 ,..., x n de reais
1≤ i < j ≤ n
não negativos cuja soma é 1.
EUREKA! N°5, 1999
46
Sociedade Brasileira de Matemática
Desigualdade entre as médias aritmética e geométrica 5. (Torneio das Cidades) Sejam a, b e c reais positivos dados. Prove que a3 a + ab + b 2 2
+
b3 b + bc + c 2 2
+
c3 c + ca + a 2
a + b+ c 3
≥
2
6. Dados os reais positivos a1 , a2 , a3 , b1 , b2 , b3 , prove que 3
( a1 + b1 )( a2 + b2 )( a3 + b3 )
≥
3
a1a2 a3 + 3 b1b2 b3
Desigualdades de Chebychev, Jensen e Cauchy
7. (Olimpíada Turca) Sejam n > 1 inteiro e x1 ,..., x n reais positivos tais n
que
∑ xi2 = 1 .
Determine
o
valor
mínimo
de
i =1
x15 x 2 + x 3 + ...+ x n
+
x 25 x1 + x 3 + ...+ x n
x n5 1 + x 2 + ...+ x n −1
+ ...+ x
.
8. (Olimpíada Romena): Seja h a altura de um tetraedro regular e h1 , h2 , h3 , h4 as distâncias desde um ponto P em seu interior às faces do tetraedro. Prove que h − h1 h + h1
+
h − h2 h + h2
h − h3 h + h3
+
+
h − h4 h + h4
≥ 12 5
9. (Banco IMO) : Sejam a, b, c, d reais não negativos tais que ab + bc + cd + da = 1 . Prove que a3 b+c + d
+
b3 c+d +a
+
c3 d +a+b
+
d3 a + b+ c
≥
1 3
10. Sejam n > 1 e x1 , x 2 ,..., x n reais positivos cuja soma é 1. Prove que x1 1− x1
+ ...+
xn 1− x n
≥
n n −1
≥
x1 + ...+ x n n −1
11. Sejam x1 , x 2 ,..., x n reais pertencentes ao intervalo [0, 1] e tais que x1 + x 2 + ... + x n = a , com 0 < a < 1 . Prove que EUREKA! N°5, 1999
47
Sociedade Brasileira de Matemática
1− a 1+ a
≤
1− x1 1− x 2 1+ x1 1+ x 2
...
1− x n 1+ x n
≤
( nn −+ aa ) n
Outras Desigualdades
12. (Desigualdade de Bernoulli): Sejam n um inteiro positivo e x ≥ −1 um n real. Prove que (1 + x ) ≥ 1 + nx . 13. (Desigualdade entre as Médias de Potências): Sejam α < β reais positivos. Então, para todos a1 , a2 ,..., a n reais positivos, vale 1
1
β β a1α + a 2α + ... + a nα α a1 + a 2 + ... + a nβ β ≤ , n n com igualdade se e só se a1 , a2 ,..., a n forem todos iguais.
14. (Desigualdade de Giroux): Sejam I1 ,..., I n intervalos fechados da reta
e considere a função f :I 1 ×...×I n →R de n variáveis, convexa separadamente em relação a cada variável. Então, se I j = [a j , b j ] , f atinge seu valor máximo em um dos 2 n pontos da forma ( c1 ,..., cn ) , com ci = a i ou bi para cada i. Prove isto e enuncie um resultado análogo à desigualdade de Giroux para uma função de n variáveis f, côncava separadamente em relação a cada variável.
15. (Olimpíada Búlgara): Sejam n ≥ 2 um inteiro e 0 ≤ x i ≤ 1 para 1≤ i ≤ n. Prove que
( x1 + x 2 +...+ x n ) − ( x1 x 2 + x 2 x3 +...+ x n −1 x n + x n x1 ) ≤ n2 16. Os três itens a seguir visam derivar uma desigualdade difícil. i. (Desigualdade de Young): Sejam p e q reais positivos tais que 1p + 1q = 1 . Prove que
xy ≤
xp p
+
yq q
, ∀ x, y ≥ 0
ii. (Desigualdade de Holder): Sejam a1 , a2 ,..., a n , b1 , b2 ,..., bn reais EUREKA! N°5, 1999
48
Sociedade Brasileira de Matemática
positivos e p, q reais positivos tais que 1p + 1q = 1 . Prove que
n a i bi ≤ a ip i =1 i =1 n
∑
∑
1/ p
n q bi i =1
∑
1/ q
iii. (Desigualdade de Minkowsky): Sejam a1 , a2 ,..., a n , b1 , b2 ,..., bn reais positivos e p um real maior que 1. Prove que p
(a1 + b1 ) p +...+(a n + bn ) p ≤
p
a1p +...+ a np + p b1p +...+ bnp
Sugestão: Faça ci = (a i + bi ) p −1 e use o ítem anterior para (ai) e (ci) e para (bi) e (ci).
Referências: [1] Shklarsky, D. O., Chentzov, N. N. e Yaglom, I. M. The USSR Olympiad Problem Book. Dover. Toronto, 1993. [2] Rousseau, C. e Lozansky, E. Winning Solutions. Springer-Verlag, 1996. [3] Lima, Elon L., Análise Real, vol. 1. IMPA, 1995.
EUREKA! N°5, 1999
49
Sociedade Brasileira de Matemática
40a. OLIMPÍADA INTERNACIONAL E 14a. OLIMPÍADA IBEROAMERICANA DE MATEMÁTICA Primeiro teste de Seleção PROBLEMA 1
Determine todos os inteiros positivos n > 1 para os quais existem um inteiro positivo k inteiros x1, x2, …xn dois a dois distintos tais que o conjunto {xi + x j ; 1 ≤ i < j ≤ n} Seja um conjunto de potências distintas de k. Observação: x1, x2, …xn não são necessariamente positivos. PROBLEMA 2
Sejam a, b, c, d números reais tais que
a = 4 − 5 − a , b = 4 + 5 − b , c = 4 − 5 + c e d = 4 + 5 + d . Calcule abcd. PROBLEMA 3
Considere um triângulo ABC e BD e CE as bissetrizes dos ângulos B e C, respectivamente ( D ∈ AC e E ∈ AB). A circunferência circunscrita a ABC tem centro O e a circunferência ex-inscrita tangente ao lado BC tem centro Ia. Estas duas circunferências intersectam-se nos pontos P e Q. Mostre que PQ é paralelo a DE. (i) Prove que IaO é perpendicular a DE. (ii) PROBLEMA 4
Sejam Q+ e Z o conjunto dos racionais extritamente positivos e o conjunto dos inteiros. Determine todas as funções f : Q+ → Z satisfazendo as seguintes condições: (1) f (1999) = 1 (2) f(ab) = f (a) + f(b), para quaisquer a, b ∈ Q+. (3) f(a + b) ≥ min{f(a), f(b)}, para quaisquer a, b ∈ Q+. A notação min{x, y} denota o menor dentre os inteiros x e y. Por exemplo, min{3, 4} = 4 e min{3, 3} = 3. PROBLEMA 5
(i) (ii)
Se m, n são inteiros positivos tais que 2n – 1 divide m2 + 9, prove que n é uma potência de 2. Se n é uma potência de 2, prove que existe um inteiro positivo m tal que 2n – 1 divide m2 + 9.
EUREKA! N°5, 1999
50
Sociedade Brasileira de Matemática
SOLUÇÕES DE PROBLEMAS PROPOSTOS Publicamos aqui algumas das respostas enviadas por nossos leitores.
21)
a + b = c.
a) Encontre todas as soluções inteiras da equação b) Encontre todas as soluções inteiras da equação
3
a + 3 b = 3 c.
Solução de Carlos Alberto da Silva Victor. a) Vamos separar em 2 etapas: i)
Tomando a = k2 ; b = t2 teremos c = (k + t)2 consequentemente (k2, t2, (k + t)2) é solução da equação para todo k, t ∈ N.
ii)
Suponha que a não seja um quadrado perfeito. Neste caso, a pode ser escrito na forma a = k2s, onde s é um produto de primos distintos. Como c = a + b + 2 ab , devemos ter b = t2 . s (t ∈ N) já que ab deverá ser um quadrado perfeito (observe que se tomarmos s com exponente ímpar diferente de 1, podemos incluí-lo em t2); Daí: c = a + b + 2 ab = k 2 s + t 2 s + 2kts
c = s (k 2 + t 2 + 2kt ) c = s. (k + t ) 2 e neste caso ( k 2 s, t 2 s, s (k + t ) 2 ) é solução da equação onde k , s, t , ∈ N. Nota: observe que o ítem (ii) representa a situação genérica e inclui as soluções de (i), fazendo s = 1. b) i)
a + 3 b = 3 c , logo c = a + b + 3(3 a 2 b + 3 ab 2 ). Vamos separar também em 3 etapas: Tomando a = k3; b = t3 teremos c = (k + t)3 com k, t ∈ N e 3
consequentemente (k 3 , t 3 , (k + t ) 3 ) é solução da equação. ii)
Suponha que a não seja um cubo perfeito. Observe que a pode ser escrito na forma: a = t 3 . α . β 2 onde α e β são produtos de primos distintos. Já que c = a + b + 3 (3 a 2 b + 3 ab2 ), devemos tomar
EUREKA! N°5, 1999
51
Sociedade Brasileira de Matemática
b = t 3 . α . β 2 pois a 2 = k 6 . α 2 β 4 (de fato,
3
a 2 b + 3 ab 2 é racional, e,
elevando ao quadrado, obtemos que a 3 ab 2 + 2ab + b3 a 2 b (e logo
a 3 ab 2 + b3 a 2 b ) também é racional, donde, assumindo a ≠ b, 3
ab 2 e
3
a 2 b são racionais e inteiros). daí:
c = k 3 . α . β 2 + t 3α . β 2 + 3( k 2 t ⋅ α . β 2 + k . t 2 . α . β 2 ) ∴
c = α . β 2 ( k 3 + t 3 + 3k 2 t + 3kt 2 ) Logo (k 3 . α . β 2 , t 3α . β 2 , α . β 2 (k + t ) 3 ) é solução da equação dada. (Novamente, as soluções em (i) são obtidas de (ii) com α = β = 1.
22)
Sejam α , β , γ , δ os ângulos de um quadrilátero, nessa ordem. Prove que esse quadrilátero é inscritível se, e somente se, a relação αβ + αδ + γβ + γδ = π 2 ocorre.
Solução de André Luiz Arruda Marques. A α
D
δ
β
B
γ C
1a. parte:
Hipótese: O quadrilátero é inscritível. Tese: É válida a relação αβ + αδ + γβ + γδ = π 2
∩ ∩ BCD BAD + = 360° Sabe-se que : ∩ BCD = 2α ⇒ 2α + 2γ = 360° = 2π ⇒ α + γ = π (1) ∩ BAD = 2γ
EUREKA! N°5, 1999
52
Sociedade Brasileira de Matemática ∩ ∩ ABC ADC + = 360° ∩ ABC = 2α ⇒ 2γ + 2 β = 360° = 2π ⇒ δ + β = π ( 2) ∩ ADC = 2 β
de (1) e (2) ⇒ (α + γ )( β + δ ) = π 2 ⇒ αβ + αδ + γβ + γδ = π 2 (c.q.d)
2a. parte:
Hipótese: É válida a relação αβ + αδ + γβ + γδ = π 2 Tese: O quadrilátero é inscritível
Têm-se a relação: αβ + αδ + γβ + γδ = π 2 Fatorando vem: (α + γ )( β + δ ) = π 2 = π ⋅ π Sabe-se que: α + β + γ + δ = 360° = 2π
xy = π 2 ⇒ x = y =π Seja: α + γ = x e β + δ = y ⇒ x + y = 2π α + γ = π Logo: ∴ o quadrilátero é inscritível (c.q.d.). β + δ = π 23)
Seja ABC um triângulo qualquer de ortocentro H e sejam ha, hb, hc os comprimentos das alturas relativas a A, B, C respectivamente. Prove que ____ ____ ____ 1 ha . AH + hb . BH + hc CH = (a 2 + b 2 + c 2 ). 2
Solução de Maria Ivete Caetano Rodrigues. A z O N
y
H
M C
B
x
EUREKA! N°5, 1999
53
Sociedade Brasileira de Matemática ____
ha = AM
____
____
____
____
CM = x
____
Notações: hb = BN ____
AB = c
BO = y
BC = a
____
____
AN = z
hc = CO
AC = b ___
____
Temos que : ∆CMH é retângulo ⇒ (CH ) 2 = (ha − AH ) 2 + x 2 2
∆ACM é retângulo ⇒ b =
ha2
2
2
2
+ x ⇒ x =b −
___
ha2
____
I II
____
substituindo II em I, temos (CH ) 2 = ha2 − 2ha AH + ( AH ) 2 + b 2 − ha2 ___
____
____
( CH ) 2 = b 2 + ( AH ) 2 − 2ha AH ____
(1) ____
____
Analogamente temos: ( BH ) 2 = ( CH ) 2 + a 2 - 2hc CH ____
____
____
( AH ) 2 = ( BH ) 2 + c 2 - 2hb BH
(2) e
(3)
somando (1), (2) e (3), obtemos: ___
____
____
2ha AH + 2hc CH + 2hb BH = a 2 + b 2 + c 2 ___
____
____
2(ha AH + 2hb BH + hc CH ) = a 2 + b 2 + c 2 ___
____
____
ha AH + hb BH + hc CH = 25)
a2 + b2 + c2 (c.q.d.) 2
Durante o ano de 1998, uma pequena livraria, que abria nos sete dias da semana, vendeu no mínimo um livro por dia e um total de 600 livros no ano todo. Diga, justificando, se existiu, obrigatoriamente, um período de dias consecutivos onde foram vendidos exatamente 129 livros.
Solução de Marcelo Rufino de Oliveira. Seja ai o total acumulado de livros vendidos até o final do i-ésimo dia. Por exemplo: a5 = total vendidos até o quinto dia a35 – a31 = total de livros vendidos entre os dias 35 e 32 (inclusive). Então: a1 < a2 < a3 < … < a364 < a365 = 600 (1) EUREKA! N°5, 1999
54
Sociedade Brasileira de Matemática
Resta agora analisar se existem ai, aj (i, j ∈ N menores que 365, j > i) tais que aj – ai = 129 Somemos agora a cada termo de (1) o valor 129: a1 + 129 < a2 + 129 < a3 + 129 < … < a364 + 129 < a365 + 129 = 729 Chamemos estes termos de bi (bi = ai + 129): b1 < b2 < b3 < … < b364 < b365 = 729 Temos então 730 termos, 365 termos ai e 365 termos bi, com ai ≠ aj (i ≠ j) e bp ≠ bk (p ≠ k). Notemos que estes 730 termos naturais estão entre 1 e 729, ou seja, existem dois valores iguais (princípios das casa dos pombos ou Princípio das Gavetas de Dirichet) entre estes 730 termos. Como cada ai é distinto e cada bi é distinto, então existe um am que é igual a um bn: am = bn => am = an + 129 => am – an = 129 Que prova que existe um período de dias consecutivos (m – n dias) onde foram vendidos exatamente 129 livros. PROBLEMA No. 9 DO VISITANTE MATEMÁTICO (Revista EUREKA! No. 2)
Um vaso de vinho está suspenso sobre outro, de igual capacidade (digamos 1 litro), cheio de água. Por um orifício no fundo de cada vaso, o vinho escorre sobre o vaso de água e a mistura se esvai na mesma velocidade. Quando o vaso de vinho estiver vazio, qual é o volume de água no vaso inferior?
Solução de Carlos Frederico Borges Palmeira. Aqui está uma solução, que usa apenas a noção de limite e o fato que 1 1 lim(1 − ) n = . n e Vamos discretizar o problema, supondo que primeiro deixamos passar uma fração 1 de um vaso para o outro, a água e o vinho se misturam, e depois retiramos n 1 do novo volume do segundo vaso. Após n passos o primeiro vaso estará vazio. n Seja v(k + 1) a quantidade de vinho no segundo vaso após o k-ésimo passo.
1 1 Temos v( k + 1) = (1 − ) (v(k ) + ), sendo v (0) = 0. n n EUREKA! N°5, 1999
55
Sociedade Brasileira de Matemática
Agora,
a
equação
de
diferenças
v(k + 1) = av( k ) + b
tem
solução
n
1− a ) ⋅ b (nada além de progressões geométricas aqui). 1− a 1 (1 − (1 − ) n 1 n n ) ⋅ 1 (1 − 1 ), e como v (0) = 0, Segue que v( n) = (1 − ) ⋅ v(0) + ( 1 n n n n 1 1 temos apenas o segundo termo, que fica (1 − ) (1 − (1 − ) n ) , e passando ao n n 1 limite quando n tende ao infinito, obtemos 1 − ( ). Esta é a fração de vinho que e resta no segundo vaso quando o primeiro fica vazio. v( k ) = a n ⋅ v(0) + (
Você sabia…
que todo poliedro convexo (com
as faces rígidas) é rígido (isto é, não pode ser deformado) mas existem poliedros não - convexos flexíveis?
Errata: No Você sabia… da EUREKA! No. 4, página 21, sobre um polinômio que assume apenas valores negativos ou primos, há um erro tipográfico na terceira linha da expressão do polinômio: onde está "…[ e3 . (e + 2) . (a +1)2 + …" leia-se "…[e3 . (e + 2) . (a + 1) 2 + …". Agradecemos também o envio das soluções da EUREKA! No.4 a: Osvaldo Ribeiro da Silva Júnior, Robério Bacelar da Silva, Manuel João de Jesus Almeida, Vicente Wilson Moura Gaeta, Rubens Henriques, Raul Rabello Mesquita, Otávio Braga, Antonio Caminha Neto, Alexandre Celestino Leite de Almeida, Luciana Rocha Pedro, Francisco Dutenhefner, Seme Gebara Neto. Continuamos esperando as soluções dos problemas 10, 16, 17, 20 e 24.
EUREKA! N°5, 1999
56
Sociedade Brasileira de Matemática
PROBLEMAS PROPOSTOS Convidamos o leitor a enviar soluções dos problemas propostos e sugestões de novos problemas para os próximos números.
26)
Sejam as funções fo (x) = xn e fi (x) = fi – 1(x + 1) – fi – 1(x) onde x, n e i são inteiros positivos. Prove que, para todo x, fn(x) = n!
27)
O triângulo equilátero ABC possui um ponto interno P tal que em P chegam três segmentos de reta (PA, PB, PC) onde PA = 6, PB = 8 e PC = 7. Com esses dados descubra qual é a área do triângulo. A
P B
28)
Seja n ≥ 2 um número inteiro. Prove que n e n + 2 são ambos primos se e somente se
29)
C
4((n − 1)! + 1) + n é inteiro. n ( n + 2)
Seja n > 1 um número inteiro. Existem n lâmpadas L0, L1, ... , Ln–1 colocadas em um círculo. Cada lâmpada está ACESA ou APAGADA. Uma seqüência de passos S0, S1, ... , Si, ... é executada. O passo Sj afeta apenas o estado da lâmpada Lj (deixando o estado de todas as outras inalterado) da seguinte forma: Se Lj–1 está ACESA, Sj muda o estado de Lj de ACESA para APAGADA, ou de APAGADA para ACESA; Se Lj–1 está APAGADA, Sj deixa o estado de Lj inalterado. As lâmpadas são rotuladas mod n, ou seja,
EUREKA! N°5, 1999
57
Sociedade Brasileira de Matemática
L – 1 = Ln –1, L0 = Ln, L1 = Ln + 1, etc. Inicialmente todas as lâmpadas estão ACESAS. Mostre que: a.
Existe um inteiro positivo M(n) tal que depois de M(n) passos todas as lâmpadas estão ACESAS de novo;
b.
Se n é da forma 2k então todas as lâmpadas estão ACESAS depois de n2 –1 passos;
c.
Se n tem a forma 2k + 1 então todas as lâmpadas estão ACESAS depois de n2 – n + 1 passos.
O problema 6 da Olimpíada Internacional de Matemática de 1994 pedia que fosse mostrada a existência de um conjunto A de inteiros positivos com a seguinte propriedade: para todo conjunto infinito de números primos S existem um inteiro positivo k ≥ 2 e dois inteiros positivos m ∈ A e n ∉ A, cada um dos quais é um produto de k elementos distintos de S. Os leitores que enviarem soluções corretas (até o dia 15 de novembro de 1999) para o seguinte problema concorrerão a um exemplar do livro "10 Olimpíadas Iberoamericanas de Matemática".
PROBLEMA "CUÁTICO": Prove que no problema acima k não pode ser escolhido independentemente do conjunto S. Mais precisamente: Prove que para qualquer conjunto de inteiros positivos A e para todo inteiro positivo k existe um conjunto infinito de números primos S tal que o produto de k elementos distintos de S está sempre em A ou o produto de k elementos distintos de S nunca pertence a A.
Nota: Os problemas 26 e 27 foram propostos por Christian Lyoiti Watanabe e Roberto Gomides respectivamente. EUREKA! N°5, 1999
58
Sociedade Brasileira de Matemática
COMO ASSINAR A EUREKA! Se você é fanático por Matemática e deseja receber na sua casa a revista EUREKA!, faça o seu pedido escrevendo para: Secretaria da Olimpíada Brasileira de Matemática, Estrada Dona Castorina, 110 Jardim Botânico - Rio de Janeiro, RJ - CEP: 22460-320. O custo de cada exemplar avulso ou atrasado é de R$4,00. Você pode fazer uma assinatura anual o que dará direito a receber as publicações do ano (mínimo 3 exemplares) por um valor promocional de R$10,00. Para isso, faça um depósito no Banco do Brasil - Agência 0598-3 Conta N°52208-2 em nome do professor Eduardo Wagner. Envie-nos a fotocopia do depósito e faça referência aos números desejados. Não esqueça de colocar seu nome e endereço completos e nós remeteremos a(s) revista(s) pelo correio. Pedidos podem ser feitos também por e-mail e comprovantes de depósito poderão ser enviados pelo fax.
Se tiver qualquer dúvida entre em contato conosco. Telefone: 021-5295077 / Fax: 021-5295023 e-mail: obm@impa.br Home-Page: http//www.obm.org.br/
Revista do Professor de Matemática A RPM é uma publicação da SBM destinada aos professores de Matemática do ensino médio, que pretende ser um veículo de circulação e intercâmbio de idéias através de seus artigos e seções. Revista do Professor de Matemática Caixa Postal 66281 CEP 05315-970 São Paulo – SP e-mail: rpm@ime.usp.br Fone/Fax: (11) 818-6124
Estamos comemorando o número 40 da RPM e 17 anos de publicação sem interrupção, com capa nova e com renovado estímulo para continuar auxiliando o professor de Matemática.
EUREKA! N°5, 1999
59
Sociedade Brasileira de Matemática
AGENDA OLÍMPICA
21a. OLIMPÍADA BRASILEIRA DE MATEMÁTICA Primeira Fase – 12 de junho (sábado) Segunda Fase – 28 de agosto (sábado) Terceira Fase – 23 de outubro (sábado) e 24 de outubro (domingo) ♦
14a. OLIMPÍADA IBEROAMERICANA DE MATEMÁTICA 12 a 19 de setembro La Habana, Cuba. ♦
2a. OLIMPÍADA IBEROAMERICANA DE MATEMÁTICA UNIVERSITÁRIA 16 de setembro
Você sabia…
Que se an + 1 é primo (com a e n naturais) então n = 2k para algum k natural? Tente provar isso!, (primos desta forma são conhecidos como primos de Fermat generalizados.) E que se a = 2 só se conhecem 5 primos desta forma (3, 5, 17, 257 e 65537), os chamados primos de Fermat? E que, por outro lado, se conhecem muitos primos de Fermat generalizados grandes (como 10183016384 + 1 e 6723416384 + 1, respectivamente o 16o e 18o. maiores primos conhecidos em 27/7/99)?
EUREKA! N°5, 1999
60
Sociedade Brasileira de Matemática
COORDENADORES REGIONAIS Amarisio da Silva Araújo Alberto Hassen Raad Antônio C. Rodrigues Monteiro Angela Camargo Benedito T. Vasconcelos Freire Claudio Arconcher Élio Mega Florêncio F. Guimarães F. Francisco Dutenhefner Gisele de A. Prateado G. Ivanilde H. Fernandes Saad João B. de Melo Neto João F. Melo Libonati Jorge Ferreira José Carlos Pinto Leivas José Clovis Saraiba José Luis Rosas Pinho José Paulo Carneiro José Vieira Alves Leonardo Matteo D'orio Licio Hernandes Bezerra Luzinalva M. de Amorim Marco Polo Marcondes Cavalcante França Pablo Rodrigo Ganassim Paulo H. Cruz Neiva de L. Jr. Reinaldo Gen Ichiro Arakaki Ricardo Amorim Roberto Vizeu Barros Sergio Claudio Ramos Seme Gebara Neto Tadeu Ferreira Gomes Tomás Menéndez Rodrigues Valdenberg Araújo da Silva Wagner Pereira Lopes Waldemar M. Canalli
(UFV) Viçosa - MG (UFJF) Juiz de Fora - MG (UFPE) Recife - PE (Centro de Educação de Adultos - CEA) Blumenau - SC (UFRN) Natal - RN (Col. Leonardo da Vinci) Jundiaí - SP (Col. ETAPA) São Paulo - SP (UFES) Vitória - ES (UFMG) Belo Horizonte - MG (UFGO) Goiânia - GO (U. Católica Dom Bosco) Campo Grande - MS (UFPI) Teresina - PI (Grupo Educ. IDEAL) Belém - PA (UEM) Maringá - PR (UFRG) Rio Grande - RS (UFMA) São Luis - MA (UFSC) Florianópolis - SC (USU) Rio de Janeiro - RJ (UFPB) Campina Grande - PB (Sistema Titular de Ensino)Belém - PA (UFSC) Florianópolis - SC (UFBA) Salvador - BA (Colégio Singular) Santo André - SP (UF Ceará) Fortaleza - CE (L. Albert Einstein) Piracicaba - SP (Esc. Tec.Everardo Passos) SJ dos Campos - SP (INPE) SJ dos Campos - SP (Centro Educ. Logos) Nova Iguaçu - RJ (Colégio ACAE) Volta Redonda - RJ (IM-UFRGS) Porto Alegre - RS (UFMG) Belo Horizonte - MG (U. do Estado da Bahia) Juazeiro - BA (U. Federal de Rondonia) Porto Velho - RO (U. Federal de Sergipe) São Cristovão - SE (Esc. Tec. Fed. de Goiás) Jataí - GO (P.M. S. João de Meriti) S. João de Meriti - RJ
EUREKA! N°5, 1999
61
CONTEÚDO 40a. OLIMPÍADA INTERNACIONAL DE MATEMÁTICA Problemas e Soluções 14a. OLIMPÍADA IBEROAMERICANA DE MATEMÁTICA Prova
2 13
ARTIGOS CONTAR DE DUAS MANEIRAS, PARA GENERALIZAR José Paulo Q. Carneiro - Universidade Santa Úrsula
15
A MATEMÁTICA DAS ABELHAS José Cloves Saraiva - Universidade Federal de Maranhão
18
O TEOREMA DE RAMSEY Carlos Gustavo T. de A. Moreira - IMPA
23
APLICAÇÕES DOS NÚMEROS COMPLEXOS À GEOMETRIA Edmilson Motta - Colégio Etapa
30
40a. OLIMPÍADA INTERNACIONAL E 14a. OLIMPÍADA IBEROAMERICANA DE MATEMÁTICA Segundo teste de Seleção
39
10a. OLIMPÍADA DE MATEMÁTICA DO CONE SUL Primeiro teste de Seleção
40
10a. OLIMPÍADA DE MATEMÁTICA DO CONE SUL Segundo teste de Seleção
41
SOLUÇÕES DE PROBLEMAS PROPOSTOS
42
PROBLEMAS PROPOSTOS
57
ASSINATURA DA REVISTA EUREKA! 2000
59
FICHA DE CADASTRAMENTO ASSINATURA DA REVISTA EUREKA!
60
COORDENADORES REGIONAIS
61
Sociedade Brasileira de Matemática
40a. OLIMPÍADA INTERNACIONAL DE MATEMÁTICA Problemas e Soluções Primeiro Dia Duração da prova: 4 horas e 30 minutos PROBLEMA 1
Determine todos os conjuntos finitos S de pontos do plano com pelo menos três elementos que satisfazem a seguinte condição: Para quaisquer dois pontos distintos A e B de S, a mediatriz do segmento AB é um eixo de simetria de S.
Solução de Fabrício Siqueira Benevides (Fortaleza - CE): Seja α o fecho convexo dos pontos de S, isto é conjunto de pontos da periferia de S que formam um polígono convexo que contém todos os pontos de S. Sejam P1 , P2 ,..., Pn os vértices do polígono formado pelo fecho convexo.
P3'
Pn
Pn'
P1
P3
P2 ______
Afirmação 1) P3 é o simétrico de Pn em relação à mediatriz de P1 P2 que ______
denotaremos por md( P1 P2 ). ______
Prova: Suponha o contrário. Seja Pn' o simétrico de Pn a md( P1 P2 ). Pn' deve estar no interior do polígono P1 , P2 ,..., Pn , pois este é o fecho convexo. EUREKA! N°6, 1999
2
Sociedade Brasileira de Matemática ∧
∧
______
Daí, P1 P2 Pn ' < P1 P2 P3 , e se P3 ' é o simétrico de P3 à md( P1 P2 ) teremos ∧
∧
P1 P2 P3 ' > P2 P1 Pn (pela simetria), aí P3 ' pertenceria a S e estaria fora do fecho convexo. Absurdo. Se Pn ' está sobre o segmento P2P3, ∧
∧
↔
______
______
teremos Pn P1 P2 = P1 P2 P3 ⇒ P3 ' ∈ P1 Pn e P1 P3 ' > P1 Pn ⇒ P3 ' está fora do fecho convexo (Absurdo). Se Pn' está sobre o prolongamento de P2P3, já temos um absurdo pois Pn' estaria fora do fecho convexo. Logo, P3 é realmente o simétrico de Pn. Assim
P3
______
_____
é
o
simétrico ∧
de
Pn
em
relação
______
à
md( P1 P2 ),
donde
∧
P1 Pn = P 2 P 3 ( e Pn P1 P2 = P1 P2 P3 ).
Analogamente, se olharmos para a mediatriz de ______
______
∧
______
P2 P3
concluimos que
∧
P1 P2 = P3 P4 ( e P1 P2 P3 = P2 P3 P4 ).
Se continuarmos a olhar para as mediatrizes de Pi Pi +1 , i = 1,..., n − 1, é facil concluir que: todos os ângulos internos de P1 , P2 ,..., Pn são iguais e: ______
_____
_____
P1 P2 = P 3 P 4 = P5 P6 = ... = P2 k +1 P2 ( k +1)
e
______
_____
_____
P2 P3 = P 4 P 5 = P6 P7 = ... = P2 q P2 q +1
(ver indices módulo n.) Temos agora 2 casos: i)
n é ímpar n = 2k + 1, teremos: ______
_____
________
______
_____
________
P1 P2 = P 3 P 4 = ... = P2 k +1 P1 = P2 P3 = P 4 P5 = ... = P2 k P2 k +1 ; ou fecho convexo é um polígono regular ii)
seja
o
n é par: n = 2k. ______
_____
_________
P1 P2 = P 3 P 4 = P2 k +1 P2 k
e
______
_____
____________
P2 P3 = P 4 P 5 ... = P2 k − 2 P2 k −1
mas md (P1P3) também é eixo de simetria e é fácil ver que o simétrico de ______
P2 está sobre a mediatriz de P1 P3 (e seu simétrico será ele mesmo). EUREKA! N°6, 1999
3
Sociedade Brasileira de Matemática ______
______
Nesse caso P1 P2 = P2 P3 , donde novamente o fecho convexo forma um polígono regular. Portanto, o fecho convexo é um polígono regular. Para finalizar, provaremos que não pode haver qualquer ponto em S além de P1 , P2 ,..., Pn . Já vimos que P1 P2 ...Pn é regular. Seja O o centro da circunferência circunscrita a P1 P2 ...Pn . (pode ser O ∈ S ou O ∉ S). Seja P um ponto de S diferente de P1 ...Pn . (P esta sobre algum lado ou no interior de P1 ...Pn ). Seja Pi o vértice do nosso polígono cuja distância a P seja mínima. E seja _____
r = md ( PPi ). É claro que r não passa por O, (que é o centro de simetria de P1 ...Pn . ), assim se d1 é a maior distância de um vértice Pj que está no semi-plano esquerdo de r a r; e d2 é a maior distância de um vértice Pk que está no semiplano direito de r a r; então d1 ≠ d2. Suponha d1 > d2. Então o simétrico de Pj deveria pertencer a S mas está fora do fecho convexo. Absurdo. Na verdade, quero dizer que se r não é um eixo de simetria de P1 ...Pn , tomamos P1 ' ,..., Pn ' os simétricos de P1 ,..., Pn em relação a r, um dos Pi' estará fora do polígono P1 ...Pn . Conclusão: Um conjunto finito de pontos S satisfaz o problema somente se e só se é o conjunto dos vértices de um dado polígono regular. PROBLEMA 2
Seja n ≥ 2 um inteiro fixo. Determinar a menor constante C para a qual a desigualdade a)
∑
x i x j ( xi2 + x 2j ) ≤ C (
1≤ i < j ≤ n
b)
∑x ) i
4
1≤i ≤ n
é válida para quaisquer números reais x1, …, xn ≥ 0. Para esta constante C, determine quando ocorre a igualdade.
Solução: Consideremos inicialmente o caso n = 2. Queremos achar a menor constante C tal que x1 x 2 ( x12 + x 22 ) ≤ C ( x1 + x 2 ) 4 para quaisquer x1 , x 2 ≥ 0. EUREKA! N°6, 1999
4
Sociedade Brasileira de Matemática
1 Para x1 = x 2 = 1 temos x1 x 2 ( x12 + x 22 ) = ( x1 + x 2 ) 4 , donde necessariamente 8 1 1 C ≥ . Vamos mostrar que C = . Para isso, basta mostrar que 8 8
1 x1 x 2 ( x12 + x 22 ) ≤ ( x1 + x 2 ) 4 para x1 , x 2 ≥ 0. 8 Como ( x1 + x 2 ) 4 = x14 + 4 x1 x 2 ( x12 + x 22 ) + 6 x12 x 22 + x 24 , isso equivale a mostrar x14 + x 24 + 6 x12 x 22 ≥ 4 x1 x 2 ( x12 + x 22 ),
que
mas isso segue diretamente de 1 ( x1 − x 2 ) 4 ≥ 0. Para n > 2 temos ainda C ≥ , pois fazendo x1 = x 2 = 1 e x k = 0 8 1 para k ≥ 3 vale a igualdade com C = . 8
1 Mostraremos que ainda vale a desigualdade do enunciado para C = . Para isso, 8 mostraremos o seguinte: Lema: Se x1 ,..., x n ≥ 0 e o número de termos xi > 0 com 1 ≤ i ≤ n é m ≥ 3 então ~
~
~
podemos substituir x1 ,..., x n por x1 , x 2 ,..., x n tais que ~
∑x = ∑x , ∑ i
i
1≤ i ≤ n
1≤ i ≤ n
xi x j ( xi2 + x 2j ) <
1≤ i ≤ j ≤ n
∑
~
~
~
~
xi x j ( xi2 + x 2j ) e o número de termos
1≤ i ≤ j ≤ n
~
xi > 0 com 1 ≤ i ≤ n é m − 1. Prova do Lema: Sejam x r e x s , r ≠ s os dois menores termos não nulos dentre
x1 , x 2 ..., x n .
Definimos
Obviamente temos i
2 j ( yi
~
~
∑x = ∑x i
1≤ i ≤ n
∑y y
~
xr = xr + xs , xs = 0 i
e
~
xk = xk
para
k ∉{r , s}.
~
e há m – 1 termos xi = 0. Note agora que
1≤ i ≤ n
+ y 2j ) = y r y s ( y r2 + y s2 ) + ( y r3 + y s3 )
1≤ i ≤ j ≤ n
∑
k∉{r , s}
EUREKA! N°6, 1999
5
yk + ( yr + ys )
∑y
k∉{r , s}
3 k,
Sociedade Brasileira de Matemática
donde
∑
~
~
~
~
xi x j (xi2 + x 2j ) −
1≤i ≤ j ≤n
∑ x x (x i
j
2 i
+ x 2j ) = −xr xs (xr2 + xs2 ) + ((xr + xs ) 3 −(xr3 + xs3 ))
1≤i < j ≤n
= x r x s (3( x r + x s )
∑x
k
∑x
k
=
k∉{r ,s}
− ( x r2 + x s2 )), mas como xr e xs são os dois menores
k∉{r , s}
termos não nulos dentre os xi (e há pelo menos 3), temos x + xs 3 xk ≥ r ⇒ 3( x r + x s ) x k ≥ ( x r + x s ) 2 > x r2 + x s2 . 2 2 k∉{r , s} k∉{r , s}
∑
∑
Para concluir observamos que: i)
Se no máximo um termo dentre os xi é não nulo então xi x j ( x12 + x 2j ) = 0, e vale a igualdade se e só se todos os termos são
∑
1≤i < j ≤ n
ii)
nulos. Se exatamente dois termos, digamos xr e xs, são não nulos, caímos no caso n = 2, ou seja, temos
4
1 xi x j ( x12 + x 2j ) ≤ xi , valendo a 8 1≤i ≤n 1≤i < j ≤ n
∑
∑
igualdade se e só se xr = xs. iii)
Finalmente, se há m termos não nulos com m ≥ 3, aplicamos o lema m – 2 vezes e obtemos x1 ' ,..., x n ' onde exatamente dois dos xi ' são não nulos e vale
∑
1≤i < j ≤n
xi x j ( x12
+
x 2j ) ≤
∑
xi ' x j ' ( xi'2 1≤i < j ≤n
+
1 x 'j2 ) ≤
4
4
1 xi ' = xi , onde 8 1≤i≤n 8 1≤i≤n
∑
∑
a segunda desigualdade segue de ii). 1 Assim , temos sempre C = , ocorrendo a igualdade se e só se há exatamente 8 dois termos xr e xs não nulos e xr = xs. PROBLEMA 3
Considere um tabuleiro quadrado n × n, onde n é um inteiro positivo par fixo. O tabuleiro está dividido em n2 quadrados unitários. Dizemos que dois quadrados distintos do tabuleiro são adjacentes se eles têm um lado comum. EUREKA! N°6, 1999
6
Sociedade Brasileira de Matemática
Marcam-se N quadrados unitários do tabuleiro de tal forma que qualquer quadrado (marcado ou não) é adjacente a pelo menos um quadrado marcado. Determine o menor valor possível para N.
Adaptação da solução de Humberto Silva Naves (Goiânia - GO): Vamos criar um algorítmo para preencher estes quadrados n × n . Quando pintarmos um dos quadradinhos temos que pintar pelo menos outro quadradinho adjacente.
Procedemos da seguinte maneira: Pintamos os dois quadradinhos do canto superior esquerdo (1,1) e (1,2). "Caminhamos" no sentido horário no bordo do quadrado, de modo que pintamos 2 quadradinhos e pulamos outros 2. Isso é possível pois o número de quadradinhos no bordo é múltiplo de quatro. Agora formamos outro quadrado menor de canto superior (3,3) e continuamos o mesmo procedimento para este quadrado,que também possui lado de medida par e assim sucessivamente. O número de quadrados pintados vai ser: N(n × n) = 2n – 2 + N((n – 4) × (n – 4)) Este algorítmo determina o menor N, pois cada quadradinho só possui um, e somente um "vizinho" pintado. Essa construção mostra por indução que é possível marcar obter uma solução. EUREKA! N°6, 1999
7
n2 n + quadrados e 4 2
Sociedade Brasileira de Matemática
Para provar que esse número de quadrados marcados é o menor possível associamos a cada quadrado coordenadas (x, y) com 1 ≤ x ≤ n e 1 ≤ y ≤ n. Se considerarmos os quadrados {(x, y) | (x e y são ímpares e x + y ≡ 2 (mod 4)) ou (x é ímpar, y é par e x + y ≡ n + 1 (mod 4))} verificamos que nenhuma peça é adjacente a mais de um dos quadrados deste conjunto. Como este conjunto tem n2 n + elementos, necessitamos no mínimo esta quantidade de peças ❏ 4 2
Segundo Dia Duração da prova: 4 horas e 30 minutos PROBLEMA 4
Determine todos os pares (n, p) de inteiros estritamente positivos tais que • p é primo, • n ≤ 2p, e • (p – 1)n + 1 é divisível por np–1.
Solução de Humberto Silva Naves (Goiânia - GO): Analiçando os casos: p = 2: temos que as únicas soluções são para n = 1 e n = 2. p = 3: temos que as únicas soluções são para n = 1 e n = 3. p >3: se n < p – 1, temos: (Caso trivial n = 1 ⇒ p é divisível por 1). n é ímpar, pois, (p – 1)n + 1 é ímpar, logo n não pode ser par. Lema: Seja a > b ≥ 3, então: ab < ba b a , mas isto é obvio para a, < ln b ln a ln x − 1 x é crescente a partir de e. (t'(x) = ≥ 0 para b ≥ 3, pois a função t ( x) = ln x (ln x) 2 Prova: Basta provar que: b ⋅ ln a < a ⋅ ln b ⇒
x ≥ e.) Afirmação: Se p –1 > n ≥ 3 ( pois n é ímpar), então:
( p − 1) n < n p −1 ⇒ ( p − 1) n + 1 = n p −1 ou ( p − 1) n + 1 < n p −1 . Como EUREKA! N°6, 1999
8
Sociedade Brasileira de Matemática
( p − 1) n + 1 ≠ n p −1 (pois ( p − 1) n + 1 é múltiplo de p e n
p – 1
não é, temos
p −1
n
( p − 1) + 1 < n . Portanto n não pode ser menor que p – 1. Afirmação: Se n > p, então n não é primo: Prova: se n for primo temos: ( p − 1) n + 1 ≡ 0(mod n) ⇒ [a n ≡ a(mod n)] ( p − 1) + 1 ≡ 0(mod n) ⇒ p ≡ 0(mod n) (um absurdo!). Logo, n é composto: n = a ⋅ b (a é o menor fator primo de n). Então: ( p − 1) ab + 1 ≡ 0(mod(ab) p−1 ) ⇒ ( p − 1) ab + 1 ≡ 0(moda p−1 ) ⇒ ( p − 1) ab ≡ −1(moda p−1 ) Seja K o menor natural tal que ( p − 1) K ≡ 1(mod a p −1 ). Devemos ter K 2ab ( pois 2 ab = K ⋅ q + r ⇒ ( p – 1)r ≡ 1 (mod ap–1) ⇒ r = 0). Além disso devemos ter K ϕ (ap–1) ⇒ K ap–2(a – 1). Então K = 2α ⋅ ac, onde α ∈ {0, 1} (pois se K tivesse um outro fator primo além do 2 que dividisse (a – 1), esse fator também teria que dividir 2ab que implica que esse fator dividiria ab, mas como o menor fator primo de n = ab é o a, teriamos absurdo). K é par, pois se não fosse teriamos K ab ⇒ (p – 1)ab ≡ 1 ≡ –1 (mod ap–1), um absurdo pois é ímpar. Logo K = 2 ⋅ ac. Temos c
c
( p − 1) 2⋅a ≡ 1(mod a p −1 ) ⇒ (( p − 1) a ) 2 ≡ 1(mod a p −1 ) ⇒ c
c
( p − 1) a ≡ −1(mod a p −1 ) ⇒ ( p − 1) a ≡ −1(mod a ) ⇒ p − 1 ≡ −1(mod a ), pois b a ≡ b(mod a )) ⇒ p ≡ 0(mod a ), um absurdo pois p ≠ a, porque p < n < 2p). Logo não podemos ter n > p. As únicas possibilidades que restam é n = p – 1 ou n = p. n = p – 1 é falsa pois p – 1 é par. Se n = p temos: ( p − 1) p + 1 ≡ 0(mod p p −1 ) e como p > 3, temos: [ p p + ( ip ) p p −1 (−1)1 + ... + ( 3p ) p 3 ⋅ (−1) p −3 + ( 2p ) p 2 ⋅ (−1) p −2 + (1p ) p1 (−1) p −1 + (−1) p ] +
+ 1 ≡ 0(mod p 3 ) ⇒ p 2 ≡ 0(mod p 3 ). (Um absurdo pois p3 não divide p2). Logo as soluções são: S = {(1, 2); (2, 2); (1, 3); (3, 3)} ∪ { (1, p ) p é primo}❏
EUREKA! N°6, 1999
9
Sociedade Brasileira de Matemática
PROBLEMA 5
Duas circunferências Γ1 e Γ2 estão contidas no interior de uma circunferência Γ e são tangentes a Γ em pontos distintos M e N, respectivamente. A
circunferência Γ1 passa pelo centro de Γ2 . A reta que passa pelos dois pontos de interseção de Γ1 e Γ2 intersecta Γ em A e B. As retas MA e MB intersectam Γ1 respectivamente em C e D. Prove que CD é tangente a Γ2 .
Solução de Pedro Paulo de Simoni Gouveia (Fortaleza - CE): Considere a figura relativa ao problema: Γ1 M
Γ O 2α
D
2α C
W Z N'' O2
A N'
N X
Γ2
Sejam O e O2 os centros de Γ1 e Γ2 respectivamente: Tome uma inversão ϕ = inversão de centro em A e razão igual à potência do ponto A em relação a Γ1 e Γ2 (que é a mesma pois A ∈ ao eixo radical de Γ1 e Γ2 ). Assim ϕ ( M ) = C e ϕ ( N ) = N '. Então a imagem de Γ pela inversão será uma reta ↔
passando por C e N ', como Γ é tangente à Γ1 e Γ2 ⇒ CN ' é tangente também à
Γ1 e Γ2 . Tome agora uma inversão ψ com centro em B e razão igual à potencia de B em relação à Γ1 e Γ2 (B ∈ eixo radical). ↔
ψ ( M ) = D e ψ ( N ) = N ". Essa inversão leva Γ na reta DN " que será tangente comum à Γ1 e Γ2 . EUREKA! N°6, 1999
10
Sociedade Brasileira de Matemática
Temos a seguinte situação. ∧
D
∧
Temos DW = ZC, e como DWC = DZC que DCZW é um trapézio isósceles ⇒ AB // CD ⇒ CD ⊥ OO2 .
C
Basta-nos
______
provar agora que O2 D é ∧
W
bissetriz de N " D C pois aí, O2 será o incentro do triângulo formado por D, C
Z
↔
O2
N''
↔
e o encontro de DN " e CN ' .
N'
Prova: ∧
∧
∧
∧
N " D O2 = α ⇒ D O 1 O2 = 2α ⇒ C O 1 O2 = 2α ⇒ C D O2 = α . ∧
∧
Logo N " O O2 = C D O2 = α ⇒ DO2 é bissetriz ⇒ Γ2 é o incírculo ⇒ Γ2 é tangente a CD ❏ PROBLEMA 6
Determine todas as funções f : R → R tais que
f ( x − f ( y )) = f ( f ( y )) + x f ( y ) + f ( x) − 1 para quaisquer x, y ∈ R. Solução: Fazendo x = f (z ), obtemos f ( f ( z ) − f ( y )) = f ( f ( z )) + f ( f ( y )) + f ( z ) f ( y ) − 1. Fazendo x = f ( y ), obtemos f (0) = 2 f ( f ( y )) + f ( y ) 2 − 1, donde
f ( f ( y )) =
1 + f (0) − f ( y ) 2 . Portanto, 2
f ( f (z) − f ( y)) = f ( f (z)) + f ( f ( y)) + f (z) f ( y) −1 =
EUREKA! N°6, 1999
11
1+ f (0) − f (z) 2 1+ f (0) − f ( y) 2 + + 2 2
Sociedade Brasileira de Matemática
( f (z) − f ( y))2 w2 . Provamos assim que f (w) = f (0) − 2 2 sempre que w puder ser escrito como f ( z ) − f ( y ). Vamos ver que qualquer w ∈ R pode ser escrito desta forma: f ( x − f (0)) − f ( x) = f ( f (0)) + xf (0) − 1. Se tivéssemos f (0) = 0 o lado esquerdo seria sempre 0 e o lado direito seria igual a – 1, absurdo, portanto f (0) ≠ 0 e f ( f (0)) + xf (0) − 1 pode assumir qualquer 1+ w − f ( f (0)) valor real w (basta tomar x = ) , mas então w = f ( z ) − f ( y ) f (0) com z = x − f (0) e y = x. + f (z) f ( y) −1 = f (0) −
w2 para todo w ∈ R. 2 Em particular f ( f ( y )) = f (0) − f ( y ) 2 / 2. Por outro lado
Mostramos assim que f (w) = f (0) −
1 + f (0) − f ( y)2 w2 1 + f (0) para , donde = f (0) ⇒ f (0) = 1, e f (w) = 1− 2 2 2 todo w ∈ R. É fácil verificar que f satisfaz a condição do enunciado ❏ f ( f ( y)) =
Você sabia…
que o livro "Problemas de
las Olimpíadas Matemáticas del Cono Sur (I a IV)" já está a venda na Secretaria da Olimpíada Brasileira de Matemática? Informações: Tel: 21-5295077 e-mail:obm@impa.br
EUREKA! N°6, 1999
12
Sociedade Brasileira de Matemática
14a. OLIMPÍADA IBEROAMERICANA DE MATEMÁTICA 12 a 19 de setembro, La Havana, Cuba O Resultado da equipe Brasileira BRA1 BRA2 BRA3 BRA4
Christian Iveson Fabrício Siqueira Benevides Fernando Paz Cardoso Rui Lopes Viana Filho
PRATA BRONZE PRATA OURO
Primeiro Dia PROBLEMA 1:
Encontre todos os inteiros positivos que são menores que 1000 e cumprem a seguinte condição: o cubo da soma dos seus dígitos é igual ao quadrado do referido inteiro. PROBLEMA 2:
Dadas duas circunferências M e N, dizemos que M bissecta N se a corda comum é um diâmetro de N. Considere duas circunferências fixas C1 e C2 não-concêntricas. a) Prove que existem infinitas circunferências B tais que B bissecta C1 e B bissecta C2. b) Determine o lugar geométrico dos centros das circunferências B. PROBLEMA 3:
Sejam P1, P2,…, Pn n pontos distintos sobre uma reta do plano (n ≥ 2). _____
Consideram-se as circunferências de diâmetro Pi Pj (1 ≤ i < j ≤ n) e colorimos cada circunferência com uma cor escolhida entre k cores dadas. Chamamos (n, k)-nuvem a esta configuração. Para cada inteiro positivo k, determine todos os n para os quais se verifica que qualquer (n,k)-nuvem contém duas circunferências tangentes exteriormente da mesma cor. Nota: Para evitar ambiguidades, os pontos que pertencem a mais de uma circunferência não são coloridos. EUREKA! N°6, 1999
13
Sociedade Brasileira de Matemática
Segundo Dia PROBLEMA 4:
Seja B um inteiro maior que 10 tal que cada um dos seus dígitos pertence ao conjunto {1, 3, 7, 9}. Demonstre que B tem fator primo maior ou igual a 11. PROBLEMA 5:
Um triângulo acutângulo ABC está inscrito numa circunferência de centro O. As alturas do triângulo são AD, BE, e CF. A reta EF intersecta a circunferência em P e Q. a) Prove que OA é perpendicular a PQ. 2
b) Se M é o ponto médio de BC, prove que AP = 2 . AD. OM . PROBLEMA 6:
Sejam A e B pontos do plano e C um ponto da mediatriz de AB.Constrói-se uma sucessão C1, C2,…, Cn ,…., da seguinte maneira: C1 = C e, para n ≥ 1, se Cn não pertence ao segmento AB, Cn+1 é o circuncentro do triângulo ABCn. Determine todos os pontos C tais que a sucessão C1, C2,…, Cn,…está definida para todo n e é periódica a partir de um certo ponto. Nota: Uma sucessão C1, C2, …, Cn,… é periódica a partir de um certo ponto se existem inteiros positivos k e p tais que C n + p = C n para todo n ≥ k.
EUREKA! N°6, 1999
14
Sociedade Brasileira de Matemática
CONTAR DE DUAS MANEIRAS, PARA GENERALIZAR José Paulo Q. Carneiro - Universidade Santa Úrsula
♦ Nível Intermediário 1. O seguinte problema é muito conhecido: quantas partidas há em um campeonato de futebol de um só turno com n times? Uma maneira de raciocinar é: cada time vai jogar n − 1 partidas (com os outros n − 1 times). Portanto, o número de partidas parece ser n(n − 1) . Mas nessa contagem, cada partida foi contada duas vezes (por exemplo, o jogo Vasco × Flamengo foi contado entre os jogos do Vasco e também entre os jogos do n(n − 1) . Flamengo). Logo, o número correto é: 2 Por outro lado, pode-se raciocinar também de outra maneira: O time T1 vai jogar
n − 1 partidas. Colocando essas partidas de fora, o time T2 vai jogar n − 2 partidas contra os clubes restantes (agora já estamos evitando a repetição desde o início!). Além das que já jogou nessa enumeração, o time T3 vai jogar n − 3 partidas, e assim por diante. Então o número total de partidas é: (n − 1) + (n − 2) + " + 1 . (n − 1)n , 2 que é a célebre fórmula da soma dos n − 1 primeiros inteiros positivos. O interessante é que, ao resolver um problema de contagem por dois métodos diferentes, chegamos a uma fórmula de caráter geral. Comparando as duas contagens, conclui-se que: 1 + 2 + " + ( n − 1) =
Deve ser notado também que esta fórmula é básica para deduzir a fórmula geral da soma dos termos de uma progressão aritmética, pois: a1 + (a1 + r ) + ( a1 + 2r ) + " + (a1 + (n − 1)r ) = na1 + (1 + 2 + " + (n − 1))r =
n( a1 + a1 + (n − 1)r ) n(a1 + a n ) (n − 1) n r == = . 2 2 2 2. Vamos agora pensar no número de partidas de um torneio usual de tênis, onde todos os jogos são eliminatórios. Observe primeiramente o torneio do fim para o início. A final é jogada pelos 2 vencedores das 2 semifinais (jogadas por 4 jogadores), que vieram de 4 quartas de final (jogadas por 8 jogadores), e assim por diante. De um modo geral, um torneio desse tipo começa com 2 n jogadores, = na1 +
EUREKA! N°6, 1999
15
Sociedade Brasileira de Matemática
que jogam 2 n −1 partidas entre si de forma eliminatória. Na fase seguinte, sobram 2 n −1 jogadores, que jogam 2 n − 2 partidas, e assim sucessivamente, até a final. O número total de partidas é, portanto: 2 n −1 + 2 n − 2 + " + 2 + 1 . Vamos agora contar o número de partidas de outra forma. Cada partida tem um perdedor (não há empate em tênis). Por outro lado, como o torneio é eliminatório, todos os 2 n jogadores iniciais, exceto o vencedor final, perdem exatamente uma partida. Logo, o número total de partidas do torneio é 2 n − 1 . Comparando os resultados, conclui-se que 1 + 2 + " + 2 n − 2 + 2 n −1 = 2 n − 1 .
3.
A última fórmula é um caso particular da fórmula da soma dos termos de a (1 − q n ) . uma progressão geométrica: a + aq + aq 2 + " + aq n −1 = 1− q Será que esta fórmula também pode ser deduzida por um raciocínio análogo aos anteriores? Para isto, imaginemos um torneio de tênis também eliminatório, mas com q n jogadores iniciais (onde q é um natural > 1 ), e onde, a partir de um ranqueamento prévio, os jogadores são grupados em q n −1 grupos (ou “chaves”) de q jogadores. Dentro de cada grupo, os jogadores previamente ordenados J 1 , ", J q jogam entre si da seguinte forma: J 1 contra J 2 ; o vencedor desta partida joga contra J 3 ; e assim por diante, até J q . Portanto, dentro de cada grupo, há q − 1 partidas, e como são q n −1 grupos, há, nesta primeira fase,
(q − 1) q n−1
partidas. De cada grupo sai um vencedor, e os q n −1 vencedores são
novamente grupados em q n − 2 grupos de q jogadores. O processo se repete, até que se atinge um único grupo de q jogadores, e desse grupo sai o vencedor do torneio. O número total de partidas é (q − 1) q n−1 + (q − 1) q n −2 + " + (q − 1) . Por outro lado, por ser o torneio eliminatório, pelo mesmo princípio anterior, o número total de partidas é também igual ao número inicial de jogadores menos um, isto é: q n − 1. Comparando as duas contagens, conclui-se que: (q − 1)(1 + q + q 2 + " + q n −1 ) = q n − 1 , que é a fórmula procurada.
EUREKA! N°6, 1999
16
Sociedade Brasileira de Matemática
Deve-se notar que esta dedução só vale para q natural > 1 . Mas a fórmula obtida serve também como uma sugestão para qualquer q real (ou complexo!). Para comprovar a sugestão, basta multiplicar os dois fatores do lado esquerdo e simplificar, obtendo o lado direito (ou observer que dois polinômios cujos valores coincidem em infinitos pontos são necessariamente iguais).
4. É curioso que se obtenham as fórmulas de soma dos termos de progressões aritméticas e geométricas através de contagens por dois métodos diferentes, porém mais importante talvez é o próprio método, que é muito fecundo e muito usado, por exemplo, em Análise Combinatória. n Só para dar um exemplo concreto, o símbolo representa o número de k subconjuntos de k elementos que se pode extrair de um conjunto com n elementos (também se usa C kn ), e pode ser calculado, como é conhecido, por:
n n(n − 1) "1 = . 1 ⋅ 2" k k Suponha que se deseje calcular quantas comissões de p pessoas se podem formar a partir de um certo conjunto que consiste de n homens e n mulheres (supõe-se 2n p ≤ n ). É claro que este número é igual a . Por outro lado, se contarmos p separadamente as comissões que são formadas por p homens, mais as que são formadas por p − 1 homens e 1 mulher, mais as que são formadas por p − 2 homens e 2 mulheres, etc., concluímos que este número também é igual a n n n n n n + " + . Daí se deduz, portanto, a interessante + p 0 0 p 1 p − 1 p
fórmula:
n n
2n
∑ k p − k = p . k =0
Para dar um exemplo concreto, tomemos n = 6 e p = 3 , obtendo: 6 6 6 6 6 6 6 6 12 + + + = 1 × 20 + 6 × 15 + 6 × 15 + 1 × 20 = 220 = . 0 3 1 2 2 1 3 0 3 Espero que o leitor tenha ficado estimulado a criar outros exemplos análogos.
EUREKA! N°6, 1999
17
Sociedade Brasileira de Matemática
A MATEMÁTICA DAS ABELHAS José Cloves Saraiva - UFMA
♦ Nível Avançado As abelhas constróem colméias para reservar o mel e para o desenvolvimento e reprodução da sua própria espécie. Cada alvéolo tem a forma de um prisma reto hexagonal fechado numa das extremidades por uma cobertura de três losângos de forma que o volume é máximo enquanto a área lateral total (incluindo a cobertura) é fixada. Com isso as abelhas fazem o menor gasto de cera na construção das paredes peliculares que o constituem. Nesta construção econômica cada parede serve para dois alvéolos contíguos, o que motiva a escolha de prismas retos com seção transversal poligonal regular, como nas figuras abaixo:
É fácil ver que as formas prismáticas com seção transversal um polígono regular de n lados que podem ser usadas para cobrir o espaçõ determinadas pela 360° condição de que é um número inteiro positivo, onde In é o ângulo interno In de um polígono regular.
360° 360° 2n 2n − 4 + 4 4 =2+ = = = que é In ( n − 2) ( n − 2) (n − 2 )180° n − 2 n um inteiro positivo se e só se n – 2 é um divisor de 4, logo devemos ter n = 3, n = 4 ou n = 6 como mostra a figura acima. Podemos escrever
Por um cálculo volumétrico simples das três posibilidades acima observando o mesmo volume, ou escolhendo a forma mais convexa, das três acima indicadas a escolha das abelhas pela forma hexagonal é de fato a mais EUREKA! N°6, 1999
18
Sociedade Brasileira de Matemática
econômica (dentre um triângulo, um quadrado e um hexágono regulares de mesmo perímetro o hexágono tem a maior área.)
Eis como as abelhas colocam os seus alvéolos hexagonais. Esses alvéolos, para maior economia de material , são fechados por três losangos iguais. O valor constante do melhor ângulo agudo de um losango de fechamento causou sério debate entre teólogos, naturalistas e matemáticos.
φ
O problema que as abelhas resolvem pode ser abordado, sem grande dificuldade, com os recursos da Matemática elementar. Como mostra a figura acima, as abelhas fecham uma das bases do prisma com três losangos congruentes cujo ângulo agudo do losango, φ, mede aproximadamente 70°32', o que dá origem ao problema do ângulo de fechamento da cobertura rômbica, que é o seguinte:
De todas as coberturas rômbicas de um prisma reto hexagonal, a que faz volume máximo para área lateral total fixada é formada por três losangos congruentes com ângulo agudo φ ≅ 70°32'. Para uma solução deste problema observe primeiro a comparação do prisma hexagonal com a cobertura rômbica: EUREKA! N°6, 1999
19
Sociedade Brasileira de Matemática
E D A C B
B
E
____
BB ' = h
φ
B r
D
r
B'
r
A
r
C
A figura também ajuda a obtermos as relações métricas necessárias para deduzir o volume V e a área lateral total S em função do raio r da base hexagonal e do ângulo φ, com vértice no ponto D, e os itens seguintes são facilmente comprovados: 1)
____
____
O trapézio ACDE é retângulo com bases CD e AE perpendiculares ao ____
plano que contém o triângulo equilátero ABC, de lado AB medindo r . 2) 3) 4)
5) 6)
____
____
EA = 2 DC
____ 2
____ 2
____ 2
EA 4 ____ 2 ____ 2 BE ___ 2 EA 2 φ 2 2 φ = BD sen = r + sen 4 2 4 2 2
BD = DE = r +
φ BE = EA + r ⇒ (4r + EA ) sen = EA + r ⇒ EA = r 3tg 2 − 1 2 2
____ 2
___ 2
2
2
___ 2
2
φ
___ 2
2
___ 2
A área lateral total S do prisma é dada por: 1/ 2 3 φ r2 2 φ − 1 + rh S = 6 r 2 tg + 3tg 2 4 2 4
EUREKA! N°6, 1999
20
1/ 2
Sociedade Brasileira de Matemática
Usamos aqui que a área de BDE é
BD ⋅ DE ⋅ senφ = 2
___ 2
φ φ EA 3 3 1 = (r 2 + )senφ = r 2 (1 + tg 2 )senφ = r 2 tg (pois senφ = 4 2 8 2 2 4 7)
2tg 1 + tg
φ 2 ). 2
φ
2
O volume V do prisma é dado por:
1/ 2 3 3 2 r 2 φ − 1 + h r 3tg 2 2 2 Fixe V, e obtemos a seguinte expressão para S em função de φ.
V=
1/ 2 3 2 φ 2 φ 4V − 1 + S = r 3tg − 3tg 2 2 2 3r
Observando a expressão acima, vemos que ela torna-se mínima se 1/ 2 φ 2φ − 1 for mínimo para valores de φ com a condição 3tg − 3tg 2 2 φ 3 tg ≥ . A seguinte desigualdade trigonométrica, que provaremos a seguir, 2 3 garante o resultado do problema do ângulo (agudo) de fechamento da cobertura do prisma:
φ
φ φ 3 então 3tg − 3tg 2 − 1 2 3 2 2 φ 2 só se tg = . 2 2 Se S = tg
1/ 2
≥
≥ 2 , valendo a igualdade se e
Demonstração: Para todo φ vale: 2
φ φ φ 3 2 tg − 1 ≥ 0 ⇒ 6 tg 2 − 6 2tg + 3 ≥ 0 2 2 2 Somando-se 3tg 2
φ
2
− 1 aos dois membros da desigualdade obtemos:
EUREKA! N°6, 1999
21
Sociedade Brasileira de Matemática
φ
9tg 2
2
− 6 2 tg
φ 2
+ 2 ≥ 3tg 2
φ
φ 3tg − 2 ≥ 3tg 2 − 1 2 2 3tg
φ
2
2
φ
φ φ − 1 ⇒ 3tg − 2 ≥ 3tg 2 − 1 ⇒ 2 2 2
1/ 2
, e como tg
φ 2
≥
3 , 3
− 2 > 0, logo: 1/ 2
φ
1/ 2
φ φ φ − 2 ≥ 3tg 2 − 1 ⇒ 3tg − 3tg 2 − 1 ≥ 2 2 2 2 2 Daí, é claro que, o menor valor de S é determinado pela igualdade: 3tg
3tg tg
φ
φ − 3tg 2 − 1 2 2
φ 2
=
1/ 2
2
φ = 2 ⇒ 2 tg − 1 = 0 ⇒ 2
2 2 ⇒ φ = 2arc tg ≅ 70°32' 2 2
Finalizando estas notas, sugerimos que o leitor faça a prova de que, o ângulo φ é o mesmo ângulo agudo entre diagonais internas de um cubo.
φ
Referência: -
Malba Tahan, As Maravilhas da Matemática, Bloch Editores.
EUREKA! N°6, 1999
22
Sociedade Brasileira de Matemática
O TEOREMA DE RAMSEY Carlos Gustavo T. de A. Moreira - IMPA
♦ Nível Avançado 1. O teorema de Ramsey para grafos. Vamos começar este artigo lembrando do exemplo 6 do artigo "O Princípio das Gavetas", de Paulo Cezar Pinto Carvalho, publicado na EUREKA! No. 5: se há 6 pessoas numa reunião então há necessariamente 3 pessoas que se conhecem mutuamente ou 3 pessoas que não se conhecem mutuamente na reunião (onde admitimos que, se a conhece b, então b conhece a). Este exemplo equivale ao seguinte: se tomamos 6 pontos, e pintamos cada segmento que une dois desses pontos de preto ou vermelho então necessariamente existe um triângulo cujos vértices são três desses pontos e cujos 3 lados são da mesma cor. Nos exercícios 8 e 9 do mesmo artigo é proposta uma generalização:
Proposição 0: Dados os inteiros a, b ≥ 2 sempre existe um número N inteiro positivo tal que, em qualquer conjunto de N pessoas, sempre existem a pessoas que se conhecem mutuamente ou b pessoas que se desconhecem mutuamente. Em homenagem a F. P. Ramsey, que provou este e outros resultados deste artigo, chamamos o menor número N com esta propriedade de R(a, b). Vamos provar este resultado seguindo os passos propostos do problema 9 do artigo do Prof. Paulo Cezar: i)
R(a, 2) = a para todo a ≥ 2. De fato, num conjunto de a pessoas, ou todas se conhecem ou existem duas que se desconhecem.
ii)
R(a, b) = R(b, a). De fato, conhecer e desconhecer desempenham um papel simétrico no enunciado.
iii)
R( a, b) ≤ R (a − 1, b) + R (a, b − 1), para a, b ≥ 3.
Para provar isto, fixemos uma pessoa P. Se temos R( a − 1, b) + R( a, b − 1) ou mais pessoas na reunião, há pelo menos R (a − 1, b) + R( a, b − 1) − 1 outras pessoas, e um dos seguintes casos se verificará: EUREKA! N°6, 1999
23
Sociedade Brasileira de Matemática
1)
P conhece pelo menos R( a − 1, b) pessoas. Neste caso, por definição de R (a − 1, b) , dentre essas R (a − 1, b) pessoas há b que se desconhecem mutuamente (e não temos mais o que provar), ou a – 1 que se conhecem mutuamente, e, juntando P a essas a – 1 pessoas, obtemos a pessoas que se conhecem mutuamente.
2)
P desconhece pelo menos R(a, b − 1) pessoas. A análise deste caso é análoga à do caso anterior, trocando os papeis de conhecer e desconhecer no argumento ❏
Já mencionamos que podemos enunciar resultados deste tipo em termos de colorações de segmentos (ou mais tecnicamente, de arestas de grafos completos) usando duas cores. O resultado acima pode ser generalizado aumentando o número de cores:
Proposição 1: Dados inteiros k ≥ 1, a1 , a 2 ,...a k ≥ 2 existe n ∈ N tal que dados n pontos, se pintarmos cada segmento que une dois desses pontos de uma dentre k cores possíveis então haverá a1 pontos tais que todo segmento que une dois desses pontos é da primeira cor, ou a2 pontos tais que todos os segmentos que unem dois desses pontos são da segunda cor, ou… ou ak pontos tais que todos os segmentos que unem dois desses pontos são da k-ésima cor. Chamamos de R( a1 , a 2 ,...a k ) o menor número com essa propriedade. Demonstração: Esta proposição pode ser provada de modo análogo à anterior. A existência dos números R (a1 ,...a k ) segue dos seguintes fatos, cuja prova deixamos como exercício: i) ii)
R( a1 ,...a k −1 , 2) = R(a1 ,...a k −1 ) para a1 ,...a k −1 ≥ 2. R (aσ (1) , aσ ( 2) ,...aσ ( k ) ) = R( a1 , a 2 ,...a k ) para a1 ,...a k ≥ 2
e qualquer
permutação σ de {1, 2, …, k}. iii)
R( a1 ,...a k ) ≤ R (a1 − 1, a 2 ,..., a k ) + R( a1 , a 2 − 1, a 3 ,..., a k ) + ... + + R (a1 , a 2 ,...a k −1 , a k − 1) − k + 2.
EUREKA! N°6, 1999
24
Sociedade Brasileira de Matemática
2. Estimativas de números de Ramsey. As demonstrações dos resultados anteriores fornecem estimativas superiores para os números de Ramsey R( a, b) e R( a1 ,...a k ) , por exemplo: i)
( a + b − 2)! , para a, b ≥ 2 (a − 1)!(b − 1)! De fato vale a igualdade para b = 2 (temos os dois lados iguais a a), e se a, b > 2 temos, por indução, R( a, b) ≤ C aa+−b1− 2 =
R( a, b) ≤ R (a − 1, b) + R (a, b − 1) ≤ C aa+−b2−3 + C aa+−b1−3 = C aa+−b1− 2 ❏ ii)
(a1 + ... + ak − 2k + 2)! =: Caa11+−...1,+a2a−k 1−,2ak3 −+22,...ak −2 (a1 − 1)!(a2 − 1)!(a3 − 2)!(a4 − 2)!...(ak − 2)! De fato, isso vale quando todos os ai exceto dois são iguais a 2, e o caso geral segue, por indução da identidade R(a1 ,...,ak ) ≤
b −1
bk bk C bb11+,...,b2b+k...+bk = C bb11+−...1,+b2b,..., + C bb11+,b...2 +−1b,..., + ... + C b11+...+kb−k1−, 1k k −1 k −1 b ,...,b
❏
É menos trivial, entretanto, dar boas estimativas inferiores para números de Ramsey. Para provar, por exemplo, que R(3,3) = 6, é necessário mostrar que há exemplos de grafos completos bicoloridos de 5 vértices sem triângulos monocromáticos, o que pode ser feito explicitamente:
Se o número de pontos, cresce, entretanto, é bastante difícil construir exemplos explícitos. As melhores estimativas conhecidas para R(k, k) se devem ao método probabilistico introduzido pelo grande matemático húngaro Pál Erdös, que se tornou uma das técnicas mais poderosas da teoria dos grafos:
Proposição 2: R(k, k) > 2k/2 para todo k ∈ N. Demonstração: Dados n pontos, pintamos aleatoriamente as arestas que ligam dois desses pontos de vermelho ou preto, com probabilidade 1/2. Dado um k (k − 1) arestas subconjunto de k desses pontos, a probabilidade de que todas as 2 EUREKA! N°6, 1999
25
Sociedade Brasileira de Matemática k ( k −1) / 2
1 que unem dois desses pontos sejam da mesma cor é 2 ⋅ . Como há 2 n! C nk = subconjuntos de k pontos do conjunto inicial de n pontos, a k!(n − k )! probabilidade de que em algum deles todas as arestas sejam da mesma cor é no k ( k −1) / 2
k ( k −1) / 2
k / 2 +1
2 nk 1 1 máximo 2C nk ⋅ que é menor ou igual a se ≤2 ⋅ k ! 2 k ! 2 n ≤ 2 k / 2 , mas 2 k / 2+1 < k! para k ≥ 4, donde há probabilidade positiva de que em nenhum subconjunto de k pontos todas as arestas sejam da mesma cor, e em particular há exemplos desta situação, donde R(k, k) é necessariamente maior que 2k/2 para k ≥ 4. Para k = 3, R(k, k) = 6 > 23/2, e para k =2, R(k, k) = 2 > 21/2, o que conclui a demonstração ❏ Vamos discutir agora como estimar a função f (n) = R(3,3,...,3), com n termos iguais a 3, ou seja, o menor número de vértices de um grafo completo tal que ao pintarmos suas arestas usando n cores necessariamente obtemos um triângulo monocromático. A demonstração da proposição 1 nos fornece f (n) ≤ nf ( n − 1) − n + 2, o que implica, por exemplo, f (n) ≤ 3n! para todo n ≥ 2. (lembremos que f (3) = R(3,3) = 6). Suponha agora que sejam dados
3n + 1 pontos, aos quais atribuímos índices 0, 1, 2
3n − 1 . Vamos descrever uma forma de, a cada par desses pontos, atribuir 2 um número entre 0 e n – 1 (o que equivale a colorir as arestas do grafo completo 3n + 1 vértices usando n cores), sem que haja 3 pontos tais que a cada par de 2 desses pontos é atribuido o mesmo número (ou seja, sem que haja triângulos monocromáticos). 2, …
Ao par de pontos de índices i e j atribuímos um número da seguinte maneira: escrevemos i − j como
n −1
∑σ
r
⋅ 3 r onde σ r ∈{−1,0,1} para todo r, e atribuimos a
r =0
{i, j} o menor k com σ k = 1 (sempre existe um tal k pois i − j > 0). EUREKA! N°6, 1999
26
Sociedade Brasileira de Matemática
Deixamos como exercício para o leitor verificar que não há triângulos monocromáticos nesta configuração (ver problema proposto No. 32 pág 55).
3n + 1 . 2 Determinar exatamente os valores de números de Ramsey clássicos R(a, b) com 3 ≤ a ≤ b que são conhecidos são: R(3, 3) = 6, R(3, 4) = 9, R(3, 5) = 14, R (3, 6) = 18, R(3, 7) = 23, R(3, 8) = 28, R (3, 9) = 36, R(4, 4) = 18, e R(4, 5) = 25. O único número Ramsey com mais de duas cores cujo valor é conhecido é R(3, 3, 3) = 17. Obtemos assim a estimativa f (n) >
3. O Teorema de Ramsey para multigrafos: Até agora estivemos falando sobre colorações de arestas de grafos, ou seja, a cada conjunto de dois vértices associamos uma cor. Uma maneira de generalizar este resultado é associar cores não a pares de vértices, mas a conjuntos de m vértices, onde m é um inteiro positivo fixo (que pode ser maior que 2). Como as configurações que aparecem são um pouco mais complicadas, vamos introduzir notações um pouco mais formais: Dado um conjunto A e um inteiro positivo k denotamos por [A]m o conjunto dos subconjuntos de m elementos de A, ou seja [ A] m = {B ⊂ A # B = k} . Dado j inteiro positivo, definimos Ij = {1, 2, …, j}. A versão m-dimensional (ou para m-hipergrafos) do teorema de Ramsey é dada pelo seguinte teorema (do qual os resultados da seção 1 são casos particulares):
Teorema (Ramsey): Sejam m, k inteiros positivos. Dados a1, a2,…, ak inteiros positivos existe um inteiro positivo, que denotaremos por Rm (a1 ,..., a k ) tal que para todo n ≥ Rm (a1 ,..., a k ) e para qualquer função f : [ I n ] m → I k existem j ∈ I k e A ⊂ I n com # A = a j tal que f ([ A] m ) := { f ( x), x ∈ [ A] m } ⊆ { j}.
Demonstração: Para m = 1 o resultado é uma aplicação simples, do princípio das gavetas: basta tomar R1 (a1 ,..., a k ) = a1 + ... + a k − k + 1. Vamos provar o resultado geral por indução em m. De fato, provaremos que podemos tomar Rm (a1 ,..., ak ) ≤ 1 + Rm−1 ( Rm (a1 − 1,..., a k ), Rm (a1 , a2 − 1..., ak ),..., Rm (a1 , a2 ,...ak − 1)) (e se algum dos ai é menor que m podemos tomar Rm (a1 ,..., a k ) = min{a1 , a 2 ,..., a k }). Note que isto fornece exatamente a recursão de proposição 1 da seção 1 no caso m = 2. EUREKA! N°6, 1999
27
Sociedade Brasileira de Matemática
Se n ≥ 1 + Rm −1 ( Rm (a1 − 1, a 2 ,..., a k ),..., Rm ( a1 , a 2 ,...a k − 1)), dada uma função ~
f : [ I n ] m → I k , definimos uma função f : [ I n −1 ] m −1 → I k da seguinte forma: ~
dado A ∈ [ I n −1 ] m −1 , definimos f ( A) := f ( A ∪ {n}). Como n − 1 ≥ Rm −1 ( Rm (a1 − 1,..., a k ),..., Rm (a1 ,...a k − 1)), existem j ∈ I k e B ⊂ I n −1 com # B = Rm (a1 ,..., a j − 1,..., a k ) tal que f ([ B] m −1 ) = { j}. Agora, por definição
de
Rm ( a1 ,..., a j − 1,..., a k )
A ⊂ B com # A = a i e
existe
i∈ Ik
com
i≠ j
e
m
f ([ A] ) = {i}, caso em que já conseguimos o que
queríamos, ou existe A ⊂ B com # A = a j − 1 e f ([ A] m ) = { j}. Nesse caso, teremos # ( A ∪ {n}) = a j e
f ([ A ∪ {n}] m ) = { j}, pois A ⊂ B , e novamente
conseguimos nosso objetivo .
Obs.: O único número de Ramsey cujo valor é conhecido com m > 2 (e a j > m, ∀ j ) é R3 (4,4) = 13 . O exercício 20 de [PC] pede para mostrar que dados 5 pontos no plano em posição geral há 4 que formam um quadrilátero convexo. O leitor poderia perguntar: o que faz um problema geométrico como este num artigo de combinatória? A resposta está ligada a uma generalização deste resultado, descoberta por Erdös e Szekeres: Dado um inteiro positivo n ≥ 4 existe um inteiro positivo f(n) tal que dados f(n) pontos no plano em posição geral há n deles que são vértices de um n-ágono convexo. Para provar isso, mostraremos que podemos tomar f (n) = R4 (n,5). Para isso, dados R4 (n,5) pontos no plano, e um conjunto de 4 desses pontos, associamos a esse conjunto o número 1 se eles formam um quadrilátero convexo, e 2 em caso contrário.Não é possível que haja 5 pontos tais que a cada 4 deles é associado o número 2, pelo resultado do exercício 20 de [PC], donde, por definição de R4 (n,5) , necessariamente há n desses pontos tais que cada 4 desses pontos formam um quadrilátero convexo, mas isso implica que esses n pontos são vértices de um n-ágono convexo.
5. O teorema de Ramsey infinito: Teorema: Sejam m, k inteiros positivos e A um conjunto infinito. Para qualquer função f: [ A] m → I k existem j ∈ I k e um conjunto infinito EUREKA! N°6, 1999
28
Sociedade Brasileira de Matemática
B ⊂ A tal que f ([ B ] m ) = { f ( x) x ∈ [ B ] m } = { j}
Demonstração: Vamos provar o resultado por indução em m. Para m = 1 o resultado segue do fato de que se X é infinito e C é finito então para toda função f := X → C existe C ∈ C tal que f −1 (c) = {x ∈ X f ( x) = c} é infinito. Seja agora m ≥ 2 e f : [ A] m → I k , onde A é infinito.Fixamos xo ∈ A, e definimos
Ao = A \ {xo } e g : [ Ao ] m −1 → I k por g (C ) := f (C ∪ {xo }), onde C é um subconjunto de m –1 elementos de Ao . Pela hipótese de indução existe um conjunto infinito Bo ⊂ Ao e j o ∈ I k tal que g o ([ Bo ] m −1 ) = { j o }. A partir daí repetimos o processo recursivamente: dado n ≥ 0 fixamos xn +1 ∈ Bn e definimos An +1 = Bn \ {xn +1} e g n +1 = [ An+1 ] m −1 → I k por g n +1(C ) = f (C ∪ {x n +1 }) para C ⊂ An +1 com m –1 elementos. Pela hipótese de indução existe Bn +1 ⊂ An +1 infinito e j n +1 ∈ I k tal que g ([ Bn+1 ] m −1 ) = { j n+1 }. Podemos agora tomar D = {x0, x1, x2,…}, que é um conjunto infinito e definir h : D → I k por h( x r ) = j r . Como Ik é finito, existe j∈Ik tal que
h −1 ( j ) = {x ∈ D h( x) = j} é infinito. Afirmamos que B = h −1 ( j ) satisfaz a condição do enunciado. De fato, dado um subconjunto X = {xi1 , xi2 ,.., xim } de B com m elementos, temos f ( X ) = g i1 ({xi2 ,..., xim }) = j i1 = h( xi1 ) = j ) ❑
Sugestão: Tente usar os resultados deste artigo para resolver o problema “Cuático”da Eureka! No5. Pág. 58. (ver também pág. 56 desta EUREKA!).
Referências: -[R]-
Ramsey, F.P., On a Problem of Formal Logic, Proc. London Math. Soc. 30 (1930) P P. 264-286. -[PC]- Paulo Cezar Pinto Carvalho, Princípio das Gavetas – Eureka! 5 PP.27-33. -[GRS]- R.L. Graham, , B.L. Rothscild, e J.H. Spencer, Ramsey Theory. Wiley. Interscience, 1990. -[ES]- Erdös e G. Szekeres – A Combinatorial Problem in Geometry – Compositio Math.2 (1935), PP-464-470. -[Rad]- Stanislaw P. Radziszowski – “Smal “Ramsey Numbers – Dinamic SurveysElectronic Journal of Combinatorics. http://www.combinatorics.org -[Er]- Pál Erdös, Some Remarks on The Theory Of Graphs, Bull. Amer. Math. Soc. 53 (1947), PP.292-294. EUREKA! N°6, 1999
29
Sociedade Brasileira de Matemática
APLICAÇÕES DOS NÚMEROS COMPLEXOS À GEOMETRIA Edmilson Motta - Colégio Etapa
♦ Nível Avançado É importante ter em mente que os números complexos não são apenas vetores; eles podem ser multiplicados. Nas aplicações à Geometria, nós faremos uso extensivo desta propriedade. Números Complexos são particularmente eficientes para certos tipos de problemas, mas podem gerar dificuldades artificiais em problemas que admitem soluções mais diretas utilizando outros métodos. Na Geometria Elementar, os triângulos são as peças básicas e a congruência e a semelhança de triângulos, os conceitos fundamentais. Nós começaremos caraterizando a semelhança de triângulos em termos de números complexos. Inicialmente, vamos estabelecer algumas convenções. Sejam z1, z2, z3, w1, w2, w3 números complexos. Nós dizemos que Uz1z2z3 e Uw1w2w3 são semelhantes, e escrevemos U z1 z2 z3 ~ U w1w2w3, se e somente se, o ângulo em zk é igual ao ângulo em wk , k = 1, 2, 3, e têm mesma orientação, isto é, ambos anti-horários ou ambos horários (Veja figura a seguir.) z1
w2 w3
z3 w1 z2
Se os triângulos tem orientações distintas (um horário, o outro antihorário), então escrevemos
Uz1 z2 z3 ~ Uw1w2w3 (reverso) Como para complexos α, β e γ distintos,
EUREKA! N°6, 1999
30
Sociedade Brasileira de Matemática
arg →
→
→
→
β −α = arg ( β − α ) − arg (γ − α ) = medida do ângulo orientado entre γ −α
αγ e αβ , então α, β, γ são colineares ⇔ a medida do ângulo orientado entre αγ e αβ é múltipla de π ___________
__
__
β −α β −α β −α β −α β− α ⇔ . = = ∈R ⇔ ⇔ γ −α γ −α γ −α γ − α __ __ γ −α Exercício 1: __
__
β −α β−α β −α Mostre que αβ ⊥ αγ ⇔ é imaginário puro ⇔ = 0. + γ −α γ − α __ __ γ −α Generalizando, para quatro pontos distintos α, β, γ, δ ∈ C, →
→
__
__
β −α β −α β−α , αβ || γδ ⇔ = ∈R ⇔ δ −γ δ − γ __ __ δ−γ →
→
→
β −α é um real δ −γ
→
além disso, αβ e γδ tem a mesma direção se, e somente se, __
__
β −α β− α β −α positivo; αβ ⊥ γδ ⇔ é imaginário puro ⇔ = 0. + δ −γ δ − γ __ __ δ−γ →
→
Exercício 2: Mostre que, se αβ ≠ 0, então α + β = α + β ⇔
α > 0. β
z1 w1 1 z 2 − z1 w2 − w1 Teorema: Uz1 z2 z3 ~ Uw1w2w3 ⇔ = = z 2 w2 1 = 0. z 3 − z1 w3 − w1 z 3 w2 1 Demonstração: Dois triângulos são semelhantes se, e somente se, (caso LAL) as razões entre as medidas de dois pares de lados correspondentes são iguais e os ângulos entre estes lados são iguais (incluindo a orientação). Assim, z − z1 w2 − w1 z − z1 w − w1 Uz1 z2 z3 ~ Uw1w2w3 ⇔ 2 = e arg 2 = arg 2 z 3 − z1 w3 − w1 z 3 − z1 w3 − w1 EUREKA! N°6, 1999
31
Sociedade Brasileira de Matemática
z1 w1 1 z 2 − z1 w2 − w1 ⇔ = ⇔ z 2 w2 1 = 0. z 3 − z1 w3 − w1 z 3 w3 1 Corolário: __
__
z1 w 1 1
__
Uz1 z2 z3 ~ Uw1w2w3 (reverso) ⇔ z 2 − z 1 = w 2 − w1 ⇔ z __ 2 w 2 1 = 0. __ __ z 3 − z1
__ __
w 3 − w1
__
z3 w 3 1
__
Demonstração: Temos que U w1 w2 w3 ~ Uw1 w2 w3 (reverso). Logo __ __
__
Uz1 z2 z3 ~ Uw1w2w3 (reverso) ⇔ Uz1 z2 z3 ~ U w1 w2 w3 Exercício 3: __ __
__
Complete a demonstração acima, verificando que U w1 w2 w3 ~ Uw1w2w3 (reverso). Vejamos agora alguns exemplos de aplicações mais específicas. • CARACTERIZAÇÕES DOS TRIÂNGULOS EQUILÁTEROS. Sendo ω 2 + ω + 1 = 0,
z1 z 3 1 Uz1 z2 z3 é equilátero ⇔Uz1 z2 z3 ~ Uz3 z1 z2 ⇔ z 2 z1 1 = 0
z3 z 2 1 ⇔
z12
+
z 22
+
z 32
− z 2 z 3 − z 3 z1 − z1 z 2 = 0
⇔ ( z1 + ω z 2 + ω 2 z 3 ) ⋅ ( z1 + ω 2 z 2 + ω z 3 ) = 0 ⇔ ( z1 + ω z 2 + ω 2 z 3 ) = 0 ou ( z1 + ω 2 z 2 + ω z 3 ) = 0 z1 1 1 z1 1 1 ⇔ z 2 ω 1 = 0 ou z 2 ω 2 1 = 0 z3 ω 2 1
z3 ω 1
⇔Uz1 z2 z3 ~ U1 ωω ou Uz1 z2 z3 ~U1ω2ω 2
EUREKA! N°6, 1999
32
Sociedade Brasileira de Matemática
(geometricamente, esta última caracterização é bastante intuitiva).
y
ω x 0
1
ω2
• O TEOREMA DE NAPOLEÃO. Sobre cada lado de um triângulo arbitrário, desenhe um triângulo equilátero (no exterior). Temos então que os baricentros desses três triângulos equiláteros são os vértices de um quarto triângulo equilátero. w3
w2
z1 ζ2
ζ3
z3 z2 ζ1
w1
Demonstração: Sejam Uz1z2z3 o triângulo dado; Uw1z3z2, Uz3w2z1, Uz2z1w3 triângulos equiláteros com a mesma orientação que U1ωω2, digamos; e ζ1, ζ2, ζ3, os baricentros desses triângulos. Então w1 + ω z 3 + ω 2 z 2 = 0
z 3 + ω w 2 + ω 2 z1 = 0 z 2 + ω z1 + ω 2 w3 = 0 Para provarmos que Uζ1ζ2ζ3 é equilátero, calculamos EUREKA! N°6, 1999
33
Sociedade Brasileira de Matemática
1 3
ζ 1 + ωζ 2 + ω 2ζ 3 = ( w1 + z 3 + z 2 ) +
ω 3
( z 3 + w 2 + z1 ) +
ω2 3
( z 2 + z1 + w3 )
1 ((w1 + ω z 3 + ω 2 z 2 ) + ( z 3 + ω w2 + ω 2 z1 ) + ( z 2 + ω z1 + ω 2 w3 )) = 0. 3 PortantoUζ1ζ2ζ3 é um triângulo equilátero. =
Exercício 4: Sendo α, β, γ, δ números complexos, temos que (α − β).(γ −δ) + (α −δ ) ⋅ (β −γ ) = (α −γ ) ⋅ (β −δ ) (a) A partir da identidade acima, mostre que α − β ⋅ γ −δ + α −δ ⋅ β −γ ≥ α −γ ⋅ β −δ . (b) Demonstre o teorema de Ptolomeu-Euler: Para quaisquer pontos A, B, C, D no plano, AB ⋅ CD + BC ⋅ DA ≥ AC ⋅ BD, com igualdade se, e somente se, estes quatro pontos são concíclicos ou colineares.
Exercício 5: (a) Sejam a, b, c, d, e, f números complexos. Prove que
a −b c − d e − f b−c a −e f −d . ⋅ =1⇔ ⋅ ⋅ = 1. c −b e − d a − f a −c f −e b−d (b) Utilizando o item anterior, resolva o seguinte problema do banco da IMO 98: AB CD EF ⋅ ⋅ = 1. Seja ABCDEF um hexágono convexo tal que ∠B + ∠D + ∠F = 360° e BC DE FA BC AE FD ⋅ ⋅ = 1. Prove que CA EF DB MAIS EXEMPLOS PROBLEMA (BANCO / IMO 98)
Seja ABC um triângulo, H o seu ortocentro, O o seu circuncentro e R o seu circunraio. Seja D o simétrico de A com relação a BC, E o simétrico de B com relação a AC e F o simétrico de C com relação a AB. Prove que D, E e F são colineares se, e somente se, OH = 2R.
EUREKA! N°6, 1999
34
Sociedade Brasileira de Matemática
RESOLUÇÃO:
Sejam a, b, c, h e 0 as coordenadas complexas do A, B, C, H e O, __
__
__
respectivamente. Conseqüentemente, a a = b b = c c = R2 e h = a + b + c. Como D é o simétrico de A com relação a BC, d satisfaz _________
__ __ __ __ __ d −b a −b = ⇔ b − c d − (b − c) a+ b c − b c = 0. c −b c −b
(1)
Temos que __ __
b− c = −
__ __ R2(b − c) R2(b2 − c2) e b c− b c = , bc bc
substituindo em (1), obtemos
− bc+ ca+ ab k − 2bc = , a a R2(−a + b + c) R2(h − 2a) d= = , bc bc d=
onde k = bc + ca + ab. Analogamente k − 2ca __ R2(h − 2b) k − 2ab __ R2(h − 2c) e= , e = ,f = e f= . b ca c ab Como __
d d1
(b − a)(k − 2ab) R2(a − b)(h − 2c) __ e −d e− d ab abc ∆= e e 1 = = __ __ 2 (c − a)(k − 2ca) R (a − c)(h − 2b) __ f −d f − d f f 1 ca abc __ __
= =
R2(c-a)(a − b) − (ck-2abc) (h − 2c) × (bk − 2abc) − (h − 2b) a2b2c2
− R 2 (b − c)(c − a )(a − b)(hk − 4abc) a 2b 2c 2
__
e h = R2k / abc, segue que D, E e F são colineares
EUREKA! N°6, 1999
35
Sociedade Brasileira de Matemática
⇔ ∆=0 ⇔ hk − 4abc= 0 __
⇔ h h = 4R2 ⇔ OH = 2R. PROBLEMA (Olimpíada Chinesa 98)
Seja D um ponto no interior de um triângulo acutângulo ABC, com DA ⋅ DB ⋅ AB + DB ⋅ DC ⋅ BC + DC ⋅ DA ⋅ CA = AB ⋅ BC ⋅ CA. Determine quais são as possíveis posições que D pode ocupar. RESOLUÇÃO:
Sejam a, b, c, e 0 as coordenadas complexas de A, B, C e D, respectivamente. Temos, então que DA ⋅ DB ⋅ AB + DB ⋅ DC ⋅ BC + DC ⋅ DA ⋅ CA = AB ⋅ BC ⋅ CA ⇔ a ⋅ b ⋅ (b − a) + b ⋅ c ⋅ (c − b) + c ⋅ a ⋅ (a − c) = (b − a)(c − b)(a − c) (*) Como ab(b − a ) + bc(c − b) + ca(a − c) = −(b − a)(c − b)(a − c) , sendo w1 = ab(b − a ),
w2 = bc(c − b), w3 = ca( a − c), (*) ⇔ w1 + w2 + w3 = w1 + w2 + w3 e portanto, w1, w2, w3 estão alinhados. Assim, existem reais positivos α e β tais que w1 = αw2 ab(b − a ) = αbc(c − b) a (b − a ) = αc(c − b) α b−c a , ⇒ =− ⋅ ⇔ ⇔ β a−c b w1 = βw3 ab(b − a ) = βca( a − c) b(b − a) = βc(a − c) ∧
∧
∧
∧
isto é, ∠A C B = 180 D − ∠A DB e, analogamente, ∠A B C = 180 D − ∠A DC e ∧
∧
∠B A C = 180 D − ∠B DC. O único ponto D no interior de um triângulo acutângulo que satisfaz essas condições é o ortocentro. PROBLEMA (Olimpíada Universitária Húngara 1995)
São dados n pontos na circunferência unitária de modo que o produto das distâncias de qualquer ponto da circunferência a estes pontos é menor ou igual a 2. Prove que os pontos são vértices de um n–ágono regular. RESOLUÇÃO:
Considere a circunferência centrada na origem e sejam z1, z2, …, zn os números complexos que representam os pontos. Podemos assumir que (−1) n ⋅ z1 ⋅ z 2 ...z n = 1 (verifique! ). EUREKA! N°6, 1999
36
Sociedade Brasileira de Matemática
Considere ainda o seguinte polinômio p( w) = ( w − z1 )( w − z 2 )...(w − z n ) = wn + a1 wn−1 + ... + an−1w + 1 = wn + Q (w) + 1 Então |p(z)| é o produto das distâncias do ponto representado pelo número complexo z aos pontos dados . Logo, se z é um número complexo de módulo 1, então | p(z)| ≤ 2. Sejam w1, w2,… wn as raízes n-ésimas da unidade. Sabe-se que w1k + w2k + ... + wnk = 0 para todo k = 1, 2,…,n – 1. Portanto Q( w1 ) + Q( w2 ) + ... + Q( wn ) = 0. Se Q(w) não é identicamente nulo, então, para algum j, Q (wj) é diferente de zero e tem parte real não negativa, pois Q(0) = 0 e Q tem no máximo n – 1 raízes. Consequentemente, p ( w j ) = 2 + Q ( w j ) > 2 , uma contradição. Desta forma o polinômio Q é identicamente nulo e p(z) = zn + 1. As raízes z1, z2, …, zn do polinômio p(z) formam um n-ágono regular.
Mais alguns exercícios relacionados. PROBLEMA 1 (IMO 63)
Todos os ângulos internos de um n-ágono são iguais e seus lados satisfazem a relação a1 ≥ a 2 ≥ ... ≥ a n . Prove que a1 = a 2 = ... = a n . PROBLEMA 2 (Treinamento para IMO 92)
Prove que para todo inteiro positivo n, (2 + i ) n ≠ (2 − i ) n . Conclua que os ângulos agudos do triângulo de lados 3, 4 e 5 são irracionais quando expressos em graus. PROBLEMA 3 (IMO 75)
Determine se existem ou não 1975 pontos sobre a circunferência unitária tais que a distância entre quaisquer dois é um número racional. PROBLEMA 4 (Olimpíada Romênia 92)
Sejam p, q ∈ C, q ≠ 0. Se as raízes da equação x 2 + px + q = 0 têm o mesmo p é um número real. módulo, mostre que q
EUREKA! N°6, 1999
37
Sociedade Brasileira de Matemática
PROBLEMA 5 (Seleção para IMO 92)
A área do polígono A1 A2 ... An é S. São dados um ângulo α e um ponto Q. Rodemos Q de um ângulo α no sentido anti-horário ao redor de Ai para encontrar um ponto Pi. Ache a área de P1 P2 ...Pn . PROBLEMA 6 (Putnam 67)
Seja ABCDEF um hexágono inscrito em uma circunferência de raio r. Mostre que ____ ____
___
se AB = CD = EF = r, então os pontos médios de BC , DE , e FA são os vértices de um triângulo equilátero. PROBLEMA 7 (IMO 86)
São dados um triângulo A1 A2 A3 e um ponto P0 no plano. Definimos As = As −3 para todo s ≥ 4 e construímos uma seqüência de pontos P0 , P1 , P2 ,... tais que Pk +1 é a imagem de Pk sob a rotação com centro Ak +1 e ângulo de 120° (sentido horário). Prove que se P1986 = P0 , então o triângulo A1 A2 A3 é equilátero. PROBLEMA 8 (Olimpíada Hungria 1899)
A0 , A1 , A2 , A3 , A4 dividem a circunferência unitária em cinco partes iguais. Prove que ( A0 A1 ⋅ A3 A2 ) 2 = 5. PROBLEMA 9 (Putnam 55)
A1 , A2 ,... An é um polígono regular inscrito em uma circunferência de raio r e n
→
centro O. P é um ponto sobre OA1 . Mostre que
∏ PA
k
= OP n − r n .
k =1
PROBLEMA 10
Dados um ponto P sobre uma circunferência unitária e os vértices A1 , A2 ,... An de um n-ágono regular inscrito, prove que PA12 + PA22 + ... + PAn2 e PA14 + PA24 + ... + PAn4 são constantes.
Bibliografia: -
Complex numbers & Geometry, The Mathematical Association of America Liang - Shin Hahn, 1994.
EUREKA! N°6, 1999
38
Sociedade Brasileira de Matemática
40a. OLIMPÍADA INTERNACIONAL E 14a. OLIMPÍADA IBEROAMERICANA DE MATEMÁTICA Segundo teste de Seleção PROBLEMA 1
Encontre todos os inteiros positivos l, m, n, primos dois a dois, tais que (l + m + n)(1/l + 1/m + 1/n) é um inteiro positivo. PROBLEMA 2
Considere UABC inscrito em uma circunferência fixa. Sejam D, E e F pontos ____ ____
___
arbitrários distintos dos vértices, pertencentes aos lados BC, CA e AB, respectivamente. Sejam P, Q e R os pontos de intersecção das retas AD, BE e CF com a circunferência, respectivamente. Mostre que AD BE CF + + ≥ 9. PD QE RF Determine também as condições sobre o UABC e os pontos D, E e F para que a igualdade ocorra. PROBLEMA 3
Para n natural, seja φ(n) a quantidade de números naturais que são primos com n e ψ(n) o número de fatores primos de n. Mostre que se φ(n) é um divisor de n – 1 e ψ(n) ≤ 3, então n é primo. PROBLEMA 4
Para reais positivos satisfazendo a + b + c = abc, mostre que 1 1 1 3 + + ≤ , e determine quando a igualdade ocorre. 1+ a2 1+ b2 1+ c2 2 PROBLEMA 5
Seja I o incentro de UABC, O1 uma circunferência passando por B e tangente à reta CI e O2 uma circunferência passando por C e tangente à reta BI. Mostre que a circunferência circunscrita ao UABC e as circunferências O1 e O2 interceptam-se em um único ponto. PROBLEMA 6
Seja Fn o conjunto de todas as bijeções f de {1, …, n} em {1, …, n} satisfazendo f (k) ≤ k +1 para k = 1, 2, …n f (k) ≠ k para k = 2, …, n. (i) e (ii) Determine a probabilidade de que f (1) ≠ 1 para um f arbitrário em Fn EUREKA! N°6, 1999
39
Sociedade Brasileira de Matemática
10a. OLIMPÍADA DE MATEMÁTICA DO CONE SUL Primeiro teste de Seleção PROBLEMA 1
Em um tabuleiro 1999 × 1999 encontra-se um certo número de torres (torre é uma peça que se move horizontalmente ou verticalmente). Prove que é possível colorir as torres utilizando três cores de modo que nenhuma torre ataque outra de mesma cor (uma torre ataca outra quando ambas estão na mesma linha ou coluna sem peças intermediárias). PROBLEMA 2
Encontre todas as soluções reais de x ⋅[x ⋅[x ⋅[x]]]= 88, onde [x] é o inteiro satisfazendo [x] ≤ x < x +1 (por exemplo, [3, 7] = 3, [4] = 4 e [–π] = – 4. PROBLEMA 3
A bissetriz do ângulo B em um triângulo ABC intercepta o lado AC no ponto D. ∧
∧
Seja E um ponto sobre o lado BC tal que 3C AE = 2B AE. Os segmentos BD e AE interceptam-se no ponto P. Se ED = AD = AP, determine os ângulos do triângulo. PROBLEMA 4
Mostre que há infinitos naturais n tais que n2+1 divide n!, onde n!= n ⋅ (n −1)...2⋅1 por exemplo, 4!= 4⋅ 3⋅ 2⋅1 = 24). PROBLEMA 5
Considere um polígono convexo com 2000 lados no plano. Prove que é possível escolher 1998 pontos no plano tais que qualquer triângulo formado por vértices do polígono tenha exatamente um dos pontos escolhidos em seu interior.
EUREKA! N°6, 1999
40
Sociedade Brasileira de Matemática
10a. OLIMPÍADA DE MATEMÁTICA DO CONE SUL Segundo teste de Seleção PROBLEMA 1
Sejam p, q, r, s inteiros não negativos tais que ( p + q)2 + q = (r + s)2 + s Prove que p = r e q = s. PROBLEMA 2
As mn casas de um tabuleiro m × n (m, n > 1 inteiros) são pintadas alternadamente de branco e preto, como em um tabuleiro de xadrez. Em seguida, colocamos em cada casa um número inteiro, de modo que a soma dos números de cada linha e a soma dos números de cada coluna sejam pares. Prove que a soma dos números escritos nas casas pretas é par. PROBLEMA 3
Seja ABCD um paralelogramo, H o ortocentro do triângulo ABD e O o circuncentro do triângulo BCD. Prove que os pontos H, O e C são colineares. PROBLEMA 4
Determine todos os ternos (a, b, c) de inteiros positivos tais que a e b são pares e ab + ba = 2c.
❖❖❖
Você sabia… Que o record de maior primo de Fermat n
generalizado (i. e., da forma a 2 + 1 ) conhecido foi batido em 2/11/1999? Este primo é 11402416384 + 1, tem 82854 dígitos e foi descoberto por Kimmo Herranen, usando um programa desenvolvido por Yves Gallot (veja http://perso.wanadoo.fr/yves.gallot/primes/gfn.html)
EUREKA! N°6, 1999
41
Sociedade Brasileira de Matemática
SOLUÇÕES DE PROBLEMAS PROPOSTOS Publicamos aqui algumas das respostas enviadas por nossos leitores. 10) Suponha que temos k moedas, todas iguais exceto por uma que tem peso ligeiramente diferente das anteriores (não se sabe se maior ou menor), e uma balança de dois pratos. a) Mostre que se k ≤
3n − 3
é possível determinar com n pesagens qual 2 é a moeda diferente, e se ela é mais leve ou mais pesada que as outras. 3n − 1 é possível determinar com n pesagens qual b) Mostre que se k = 2 é a moeda diferente, mas nem sempre é possível dizer se ela é mais leve ou mais pesada que as outras. 3n − 1 não é sempre possível determinar qual é a c) Mostre que se k > 2 moeda diferente.
Solução do Editor: Fato 1: Se tivermos 3n ou menos moedas com apenas uma diferente, e que seja sabido se a moeda diferente é mais leve ou mais pesada então é possível determinar a moeda diferente com apenas n pesagens. Prova : Se n = 0 a afirmação é óbvia. Vamos provar o fato por indução. Se tivermos 3 n +1 ou menos moedas de modo que uma delas é, digamos, mais pesada que as outras, dividimos as moedas em três grupos, A, B e C, de 3 n ou menos moedas cada, de modo que A e B tenham o mesmo número de moedas. Pesamos o grupo A com o grupo B. Se os pesos forem diferentes, o grupo mais pesado tem 3 n ou menos moedas e contém a moeda diferente, e portanto podemos , com mais n pesagens (por hipótese de indução), determiná-la. Se os pesos forem iguais a moeda mais pesada estará no grupo C, e , como antes, podemos determiná-la em mais n pesagens.
EUREKA! N°6, 1999
42
Sociedade Brasileira de Matemática
Corolário 1:
3n − 1 moedas das quais uma é diferente, e tivermos um 2 2k + 1 n −1 grupo de pelo menos moedas padrão então é possível ≤3 3 determinar a moeda diferente e se ela é mais leve ou mais pesada com n pesagens. Se tivermos k ≤
3n − 1 2k + 1 n −1 moedas dentre as , e Prova: Separamos um grupo A de ≤ 3 2 3 2k + 1 n −1 moedas padrão. Se der pesamos com um grupo de ≤3 3 diferente saberemos que a moeda diferente está no grupo A e se ela é mais leve ou mais pesada, e, usando o Fato 1, podemos determiná-la com mais n – 1 pesagens. Se der igual a moeda diferente está no grupo das n 3 n −1 2k + 1 3 − 1 n −1 moedas restantes, o que prova o 3 k− ≤ − = 2 2 3 corolário por indução, dado que temos uma quantidade suficiente de moedas padrão e, para n = 1, se tivermos uma moeda padrão é só pesá-la com a moeda diferente para descobrir se esta é mais leve ou mais pesada. Corolário 2: Se tivermos k ≤
3n + 1 moedas das quais uma é diferente e tivermos pelo 2
2k − 1 n −1 menos ≤ 3 moedas padrão então é possível determinar com n 3 pesagens a moeda diferente. Prova: Para n = 1 temos 2 moedas. Pesamos uma delas com uma moeda padrão. Se der diferente é ela a diferente, se der igual é a outra. O resto da prova (por indução) é igual à do corolário 1.
3n − 1 moedas cada uma das quais 2 é diferente tais que o peso total de A é maior que o peso total de B e pelo
Fato 2: Se tivermos dois grupos A e B de k ≤
EUREKA! N°6, 1999
43
Sociedade Brasileira de Matemática
2k + 1 n − 2 moedas padrão então é possível determinar menos 2 ≤ 2⋅3 9 com n pesagens a moeda diferente e se ela é mais leve ou mais pesada.
2k + 1 n−2 Prova: Seja r = ≤ 3 . Pesamos de um lado 2r moedas do grupo A, r 9 do grupo B e do outro r moedas do grupo A, r do grupo B e 2r moedas padrão. Se o primeiro lado tiver peso maior dividimos suas 2r moedas do grupo A em dois grupos de r e os pesamos. Se der igual a moeda diferente será mais leve e estará entre as r ≤ 3 n − 2 moedas do grupo B que estavam do outro lado. Se der diferente a moeda diferente será mais pesada e estará no lado mais pesado nessa última pesagem. Nesses casos o Fato 1 implica o resultado pois com mais n – 2 pesagens descobrimos a moeda diferente. Se o primeiro lado tiver peso menor a análise é análoga, trocando os papeis dos grupos A e B. Finalmente, se der igual a moeda diferente estará nos grupos A' ⊂ A ou n −1 2k + 1 3 − 1 moedas que não foram pesadas, e o B ' ⊂ B de k − 3 ≤ 2 9 resultado segue por indução ( O caso n = 1 é trivial) ❏ Vamos agora resolver os ítens originais do problema:
3n − 3 moedas pesamos dois grupos de 2 n k − 1 3 − 1 moedas cada. Se der diferente usamos o Fato 2, e se m= ≤ 2 3
a) Se
temos
k≤
der igual usamos o Corolário 1 para as
k − 2m ≤
3 n −1 − 1 moedas 2
restantes.
3n − 1 3 n −1 − 1 pesamos dois grupos de moedas cada. Se 2 2 der diferente usamos o Fato 2, e se der igual usamos o Corolário 2 para as 3 n −1 + 1 moedas restantes. 2
b) Se temos k =
EUREKA! N°6, 1999
44
Sociedade Brasileira de Matemática
Para ver que não é possível descobrir a moeda diferente e se ela é mais leve ou mais pesada com n pesagens, observamos que se na primeira 3 n −1 − 1 moedas em cada prato e der pesagem pesamos mais que 2 3n −1 +1 = 3n −1 +1 > 3n −1 diferente, teremos no mínimo 2 2 possibilidades para a moeda diferente, e não podemos distingui-las com n – 1 pesagens (que dão no máximo 3 n −1 possíveis resultados finais). 3 n −1 − 1 moedas em cada prato na Assim, devemos ter no máximo 2 3 n −1 + 1 moedas. O primeira pesagem, e se der igual sobram pelo menos 2 resultado seguirá do 3k + 1 moedas das quais sabemos apenas que uma é diferente, e Lema: Se temos 2 algumas moedas padrão não podemos distinguir a moeda diferente e se ela é mais leve ou mais pesada com k pesagens. Prova: Pelo argumento acima, não podemos usar mais de 3 k −1 moedas desconhecidas na primeira pesagem, e se esta der equilíbrio sobram pelo 3 k −1 + 1 moedas, e o resultado segue por indução (o caso k = 0 é menos 2 trivial). 3n − 1 moedas em cada prato na c) Como antes não podemos usar mais que 2 3 n −1 + 3 primeira pesagem, e se esta der igual sobrarão pelo menos 2 moedas, caso em que não é possível determinar a moeda diferente com mais n – 1 pesagens, o que pode ser provado do mesmo modo que o lema anterior ❏
EUREKA! N°6, 1999
45
Sociedade Brasileira de Matemática
16) Seja
l
a
reta
{( x, y ) ∈ R 2 y = 0}, C1
o
círculo
centrado
em
1 1 1 1 (0, ) de raio e C 2 o círculo centrado em (1, ) de raio . 2 2 2 2 Seja F o conjunto de círculos em R2 com as seguintes propriedades: i) ii)
{C1, C2} ⊂ F Se C e C’ pertencem a F, são tangentes entre si e tangentes a l ~
então todo círculo C tangente aos dois círculos C e C’ e à reta l pertence a F. iii)
~
Se F é um conjunto de círculos satisfazendo as propriedades i) e ~
ii) então F ⊂ F . Determine o conjunto dos pontos de tangência dos círculos C ∈ F com a reta l.
Solução de Zoroastro Azambuja Neto (Rio de Janeiro - RJ): O conjunto dos pontos de tangência será o conjunto {( x,0), x ∈ Q ∩ [0,1]}. Cada p racional x ∈ [0,1] será representado por onde p é inteiro, q é inteiro positivo e q mdc( p, q) = 1. Para provar isso mostraremos os seguintes fatos por indução: p 1 O círculo tangente em ( , 0) terá raio . i) q 2q 2 p r e Se os círculos tangentes em são tangentes entre si ii) q s então ps − qr = 1 Para isso, notemos que se dois círculos centrados em (x, r1) e (y, r2) são tangentes à reta l e tangentes entre si então (r1 − r2 ) 2 + d 2 = (r1 + r2 ) 2 ⇒ d 2 = 4r1r2 ⇒ d = 2 r1 r2 , onde d = x − y .
EUREKA! N°6, 1999
46
Sociedade Brasileira de Matemática r1 + r2 r1
r2
d
As afirmações i) e ii) são verdadeiras para os círculos iniciais C1 e C2. Se o p 1 círculo C é tangente a l e tem centro , 2 , o círculo C ' é tangente a l e tem q 2q ~ r 1 centro , 2 e qr – ps = 1 então, se o círculo C tangente a C e C ' e à reta l s 2s p p r r tem centro (x, y) com < x < então, se d ' = x − e d ' ' = − x, devemos q s q s
d'=
ter
2( q + s ) qs
2 q
y 2
e
d" =
2 s
y , 2
e
d '+ d " =
r p 1 − = , s q qs
donde
y 1 p p(q + s) + 1 1 1 = ⇒y= e d'= ⇒ x = + d'= = 2 q(q + s) q q(q + s) 2 qs 2( q + s )
~ p+r p+r (pois ps = qr – 1). Assim, C é tangente em ,0 e tem q+s q+s 1 raio . 2(q + s ) 2
=
Como q ( p + r ) − p (q + s ) = qr − ps = 1 e (q + s )r − ( p + r ) s = qr − ps = 1 vemos ~
que C satisfaz i) e ii). Esses fatos implicam que todos os círculos criados terão centro em pontos p p racionais. Basta provar agora que para todo racional ∈ [0,1], o ponto , 0 é q q ponto de tangência de algum dos círculos. Faremos isto por indução em q (para q = 1 o resultado é óbvio): basta mostrar se mdc(p, q) = 1 e q ≥ 2 que e possível escrever p = p' + p" e q = q' + q" com p', p", q', q" inteiros, q', p" ≥ 0 e q'p"– p'q" = 1. Estas equações podem ser escritas como q ' ( p − p' ) − p ' (q − q ' ) = 1, ou seja, q ' p − p' q = 1, onde 0 < q ' < q e 0 < p' < p. Como mdc(p, q) = 1 existem x, y ∈ Z EUREKA! N°6, 1999
47
Sociedade Brasileira de Matemática
com px + qy = 1, e teremos para todo k ∈ Z, p( x + kq) + q ( y − kp) = 1. Certamente podemos escolher k de modo que 0 < x + kq < q (note que x não é múltiplo de q, senão 1 = px + qy também seria), e então tomamos q ' = x − kq e p' = kp − y pq '−1 , mas 1 ≤ q' < q, donde 0 ≤ p ' < p). (temos p' = q
17) Dado n ∈ N, uma partição π de n é uma lista ordenada π = (a , a ,...,a ) , r, a , a ,..., a ∈N* com a ≤ a ≤ ...≤ a e a + a + ...+ a = n. 1
2
1
r
2
1
r
2
1
r
2
r
Seja Pn o conjunto das partições de n. Para π ∈ Pn , definimos A(π) como o número de termos iguais a 1 em π ( ou seja , A(π ) = # {i ∈ {1,2,..., r} a i = 1}), e B(π) como o número de termos distintos na partição π (ou seja, B (π) = # {a1, a2, …, ar}). Prove que
∑ A(π ) = π∑ B(π ) para todo n ∈ N.
π ∈Pn
∈Pn
Solução de Zoroastro Azambuja Neto (Rio de Janeiro - RJ): Provaremos o resultado por indução em n. Para n = 1 o resultado é óbvio, pois a ~
única partição de 1 é (1). As partições π = (a ,..., a ) de n + 1 com a = 1 (isto é, 1
1
n
~
com A(π ) > 0) são obtidas das partições π = (a ,..., a ) de n acrescentando um 1 2
r
na primeira posição. Esta operação acrescenta 1 ao A(π) e acrescenta 1 ao B(π) ~
caso A(π) = 0, e 0 caso contrário (ou seja, caso A(π ) ≥ 2). Admitindo o resultado para n, o resultado para n + 1 segue então de ~
~
~
~
~
~
∑ A(π ) − ∑ B(π ) =#{π partição de n + 1 | A(π ) ≥ 2} − ∑ B(π ). ~
π ∈Pn +1
~
π ∈Pn +1
π ∈Pn +1 ~
A (π ) = 0
O lado direito é igual a 0, pois, considerando ~
~
~
~
~
f = {π ∈ Pn +1 | A(π ) ≥ 2} → {π ∈ Pn +1 | A(π ) = 0}, tal que se π = (a1 , a 2 ,...a r ) EUREKA! N°6, 1999
48
Sociedade Brasileira de Matemática ~
~
então os elementos da partição f (π ) são A(π ), a
~
A(π ) +1
,..., a r (somamos todos os
~
~
A(π ) ≥ 2 termos de π iguais a 1), é fácil ver que f é sobrejetiva, e , dado ~
~
~
~
~
π ∈ Pn +1 com A(π ) = 0, # {π '∈ Pn +1 | A(π ) ≥ 2 e f (π ' ) = π } = B(π ), (escolheremos um elemento de π e o decompomos como soma de uns) donde ~
~
~
# {π ∈ Pn +1 | A(π ) ≥ 2} = ~
∑ B(π ).
π ∈Pn +1 ~
A(π ) = 0
20) Diga se existe uma função polinomial de R2 em R cuja imagem seja o intervalo (0,+∞) = {x ∈ R x > 0}.
Solução de Zoroastro Azambuja Neto (Rio de Janeiro - RJ): Considere a função f ( x, y ) = x 2 + ( xy − 1) 2 É fácil ver que f ( x, y ) ≥ 0, ∀( x, y ) ∈ R 2 , e se f ( x) = 0 então x = 0 e xy – 1 = 0, o 1 = a, donde Im (f) = (0, + ∞). que é impossível. Dado a > 0, f a , a
24) Na loteria de Truchilândia, cada bilhete tem um número de três algarismos que usa somente os algarismos 1, 2, 3, 4 (é permitido repetir os dígitos). Um bilhete é ganhador se coincide em pelo menos duas posições com o número sorteado. Um apostador quer compar vários bilhetes, de maneira que um deles ganhe com certeza, mas gastando o mínimo possível. Determinar quantos bilhetes deve comprar e quais bilhetes deve comprar. Obs. Se o bilhete sorteado for o 423 então 123 é um bilhete ganhador, mas 243 não é. Solução de Flávio Wagner Rodrigues (São Paulo - SP): 1. Vamos observar inicialmente que o sorteio dessa loteria admite 64 resultados possíveis. EUREKA! N°6, 1999
49
Sociedade Brasileira de Matemática
2. Com um único bilhete o apostador tem 10 resultados favoráveis. De fato, para cada par de posições 1 e 2, 1 e 3 e 2 e 3 existem quatro bilhetes favoráveis o que nos dá um total de 12 mas, o número de seu bilhete é contado 3 vezes e portanto são apenas 10 resultados favoráveis distintos. 3. Segue-se de 1 e 2 que 6 bilhetes não são suficientes para garantir a vitória do apostador. .Vamos mostrar agora que 7 bilhetes também não são suficientes. Como são quatro algarismos e 7 jogos existe pelo menos um algarismo que aparece uma única vez na primeira posição. Vamos supor que existe um único jogo que tenha o 1 na primeira posição. Esse jogo garante a vitória do jogador em 7 jogos começados com 1. Como existem 16 jogos começados por 1, a vitória nos outros 9 deve ser garantida pelos elementos que aparecem nas posições 2 e 3 dos demais jogos. Como existem apenas 6 outros jogos existirão 3 jogos começados por 1 que se forem sorteados não darão vitória ao jogador.
4. Existem vários conjuntos de oito jogos que resolvem o problema. Vamos dar dois exemplos e mostrar em seguida como esses conjuntos podem ser construídos. 5. Exemplo 1 : 124, 131, 212, 313, 342, 243, 421, 434 Exemplo 2 : 123, 111, 232, 334, 342, 244, 413, 421 Exemplo 3 : 111, 122, 212, 221, 333, 344, 434, 443 Roteiro para construir um conjunto de oito jogos que resolve o problema. Inicialmente vamos observar que o conjunto deverá satisfazer duas condições importantes: 1- Cada algarismo deve aparecer em cada posição exatamente duas vezes. 2- É necessário que existam jogos que contenham elementos repetidos como 11, 22, 33, 44 não importando em que posições. 1- Escolha dois jogos que tenham o 1 na primeira posição e algarismos distintos nas outras posições. Por exemplo 1 3 4 e 1 4 2. Não é difícil mostrar que dos 16 jogos que começam por 1 esses dois jogos garantem a vitória do apostador em 12. Restam 4 jogos para os quais será necessário EUREKA! N°6, 1999
50
Sociedade Brasileira de Matemática
que a vitória seja garantida pelas posições 2 e 3 dos demais jogos. No nosso exemplo, vamos necessitar de jogos terminando em 11, 13, 21 e 23 2- A primeira observação acima e a conveniência de não repetir ligações nos permitem dizer que os dois jogos que faltam devem terminar por 32 e 44. A distribuição dos algarismos que devem ocupar as primeiras posições deve ser feita levando em conta as condições iniciais evitando sempre que possível a repetição de ligações. Uma possibilidade seria : 134, 142, 211, 223, 313, 321, 432, 444
26) Sejam as funções fo (x) = xn e fi (x) = fi – 1(x + 1) – fi inteiros positivos. Prove que, para todo x, fn(x) = n!
– 1(x)
onde x, n e i são
Solução de Bruno Fernandes Cerqueira Leite (São Paulo - SP): Antes de começar, é importante deduzir (pelo Binômio de Newton) que: a ( x + 1) a − x a = a ⋅ x a −1 + x a − 2 + ... (isto será muito usado na demonstração). 2 Agora temos f 0 ( x) = x n e f i +1 ( x) = f i ( x + 1) − f i ( x)
n f ( x) = f 1 ( x) = f 0 ( x + 1) − f 0 ( x) = ( x + 1) n − x n = n ⋅ x n −1 + x n − 2 + ... ⇒ 1 n 2 x n −1 + a1 x n − 2 + a 2 x n −3 + ..., onde a1, a2, …, etc são coeficientes racionais (que não nos interessam).
[
]
f 2 ( x) f1 ( x + 1) f1 ( x) = − = ( x + 1) n−1 + a1 ( x + 1) n−2 + ... − n n n f ( x) − x n−1 + a1 x n−2 + ... = ( x + 1) n−1 − x n−1 + a1 ( x +1) n−2 − x n−2 + ... ⇒ 2 = n n−2 n −1 n−3 n−3 n − 2 n −4 x + ... + a1 (n − 2) x + x + ... + ... (n −1) x + 2 2 f 2 ( x) = f1 ( x + 1) − f1 ( x) ⇒
[
] [
] [
]
f 2 ( x) = x n−2 + b1 x n−3 + b2 x n−4 + ..., onde b1, b2, …, etc, são coeficientes n(n −1) racionais (cuja determinação não nos interessa). ⇒
EUREKA! N°6, 1999
51
Sociedade Brasileira de Matemática
Continuando com esses procedimentos, podemos chegar a expressões análogas para f3(x), f4(x), etc. A semelhança entre essas expressões nos leva a "desconfiar"que
f b ( x) = x n−b + β1 x n−b−1 + β 2 x n−b−2 + ..., onde An,b B j , j = 1,2, etc; são coeficientes (não nos interssam) n! An,b = n(n − 1)(n − 2)... = (n − b)!
b fatores Vamos provar isto por indução. Suponha que a fórmula valha para f1(x), …, fb–1(x). Teremos:
f b ( x) = f b−1 ( x +1) − f b−1 ( x) ⇒
f b ( x) f b−1 ( x +1) f b−1 ( x) = − e por hipótese de An,b−1 An,b−1 An,b−1
indução, f b ( x) = ( x + 1) n−b+1 + α1 ( x + 1) n−b + ... − x n−b+1 + α1 x n−b + ... = ( x + 1) n−b+1 − x n−b+1 + An,b−1
[
][
] [
]
n−b+1 n−b−1 n−b x +... +α1(n−b)xn−b+1 + xn−b+2 +... 2 2
α1[(x +1)n−b + xn−b] +...= (n−b+1)xn−b +
+...⇒
[
]
fb(x) fb(x) = (n −b +1) xn−b + β1xn−b−1 + β2xn−b−2 +... ⇒ = An,b−1 An,b−1(n −b +1)
fb(x) = n! ⋅ (n −b +1) (n −b +1)!
fb (x) f (x) = xn−b + β1xn−b−1 + β2xn−b−2 +... ⇒ b = xn−b + β1xn−b−1 + β2xn−b−2 +..., n! An,b (n − b)! confirmando o que foi "desconfiado". f (x) Agora vamos usar a fórmula para n = xn−n = 1 ⇒ fn (x) = An,n = n! An,n =
EUREKA! N°6, 1999
52
Sociedade Brasileira de Matemática
27) O triângulo equilátero ABC possui um ponto interno P tal que em P chegam três segmentos de reta (PA, PB, PC) onde PA = 6, PB = 8 e PC = 7. Com esses dados descubra qual é a área do triângulo.
A
P B
C
Solução de Ricardo Klein Hoffmann (Porto Alegre - RS): A
6 P 8
α
7
B
C 6
8 D
1. Seja BCD o triângulo congruente ao triângulo APB onde: ___ ___ PB = BD = 8 ___ ___ AP = CD = 6 ___ ___ AB = BC = x ∧
∧
___
2. Temos que A B C = P B D = 60°. Então, o ∆PBD é equilátero com PD = 8. ∧
3. Seja C P D = α 4. ∆(PDC ) → Lei dos cossenos
EUREKA! N°6, 1999
53
Sociedade Brasileira de Matemática
6 2 = 7 2 + 8 2 − 2 ⋅ 7 ⋅ 8 ⋅ cos α ∴ cos α = 5.
∆(BPC ) → Lei dos cossenos
135 11 daqui, senα = 16 16
x 2 = 8 2 + 7 2 − 2 ⋅ 8 ⋅ 7 ⋅ cos(α + 60°) ∴ x 2 =
149 + 63 5 2
x2 3 =S 4 3 149 + 63 5 Então, S = ⋅ 4 2
6. Área ∆( ABC ) =
Portanto, a área do ∆( ABC ) =
3 ⋅ (149 + 63 5 ). 8
28) Seja n ≥ 2 um número inteiro. Prove que n e n + 2 são ambos primos se e somente se
4((n − 1)! + 1) + n é inteiro. n(n + 2)
Solução de Marcelo Rufino de Oliveira (São José dos Campos - SP): Antes de iniciar a resolução, vou enunciar e provar os seguintes Teoremas, que serão de grande utilidade:
1o Teorema (Teorema de Wilson): “Se p é um número primo, então o número (p – 1)! + 1 é divisível por p.” Prova: Consideremos a congruência linear: ax ≡ 1 (mod p) onde a é qualquer dos p – 1 inteiros positivos da seqüência 1, 2, 3, …, p – 1 (1) Então, o mdc (a, p) = 1 e é sabido que esta congruência admite uma única solução módulo p, isto é, existe um único inteiro a1, com 1 ≤ a1 ≤ p – 1, tal que aa1 ≡ 1 (mod p) Se p é primo, temos que a = a1 se e somente se a = 1 ou a = p – 1. De fato, a congruência quadrática a2 ≡ 1 (mod p) é equivalente à seguinte (a – 1)(a + 1) ≡ 0 (mod p) de modo que p | (a – 1) ou p | (a + 1), o que implica que: a – 1 ≡ 0 (mod p) e a = 1 ou a + 1 ≡ 0 (mod p) e a = p – 1 EUREKA! N°6, 1999
54
Sociedade Brasileira de Matemática
Então para cada a distinto da seqüência (1) existe somente um a1 correspondente na seqüência (1) que satisfaz aa1 ≡ 1 (mod p). Omitindo os pares 1 e p – 1, com os p – 3 inteiros restantes: 2, 3, …, p – 3, podemos formar (p – 3)/2 pares a, a1, com a ≠ a1, e tais que aa1 ≡ 1 (mod p). Multiplicando todas essas (p – 3)/2 congruências, obtemos: 2.3.4…(p – 3)(p – 2) ≡ 1 (mod p) => (p – 2)! ≡ 1 (mod p) Multiplicando por p – 1: (p – 1)! ≡ p – 1 (mod p) => (p – 1)! ≡ – 1 (mod p)
2o Teorema: “Uma condição necessária e suficiente para que um número natural p > 1 seja primo é que o número (p – 1)! + 1 seja divisível por p.” Prova: Suponhamos que p não seja primo. Então existe um divisor q de p tal que 1 < q < p. O número (p – 1)! + 1 sendo divisível por p, também deve ser divisível por q. Desde que q < p, então q ≤ p – 1, implicando que q divide algum inteiro positivo entre 2 e p – 1. Assim q divide (p – 1)!. Como q divide (p – 1)! + 1 então q também divide 1, que é uma contradição, pois 1 < q < p. 1a parte (ida) I) Suponhamos que n e n + 2 são números primos. De acordo com o Teorema de Wilson nós temos que (n – 1)! + 1 é divisível por n e (n + 1)! + 1 é divisível por n + 2. Como n divide 4((n – 1)! + 1) e, evidentemente, n também divide n, então divide a sua soma, ou seja, 4((n – 1)! + 1) + n é divisível por n. Basta analisar agora se 4((n – 1)! + 1) + n também é divisível por n + 2. x = 4((n – 1)! + 1) + n = 4((n – 1)! + 1) + n + 2.(n + 1)! – 2.(n + 1)! ⇒ x = 2.(n + 1)! + n + 4 – 2[(n + 1)! – 2.(n – 1)!] ⇒ x = 2[(n + 1)! + 1] + (n + 2) + [(n – 1)!n(n + 1) – 2.(n – 1)!] ⇒ x = 2[(n + 1)! + 1] + (n + 2) + (n – 1)![n2 + n – 2] ⇒ x = 2[(n + 1)! + 1] + (n + 2) + (n – 1)!(n + 2)(n – 1) Como n + 2 divide tanto 2[(n + 1)! + 1], como (n + 2) e também (n – 1)!(n + 2)(n – 1), então divide a sua soma, que é x = 4((n – 1)! + 1) + n. Como n e n + 2 são ambos primos e dividem 4((n – 1)! + 1) + n, então o seu produto também divide 4((n – 1)! + 1) + n. 2a parte (volta) II) Suponhamos agora que para um número natural n > 1, o valor 4((n – 1)! + 1) + n é divisível pelo produto n(n + 2). Se n for par, isto é, se n = 2k, onde k é um número natural, então n – 1 ≥ k. EUREKA! N°6, 1999
55
Sociedade Brasileira de Matemática
Isto implica que (n – 1)! é divisível por k e também que 4.(n – 1)! é divisível por 2k. Consequentemente 4.(n – 1)! é divisível por n, e como 4((n – 1)! + 1) + n = 4.(n – 1)! + n + 4 é divisível por n, então 4 é divisível por n. Como n é par então n = 2 ou n = 4. Entretanto é fácil verificar que 4((n – 1)! + 1) + n não é divisível por n(n + 2) caso n = 2 ou n = 4. Assim, o fato de que 4((n – 1)! + 1) + n é divisível por n implica que (n – 1)! + 1 é divisível por n. Pelo 2o Teorema temos que n é um número primo. Fazendo as mesmas operações da 1a parte da solução: 4((n – 1)! + 1) + n = 2[(n + 1)! + 1] + (n + 2) + (n – 1)!(n + 2)(n – 1) Como 4((n – 1)! + 1) + n, n + 2 e (n – 1)!(n + 2)(n – 1) são todos termos divisíveis por n + 2, então (n + 1)! + 1 também é divisível por n + 2. Pelo 2o Teorema temos que n + 2 também é um número primo.
Agradecemos também o envio das soluções a: Carlos Alberto da Silva Victor (Rio de Janeiro - RJ), Tertuliano Franco (Salvador - BA), Manuel João de Jesus Almeida Rio de Janeiro - RJ) e Daniel Pessôa Martins Cunha (Fortaleza - CE). Seguimos esperando a solução do problema 29. O prazo para recebimento das soluções do problema "Cuático" publicado na revista EUREKA! No. 5 (que concorre a prêmio!) foi prorrogado até dia 15 de fevereiro. Problema "Cuático" Prove que para qualquer conjunto de inteiros positivos A e para todo inteiro positivo k existe um conjunto infinito de números primos S tal que o produto de k elementos distintos de S está sempre em A ou o produto de k elementos distintos de S nunca pertence a A. Errata: No primeiro teste de seleção para a 40a. IMO e a 14a. OIM, publicado na EUREKA! No. 5, pág 50, no problema 4, onde diz: "Por exemplo, min{3, 4} = 4…" deve dizer "Por exemplo, min {3, 4} = 3…"
EUREKA! N°6, 1999
56
Sociedade Brasileira de Matemática
PROBLEMAS PROPOSTOS Convidamos o leitor a enviar soluções dos problemas propostos e sugestões de novos problemas para os próximos números. 30) Determine todas as funções f : R → R que satisfazem as condições: 1 f ( x) (i) f(– x) = – f(x), (ii) f(x + 1) = f(x) + 1, (iii) f = 2 para x ≠ 0. x x 31) Seja x1, x2, x3, … uma seqüência de números reais não negativos satisfazendo x n − 2 x n −1 xn = para n = 3, 4, 5, … Estabeleça condições necessárias e 2 x n − 2 − x n −1 suficientes em x1 e x2 para xn ser inteiro para infinitos valores de n. 32) a) Prove que todo número inteiro não nulo m admite uma única representação da forma m =
n −1
∑σ
k
⋅ 3 k , onde n é um inteiro positivo e σ k ∈ {−1,0,1} para
k =0
todo k, com σ n −1 ≠ 0. b) Dado um conjunto de
3n + 1 pontos V = {P0 , P1 ,..., P3n −1 } , escrevemos em 2 2
cada aresta que une dois desses pontos Pi e Pj (i ≠ j) um número pertencente a {0, 1, …, n – 1} da seguinte forma: escreveremos
i− j =
n −1
∑σ
k
⋅ 3 k , com σ k ∈{−1,0,1}, ∀k , e associamos à aresta Pi Pj o
k =0
número m = min{k ≥ 0 σ k = 1}. Prove que não existe nenhum triângulo cujos vértices pertençam a V com o mesmo número escrito em seus três lados.
33) Na parede interna de um vaso cilíndrico de cristal existe uma gota de mel num ponto B situado a três centímetros do seu bordo superior. Na parede externa, num ponto A diametralmente oposto ao da gota, está uma formiga. Sabendo que a altura do vaso é de 20cm e o seu diâmetro é 10 cm. Indicar o caminho mais curto para que a formiga atinja a gota de mel. 34) ABC é um triângulo, tal que AB = c, AC = b e BC = a. Por um ponto interior P deste triângulo, são traçadas paralelas aos seus lados. EUREKA! N°6, 1999
57
Sociedade Brasileira de Matemática
Sabe-se que as intersecções, da paralela ao lado de medida a, com os lados deste triângulo, determinam um segmento de medida a'. Analogamente, as paralelas aos lados de medidas b e c, determinam com os lados do triângulo, segmentos de medidas b' e c' respectivamente. a ' b' c' Nestas condições demonstre que + + = 2. a b c
35) Sabendo que num triângulo ABC a altura relativa ao vértice A mede 12cm. e a altura relativa ao vértice B mede 20cm., determine todos os valores possíveis para a altura relativa ao vértice C.
Problemas 30 e 31 propostos por Marcelo Rufino de Oliveira (São José dos Campos - SP), problema 33 proposto por Gleydson Chaves Ricarte (Fortaleza - CE), problema 34 proposto por Paulo Jacob (Lista de discussão de problemas), problema 35 proposto por Carlos Alexandre Gomes da Silva (Natal - RN).
Você sabia…
Que ninguém sabe se existem infinitos primos p tais que p + 2 também seja primo (mas todo mundo acredita nisso)? Um para de primos p, p + 2 é chamado um par de primos gêmeos. Os maiores primos gêmeos conhecidos são 361700055 ⋅ 239020 ± 1, que têm 11755 dígitos e foram descobertos este ano por Henri Lifchitz. Também não se sabe se todo número par maior ou igual a 4 pode ser escrito como soma de dois primos (esta é a conjectura de Golbach).
EUREKA! N°6, 1999
58
Sociedade Brasileira de MatemĂĄtica
ASSINATURA DA REVISTA EUREKA! 2000 Se vocĂŞ deseja seguir recebendo na sua casa a revista EUREKA!, faça o seu pedido escrevendo para: Secretaria da OlimpĂada Brasileira de MatemĂĄtica, Estrada Dona Castorina, 110 Jardim Botânico - Rio de Janeiro, RJ CEP: 22460-320. VocĂŞ pode fazer uma assinatura anual o que darĂĄ direito a receber as publicaçþes do ano 2000 (mĂnimo 3 exemplares) por um valor de R$10,00. Para isso, faça um depĂłsito no Banco do Brasil - AgĂŞncia 0598-3 Conta N°52208-2 em nome do professor Eduardo Wagner. Envie-nos a fotocopia da ficha do verso preenchida e uma cĂłpia do recibo do depĂłsito. NĂłs remeteremos a(s) revista(s) pelo correio. Pedidos podem ser feitos tambĂŠm por email e comprovantes de depĂłsito poderĂŁo ser enviados pelo fax. Se tiver qualquer dĂşvida entre em contato conosco. Telefone: 21-5295077 / Fax: 21-5295023 e-mail: obm@impa.br Home-Page: http//www.obm.org.br/
LISTA DE DISCUSSĂ&#x192;O DE PROBLEMAS DE MATEMĂ TICA OLĂ?MPICA EstĂĄ aberta uma lista de discussĂŁo de problemas de MatemĂĄtica OlĂmpica. A lista ĂŠ aberta Ă todos. O endereço da lista ĂŠ: Obm-l@mat.puc-rio.br Para assinar, envie uma mensagem para: Majordomo@mat.puc-rio.br Com texto Subscribe obm-l Quem assim proceder deve receber um pedido de confirmação auto-explicativo (automaticamente gerado pelo majordomo em inglĂŞs). Em caso de problemas, escreva para o Professor Nicolau Saldanha (administrador desta lista): e-mail: nicolau@mat.puc-rio.br Desta lista estĂŁo participando muitos alunos e professores e nela sĂŁo discutidos problemas e aspectos teĂłricos de matemĂĄtica. Trata-se de um meio informal e eficiente de preparação para as olimpĂadas.
EUREKA! N°6, 1999
59
Sociedade Brasileira de Matemática
ASSINATURA 2000 DA REVISTA EUREKA! Atenção! Leia atentamente as instruções da página anterior. Por favor preencha esta ficha com letra de forma.
Nome: Endereço: Bairro: Cidade: Cep: Telefone: ( Fax: ( e-mail:
Estado: ) )
Números atrasados podem ser comprados utilizando a mesma forma de pagamento da assinatura, sendo os preços: ◆ ◆ ◆
Números 1, 2 e 3 Números 4, 5 e 6 Números por separado
Você sabia…
R$10,00 R$10,00 R$4,00 (cada)
que ninguém sabe se para
todo n ∈ N existe k ∈ N tal que a k ( n) = 1 , onde a1 ( n) = n e
n / 2, se n é par a k +1 (n) = 3n + 1, se n é ímpar
?
(Isto é conhecido como o problema 3 x + 1, e a resposta é afirmativa para todo n ≤ 2 40 )
EUREKA! N°6, 1999
60
Sociedade Brasileira de Matemática
COORDENADORES REGIONAIS Amarisio da Silva Araújo Alberto Hassen Raad Antônio C. Rodrigues Monteiro Angela Camargo Benedito T. Vasconcelos Freire Claudio Arconcher Clauss Haetinger Crescêncio das Neves Élio Mega Enzo Marcom Takara Flávia Jerônimo Barbosa Florêncio F. Guimarães Filho Francisco Dutenhefner Gisele de A. Prateado Gusmão Ivanilde H. Fernandes Saad João Benício de Melo Neto João F. Melo Libonati Jorge Ferreira José Carlos Pinto Leivas José Cloves Saraiva José Gaspar Ruas Filho José Luis Rosas Pinho José Paulo Carneiro José Vieira Alves Leonardo Matteo D'orio Licio Hernandes Bezerra Luzinalva M. de Amorim Marcondes Cavalcante França Pablo Rodrigo Ganassim Paulo H. Cruz Neiva de L. Jr. Reinaldo Gen Ichiro Arakaki Ricardo Amorim Roberto Vizeu Barros Sergio Claudio Ramos Seme Gebara Neto Tadeu Ferreira Gomes Tomás Menéndez Rodrigues Valdenberg Araújo da Silva Wagner Pereira Lopes Waldemar M. Canalli
(UFV) Viçosa - MG (UFJF) Juiz de Fora - MG (UFPE) Recife - PE (Centro de Educação de Adultos - CEA) Blumenau - SC (UFRN) Natal - RN (Col. Leonardo da Vinci) Jundiaí - SP (UNIVATES) Lajeado - RS (UFAM) Manaus-AM (Col. ETAPA) São Paulo - SP (Col. Singular) Santo André - SP (UFPB Campus I) João Pessoa - PB (UFES) Vitória - ES (UFMG) Belo Horizonte - MG (UFGO) Goiânia - GO (U. Católica Dom Bosco) Campo Grande - MS (UFPI) Teresina - PI (Grupo Educ. IDEAL) Belém - PA (UEM) Maringá - PR (UFRG) Rio Grande - RS (UFMA) São Luis - MA (ICMC-USP) São Carlos - SP (UFSC) Florianópolis - SC (Univ. Santa Úrsula) Rio de Janeiro - RJ (UFPB) Campina Grande - PB (Sistema Titular de Ensino)Belém - PA (UFSC) Florianópolis - SC (UFBA) Salvador - BA (UF Ceará) Fortaleza - CE (L. Albert Einstein) Piracicaba - SP (Esc. Tec.Everardo Passos) SJ dos Campos - SP (INPE) SJ dos Campos - SP (Centro Educ. Logos) Nova Iguaçu - RJ (Colégio ACAE) Volta Redonda - RJ (IM-UFRGS) Porto Alegre - RS (UFMG) Belo Horizonte - MG (U. do Estado da Bahia) Juazeiro - BA (U. Federal de Rondonia) Porto Velho - RO (U. Federal de Sergipe) São Cristovão - SE (Esc. Tec. Fed. de Goiás) Jataí - GO (P.M. S. João de Meriti) S. João de Meriti - RJ
EUREKA! N°6, 1999
61